Financial Accounting and Reporting I VOL I

You might also like

Download as pdf or txt
Download as pdf or txt
You are on page 1of 332

CAF-05

Financial Accounting
and Reporting I
Vol. I

Practice Manual Examination


Questions & Answers

Book Contains Translations of


QURANIC AYATS & AHADITHS,
Therefore Handle Carefully.
Table of Contents
Chapter Topic Page No.

1 REVALUATION OF PROPERTY, PLANT AND 01


EQUIPMENT

STATEMENT OF CHANGES IN EQUITY 44


2

IAS 23 BORROWING COST 57


3

STATEMENT OF CASH FLOW 103


4

RATIOS 175
5

6 IAS-8 233

7 ALL TEST INCLUDING SPRING 2020 276


Revaluation of Property, Plant and Equipment:
Before Revaluation
Discussion of Retained earnings

Suppose: First year of Business of a sole proprietor:


Capital Account
b/d --
Drawings 80,000 Cash 500,000
Profit 150,000
c/d 570,000

Second year of Business


b/d 570,000
Drawings 130,000 Profit 125,000
c/d 565,000

COMPANY
(First year of business)
Capital Account
1st year
b/d --
Cash 500,000
c/d 500,000

Retained Earnings Account


b/d --
Dividend 80,000 Profit 150,000
c/d 70,000

Dividend = Drawings by Owner of Company


Statement of financial position
At the end of 1st year
Non current assets XX
Current assets XX
XX
Capital & Liabilities:
Capital 500,000
Retained Earnings 70,000
570,000

1 Page 1 of 43
Second year of business
Capital Account
b/d 500,000
c/d 500,000

Retained Earnings Account


b/d 70,000
Dividend 130,000 Profit 125,000
c/d 65,000

Revaluation Model

Subsequent Measurement: (Measurement after initial recognition)


Cost model Revaluation Model
Cost - Revalued Amount -
Less Acc Depreciation (-) Less Acc Depreciation (-)
Carrying Amount __-__ Carrying Amount __-__

Revaluation Model
If revaluation model is adopted, carrying amount is compared with fair value at the date of revaluation.
For example:
1. Revaluation Date is 30-6-2010
Machine
Cost 500,000
Less Acc depreciation (150,000)
Carrying Amount 350,000
Fair value 600,000
There is an unrealized gain or expected gain of Rs 250,000 which is called as revaluation surplus.
Machine 250,000
Revaluation surplus 250,000
(This entry has no effect on profit or loss for the year)
Revaluation surplus is presented within equity in statement of financial position.

2) Revaluation date is 30-6-2015

Building
Cost 1,500,000
Less Acc depreciation (400,000)
Carrying Amount 1,100,000
Fair value 150,000

There is an expected loss of Rs 950,000 which is termed as revaluation loss.

2 Page 2 of 43
Revaluation loss 950,000
Building 950,000
Revaluation loss is recognized in statement of profit or loss in the period in which it arises.
Conclusion: As a result of revaluation there could be a revaluation surplus or revaluation loss. After revaluation,
value of asset is equal to its fair value. After revaluation, fair value is depreciated over the asset’s remaining useful
life.

Example
1st revaluation as on 30-6-2011; Revaluation surplus = Rs 300,000
Machine 300,000
Revaluation surplus 300,000
2nd revaluation as on 30-6-2013
(i) Ist scenario; Revaluation loss = Rs 200,000
(ii) 2nd scenario; Revaluation loss = Rs 350,000
Important point to remember
Revaluation is not compulsory every year as per IAS-16.Revaluation entries are only required whenever there is a
material difference between carrying amount and fair value.
If there is a loss on revaluation of an asset in an accounting period but there is a balance of any revaluation surplus
related to same asset because of any previous revaluation then first adjust the loss against the surplus and the
balance of loss (if any) is recognized in statement of profit or loss.
Keeping in view the above point; adjusting entries will be;
Scenario i
Revaluation Surplus 200,000
Machinery 200,000
(No effect of this entry on statement of profit or loss)

Scenario ii
Revaluation Surplus 300,000
Revaluation Loss (bal) 50,000
Machinery 350,000
(Rs 50,000 will be recognized in statement of profit or loss)

Treatment of Acc Depreciation at the time of revaluation:


Machinery as on 30-6-2013
Cost 500,000
Less Acc depreciation (150,000)
Carrying Amount 350,000
Fair value 250,000
Revaluation loss 100,000

The adjustment of accumulated depreciation is only required if the asset account and accumulated depreciation
accounts are separately prepared. In such a situation, asset is revalued in two steps:
a) Eliminate the existing accumulated depreciation against the amount stated against the asset.
Acc Depreciation 150,000

3 Page 3 of 43
Machinery 150,000
(The effect of this adjustment is that asset account is converted into WDV)
b) Restate the balance of WDV upto fair value.
Revaluation loss 100,000
Machinery 100,000
Entry (a) always remains same while entry (b) can be different depending on whether there is surplus or loss.

Another Example
Cost of Equipment 1,500,000
Less Acc depreciation (700,000)
Carrying Amount 800,000
Fair value 1,100,000
Revaluation surplus 300,000
Prepare accounting entries related to revaluation:

a)
Acc Depreciation 700,000
Equipment 700,000
b)
Equipment 300,000
Revaluation surplus 300,000

 Before any further discussion please recall the calculation of retained earnings:
Opening balance of retained earnings -
Profit for the period -
Less: Dividend for the period (-)
Closing balance of retained earnings -

When the revaluation surplus is realized; (i.e transferred to retained earnings)


There are two methods:
a) Transfer the surplus (in full) when the asset is or b) Transfer the surplus as the asset is used and
disposed off depreciated (annually).

1) Transfer the surplus (in full) when the asset is disposed off:
Example
A machinery having a revalued amount of Rs 260,000 and accumulated depreciation amounting to Rs 13,000 is
sold for Rs 253,000 on 30-6-2013. On this date, balance of revaluation surplus is 75,000.
Required:
Prepare necessary journal entries to account for the disposal.
Solution
a)
Cash 253,000
Acc Depreciation 13,000
Machinery 260,000
Gain (bal) 6,000

4 Page 4 of 43
(Rs 6,000 is treated as an other income in income statement)

b)

Revaluation Surplus 75,000


Retained Earnings 75,000
(Transfer of whole surplus to retained earnings on disposal but not through statement of profit or loss)

Movement of retained earnings and presentation of transfer of revaluation surplus:


Opening balance of retained earnings -
Profit after tax -
Transfer of surplus -
Dividend for the period (-)
Closing balance of retained earnings -

Another Example:
An equipment has following values at the time of disposal:
Revalued Amount 600,000
Acc Depreciation (150,000)
450,000
Revaluation Surplus 300,000
It is sold for Rs 325,000
a)
Cash 325,000
Acc Depreciation 150,000
Loss 125,000
Equipment 600,000
b)
Revaluation Surplus 300,000
Retained Earnings 300,000

2) Transfer the surplus annually:


In this case surplus is transferred to retained earnings as the asset is depreciated annually. The basis of
calculation of amount of transfer should be exactly similar to the basis of calculation of depreciation. The entry
of transfer remains same as discussed earlier; i.e
Revaluation Surplus Xxx
Retained Earnings xxx

If the question is silent regarding the policy of transfer of surplus then transfer the surplus as the asset is used and
depreciated (to adjusted the effect of extra depreciation annually; otherwise if the policy of transfer of surplus is
on disposal then that effect will be adjusted on disposal).

Revaluation model:
1) This model involves revaluing the asset’s carrying amount to its fair value (FV) (Also known as revalued
amount).
2) If FV is more than carrying amount then revaluation surplus
3) If FV is less than carrying amount revaluation loss

5 Page 5 of 43
4) If there is revaluation surplus already in existence for an asset because of a previous revaluation, then
subsequent revaluation loss is adjusted against surplus. If loss is more than surplus, difference is recorded in
income statement.
5) Depreciation should be charged on cost of asset. If there is a surplus then effect of extra depreciation is
transferred to retained earnings. If there is a revaluation loss then effect of less charged depreciation should
be charged when there is a subsequent surplus in future.
6) After revaluation, revalued amount (FV) is depreciated over remaining useful life.
7) When revaluation surplus is realized (means transferred to retained earnings)
a) At the time of disposal/end of useful life; or
b) As the asset is used by the entity and depreciated (period wise)

If the question is silent then follow the (b) policy.


8) Treatment of accumulated depreciation at the time of revaluation:
a) Eliminate against cost (net replacement value method)
b) Restate up to revalued amount
9) If an asset is revalued, then all the assets in the class of asset need to be revalued. The following are examples
of classes of property, plant and equipment.
(i) Land
(ii) Building
(iii) Plant and Machinery
(iv) Motor Vehicles
(v) Furniture & Fixtures
(vi) Office Equipment.
10) Revaluation is not compulsory annually for items of property, plant and equipment carried out at revaluation
model. Instead revaluation is only required whenever there is a material differences between fair value and
carrying amount. [Para 34]
11) As the land is not depreciated in normal circumstances therefore its surplus is transferred at the time of
disposal.

Q.1 Faraday Pharmaceutical Limited (FPL) acquired a building for Rs. 200 million on July 1, 2005. The following
information relating to the building is available:
1) It is being depreciated on the straight line basis, over 20 years.
2) FPL uses the revaluation model for subsequent measurement of its property, plant and equipment and
accounts for revaluations on the *net replacement value method. The details of revaluation carried out by
the independent valuer during the past years are as follows:
Revaluation date Fair value
Rupees in million
July 1, 2006 230
July 1,2007 170
July 1, 2008 180
3) FPL transfers the maximum possible amount from the revaluation surplus to retained earnings on an annual
basis.
4) There is no change in the useful life of the building.

Required:
a) Prepare the journal entries to record the above transactions from the date of acquisition of the building to the
year ended June 30, 2009.
b) Prepare following ledgers for all relevant years.
 Building Account
 Accumulated Depreciation Account
 Revaluation Surplus Account
*Net replacement value method simply means eliminate the existing accumulated depreciation against cost.

6 Page 6 of 43
Q.2 Following information is available to you

1) Cost of plant is Rs. 360,000 purchased on 1 July 2009.


2) It is depreciated on straight line basis over a period of 10 years.
3) Detail of revaluation was as follows
Revaluation Date FV
1 July 2010 400,000
1 July 2011 280,000
1 July 2012 290,000
Required: Prepare following ledgers from the date of purchase upto the year ended 30-6-2013.
a) Plant Account
b) Accumulated Depreciation Account
c) Revaluation Surplus Account

Q.3 Awesome Industries Limited (AIL) manufactures components for textile machinery. It purchased a plant on 1 July
2008 at a cost of Rs. 200 million. It has an estimated useful life of five years and no residual value.
AIL revalues its plant on an annual basis. The details of revaluations performed by Supreme
Valuation Service, an independent firm of valuer, are as follows:

Fair value - 1 July 2009 Rs. 180 million


Fair value - 1 July 2010 Rs. 108 million
Fair value - 1 July 2011 Rs. 88 million

Required:
Prepare journal entries for the year ended 30 June 2009, 2010, 2011 & 2012.

Q.4 MWL is a listed company and is engaged in the business of purifying and marketing of bottled water.
MWL purchased a bottling plant on 1-7-2006 at a cost Rs 90 million. The plant has a useful life of 10 years with no
residual value. Deprecation is charged on straight line method over plants’ useful life. MWL revalues its plant at
the end of every two years.
The revalued amounts determined by Jet Values, an independent firm of valuers are as follows
i. On 30-6-2008 : Rs 64 million
ii. On 30-6-2010 : Rs 60 million

However there was no change in the expected useful life and residual value of the plant since the purchase of
bottling plant

Required:
Prepare the relevant accounting entries up to the year end 30-6-2011 from the date of purchase of Bottling Plant

Important Points to remember


Entry of elimination of accumulated depreciation On the date of revaluation

Depreciation on revalued amount (FV) After revaluation on the basis of remaining life

Transfer of surplus After revaluation in the periods in which depreciation is


charged on the basis of revalued amount.

7 Page 7 of 43
Disclosures of property, plant and equipment:
 Schedule of PPE.
 Measurement basis
 Useful life / rate of depreciation
 Method of depreciation
 Detail of disposal of an item of PPE.

Disclosure requirements if an item of property, plant and equipment is revalued.


When items of property, plant and equipment are stated at revalued amounts the following must be disclosed:
 The effective date of the revaluation (if there are more than one revaluation dates then the latest date);
 Whether an independent valuer was involved and its name if available;
 For each revalued class of property, plant and equipment, the carrying amount that would have been
recognised had the asset been carried under the cost model; and
 The revaluation surplus, indicating the change for the period and any restrictions on the distribution of the
balance to shareholders.

Point to remember:
Revaluation surplus is presented in other comprehensive income in statement of comprehensive income.
Solutions:
A.1 Faraday
a) Journal Entries
Debit Credit
Date Particulars
Rs. in,000 Rs. in,000
1-Jul-05 Building 200,000
Bank 200,000
(Record purchase of plant)
30-Jun-06 Depreciation 10,000 10,000
Accumulated depreciation – Building
(Record depreciation for the year 2005-6)
Working: Rs. 200,000 20 = Rs. 10,000
1-Jul-06 Accumulated depreciation – Building 10,000
Building 10,000
(Reversal of prior year depreciation)
1-Jul-06 Building 40,000
Surplus on revaluation of fixed assets 40,000
(Increase in value through revaluation)
Working: Rs. 230,000 - Rs. 190,000 = Rs. 40,000
30-Jun-07 Depreciation 12,105
Accumulated depreciation - Building 12,105
(Record depreciation for the year 2006- 7)
Working: Rs. 230,000 19 = Rs. 12,105
30-Jun-07 Surplus on revaluation of fixed assets 2,105
Retained earnings/Profit & loss account 2,105
(transfer of surplus through retained earning to the extent of excess
depreciation)

8 Page 8 of 43
Working: Rs. 40,000 - 19 = Rs. 2,105
l-Jul-07 Accumulated depreciation - Building 12,105
Building 12,105
(Reversal of prior year depreciation)
l-Jul-07 Surplus on revaluation of fixed assets 37,895
Revaluation expense 10,000
Building 47,895
(Decrease in value through revaluation)
Working:
Reversal of Surplus balance (Rs. 40,000 - Rs. 2,105) Rs. 37,895.
Balancing figure of Rs. 10,000 charged to Profit and Loss
Building value decline: (Rs. 230,000 - Rs. 12,105) - Rs. 170,000 =Rs.
47,895
30-Jun-08 Depreciation 9,444
Accumulated depreciation - Building 9,444
(Record depreciation for the year 2007-8)
Working: Rs. 170,000 18 = Rs. 9,444
l-Jul-08 Accumulated depreciation - Building 9,444
Building 9,444
(Reversal of prior year depreciation)
l-Jul-08 Building 19,444
Reversal of Revaluation Loss 9,444
Surplus on revaluation of fixed assets (balancing) 10,000
(Reversal of prior year Revaluation Loss)
Working:
Revaluation income = Rs. 10,000 - [ Rs. 10,000 - Rs. 9,444] = Rs. 9,444
Building: [Rs. 170,000 - Rs. 9,444] - Rs. 180,000 =Rs. 19,444
30-Jun-09 Depreciation 10,588
Accumulated depreciation - Building 10,588
(Record depreciation for the year 2007-8)
Working: Rs. 180,000 17 = Rs. 10,588
30-Jun-09 Surplus on revaluation of fixed assets 588
Retained earnings 588
(Reverse the excess depreciation)
Working: Rs. 10,000 17 = Rs. 588

b)
Ledger Accounts
Building (cost)
Bank 200,000,000
C/D 200,000,000
B/D 200,000,000 Acc Dep 10,000,000
Revaluation 40,000,000
Surplus
C/D 230,000,000
B/D 230,000,000 Acc Dep 12,105,263
Revaluation Surplus 37,894,737
Revaluation Loss 10,000,000
C/D 170,000,000

9 Page 9 of 43
B/D 170,000,000 Acc Dep 9,444,444
Revaluation 10,000,000
Surplus (W-1)
Reversal of Loss 9,444,444
C/D 180,000,000

Accumulated Depreciation
C/D 10,000,000 Depreciation 10,000,000
(200 ÷ 20)
Building 10,000,000 B/D 10,000,000
C/D 12,105,263 Depreciation 12,105,263
(230 ÷ 19)
Building 12,105,263 B/D 12,105,263
C/D 9,444,444 Depreciation 9,444,444
(176 ÷ 18)
Building 9,444,444 B/D 9,444,444
C/D 10,588,235 Depreciation 10,588,235
(180 ÷ 17)

Revaluation Surplus

- -
R/E 2,105,263 Building 40,000,000
C/D 37,894,737
Building 37,894,737 B/D 37,894,737
C/D -
R/E 588,235 B/D -
C/D 9,411,765 Building 10,000,000

The amount of transfer of surplus can also be calculated as the difference between depreciation based on revalued
amount and the asset’s cost. Therefore sometimes in questions, words “incremental depreciation on account of
revaluation” will have same meaning i.e transfer that amount from revaluation surplus to retained earnings.

A.2
Answer
Plant
1-7-2009
Cash 360,000
C/D 360,000
B/D 360,000 Acc Depreciation 36,000
Revaluation Surplus 76,000
C/D 400,000
B/D 400,000 Acc Depreciation 44,444
Revaluation Surplus 67,556

10 Page 10 of 43
Revaluation Loss 8,000
C/D 280,000
B/D 280,000 Acc Depreciation 35,000
Reversal of loss (W-1) 7,000
Revaluation Surplus 38,000
C/D 290,000

Accumulated Depreciation
Depreciation 36,000
C/D 36,000
Plant 36,000 B/D 36,000
Depreciation 44,444
C/D 44,444
Plant 44,444 B/D 44,444
Depreciation 35,000
C/D 35,000
Plant 35,000 B/D 35,000
Depreciation 41,429
C/D 41,429

Revaluation Surplus
Retained Earnings 8,444 Plant 76,000
C/D 67,556
Plant 67,556 B/D 67,556
C/D -
Retained Earnings 5,429 B/D -
Plant 38,000
C/D 32,571

A.3
Awesome Industries
1-7-2008
Plant 200 M
Cash 200 M
30-6-2009
Depreciation 40 M
Acc Depreciation 40 M (200 M ÷ 5)
1-7-2009
Acc Depreciation 40 M
Plant 40 M
1-7-2009
Plant 20 M
Revaluation Surplus 20 M (180- 160)
30-6-2010

11 Page 11 of 43
Depreciation 45 M
Acc Depreciation 45 M (180M÷4)
30-6-2010
Revaluation Surplus 5M
Retained Earnings 5M (20M÷4)
1-7-2010
Acc Depreciation 45 M
Plant 45M
1-7-2010
Revaluation Surplus 15 M
Revaluation Loss 12 M
Plant 27 M [(180 – 45) -108]
30-6-2011
Depreciation 36 M
Acc Depreciation 36 M (108÷3)
30-6-2011
No transfer as no surplus

1-7-2011
Acc Depreciation 36 M
Plant 36 M
1-7-2011
Plant 16 M
Revaluation Surplus 8M
Reversal of loss 8M

30-6-2012 Depreciation 44 M
Acc depreciation 44 M (88÷2)

30-6-2012 Revaluation Surplus 4M


Retained Earning 4M (8÷2)

A.4

Solution
MWL
Journal Entries
1-7-2006 Rs 000 Rs 000
Plant 90,000
Cash/Bank 90,000
30-6-2007
Depreciation 9,000
Acc Depreciation 9,000
(90,000÷10)
30-6-2008

12 Page 12 of 43
Depreciation 9,000
Acc Depreciation 9,000
30-6-2008
Acc Depreciation 18,000
Plant 18,000
30-6-2008
Revaluation Loss 8,000
Plant 8,000
(WDV (90,000 – 18,000) = 72,000
FV = 64,000)
30-6-2009
Depreciation 8,000
Acc Depreciation 8,000
(64,000÷8)
30-6-2010
Depreciation 8,000
Acc Depreciation 8,000
30-6-2010
Acc Depreciation 16,000
Plant 16,000
30-6-2010 Plant 12,000
Reversal of loss (W-1) 6,000
Revaluation Surplus (W-1) 6,000

30-6-2011
Depreciation 10,000
Acc Depreciation 10,000
(60,000÷6)
30-6-2011
Revaluation Surplus 1,000
Retained Earning 1,000
(6000 ÷ 6)

Self-Test Questions (Revaluation)

Q.1
Rupees
Cost of machine at 1/1/20X1: 100,000
Fair value
• 1/1/20X2 180,000
• 1/1/20X3 60,000
• 1/1/20X4 77,000
• 1/1/20X5 120,000
Depreciation: 10% per annum to a nil residual value
The company’s policy is to transfer the realized portion of the revaluation surplus to retained earnings as the asset
is used.

13 Page 13 of 43
Required:
Prepare the asset’s account as a net carrying amount account (i.e. do not prepare separate cost and accumulated
depreciation accounts) as well as revaluation surplus accounts for the year ended 31-12-2001 to 31-12-2005.

Q.2 A company always revalues its fixed assets to fair value using net replacement vale method. Following
information relates to its specialized vehicle.

Date Rupees
Purchase price 1 July 2005 500,000
Fair value 1 January 2007 420,000
Fair value 1 January 2009 165,000

Useful life: five year


Required:
a) Journalize the above transaction from the date of Purchase to 31 December 2009
b) Prepare for the year ended 31 December 2009, Property plant and equipment schedule in notes to
financial statements (including comparative figures).

Solutions
Self-Test Questions (Revaluation)

A.1
Machinery Account – WDV
1-1-2001
Bank 100,000 Depreciation 10,000
C/D 90,000
B/D 90,000 Depreciation 20,000
Revaluation Surplus 90,000 (180,000 ÷ 9)
C/D 160,000
B/D 160,000 Revaluation Surplus 80,000
Revaluation Loss 20,000
Depreciation 7,500
(60,000 ÷ 8)
C/D 52,500
B/D 52,500 Depreciation 11,000
(77,000 ÷ 7)
Reversal of Loss (W-1) 17,500
Revaluation Surplus 7,000
C/D 66,000
B/D 66,000 Depreciation 20,000
(120,000 ÷ 6)
Revaluation Surplus (W-2) 54,000

C/D 100,000

14 Page 14 of 43
W-1 WDV=52,500 W-2 WDV=66,000
HCA=70,000 FV=120,000
FV=77,000

Revaluation Surplus
1-1-2001 - -
C/D - B/D -
Retained Earnings 10,000 Machine 90,000
(90,000 ÷ 9)
C/D 80,000
Machine 80,000 B/D 80,000
C/D -
Retained Earnings 1,000 B/D -
(7,000÷7)
Machine 7,000
C/D 6,000
Retained Earnings 10,000 B/D 6,000
(60,000÷6)
Machine 54,000
C/D 50,000

A.2

a) Journal Entries
1-7-2005
Vehicles 500,000
Bank 500,000
31-12-2003
Depreciation 50,000
Acc Depreciation 50,000
(500,000÷5 x6/12)
31-12-2006
Depreciation 100,000
Acc Depreciation 100,000
(500,000÷5)

1-1-2007
Acc Depreciation 150,000
Vehicle 150,000
1-1-2007
Vehicle 70,000
Revaluation Surplus 70,000
31-12-2007
Depreciation 120,000

15 Page 15 of 43
Acc Depreciation 120,000
(420,000÷3.5)
31-12—2007
Revaluation Surplus 20,000
Retained Earnings 20,000
(70,000÷3.5)
31-12-2008
Depreciation 120,000
Acc Depreciation 120,000
31-12-2008
Revaluation Surplus 20,000
Retained Earnings 20,000
(70,000÷3.5)
1-1-2009
Acc Depreciation 240,000
Vehicle 240,000
1-1-2009
Revaluation Surplus 15,000
Vehicle 15,000
31-12-2009
Depreciation 110,000
Acc Depreciation 110,000
(165,000 ÷ 1.5)
31-12-2009
Revaluation Surplus 10,000
Retained Earnings 10,000
(15,000 ÷ 1.5)

b)
Notes to Financial Statements
For the year ended 31-12-2009
Vehicles 2009 2008
Cost/Revalued Amount
Opening Balance 420,000 420,000
Revaluation Surplus/Loss (15000) -
Adjustment of revaluation (240,000) -
Closing Balance 165,000 420,000

Acc Depreciation
Opening Balance 240,000 120,000
Adjustment of revaluation (240,000) -
Depreciation for the year 110,000 120,000
Closing Balance 110,000 240,000
Carrying Amount 55,000 180,000

16 Page 16 of 43
Extra questions of revaluation
Question No. 1
On 1st January 2014 Omega Chemicals Limited (OCL) changed its valuation model from cost to revaluation for its
buildings. The following information pertains to its buildings as at 01-01-2014.

Estimated useful life Revalued amounts


Accumulated
as originally Cost as per valuation
depreciation
estimated report
------------------ Rs. In million ---------------
Factory buildings 20 years 100.00 37.50 52.00
Office buildings 25 years 164.50 26.32 149.94

As per the report of the professional valuer, the was no change in estimated useful life of the buildings:
On 1 July 2014, one of the office buildings was sold for Rs. 30 million. On 01-01-2014 written down value before
valuation and revalued amount of the sold building amounted to Rs. 27.72 million and Rs. 31.92 million
respectively.
On 31 December 2014, factory buildings were revalued at Rs. 64 million where as there was no change in value of
the office buildings.
OCL uses straight line method of depreciation which is charged from the date the asset is available for use upto the
date of disposal. Revaluation is to be accounted for by using net replacement value method.
Required:
In the light of the requirements of the international financial reporting standards, prepare accounting entries from
the above information for the year ended 31 December 2014. (17)
Question No. 2
Shahzad Textile Mills Limited (STML) purchased a plant for Rs. 500 million on 1 July, 2010. the plant has an
estimated useful life of 10 years and no residual value.
STML uses revaluation model for subsequent measurement of its property, plant and equipment and accounts for
revaluations on net replacement value method. The details of revaluations performed by an independent firm of
valuers are as follows:

Revaluation date Fair value


1 July 2011 Rs. 575 million
1 July 2012 Rs. 390 million
1 July 2013 Rs. 380 million

Required:
Prepare journal entries to record the above transactions from the date of acquisition of the plant to the year
ended 30 June 2014. (Ignore tax implications) (15)
Question 3
PQR Enterprises was incorporated on 1 July 2012. The company depreciates its property, plant and equipment on
straight line basis over their useful life. It uses revaluation model for subsequent measurement of the property,
plant and equipment and has a policy of revaluing these after every two years.

17 Page 17 of 43
Following information pertains to its property, plant and equipment:

Value as determined
Cost as on 01- WDV as on
by professional valuer Useful life in years
07-2013 01-07-2013
on 30-06-2014
Assets
Remaining as
Original at
-------------- Rs. In million ------------------ determined by
acquisition
valuer
Office building 6,000 5,500 5,750 12 8
Factory building 4,400 3,960 3,320 10 9
Warehouse 4,500 4,050 3,350 10 8

During the year there were no addition or deletion in the above assets.
As per policy, PQR transfers the maximum possible amount from the revaluation surplus to retained earnings on
an annual basis.
Required:
Prepare necessary journal entries for the year ended 30 June 2014 and 2015. (12)
Question 4
The following information pertains to Sherdil Limited (SL):
(i) Buildings and equipment were acquired on 1 January 2014 for Rs. 450 million and Rs. 50 million
respectively.
(ii) The relevant information relating to both assets is summarised below:

Subsequent
Assets Depreciation method Life/rate
measurement
Buildings Straight line 20 years Annual revaluation
Equipment Reducing balance 10% Cost

SL transfers the maximum possible amount from revaluation surplus to retained earnings on an annual
basis.
(iii) The revalued amount of buildings as determined by Accurate Valuers (Private) Limited, an independent
valuation company, on 1 January 2015 and 2016 was Rs. 456 million and Rs. 378 million respectively.
(iv) Equipment costing Rs. 35 million was purchased on 1 August 2015. Half of the equipment purchased on 1
January 2014 was disposed off on 30 June 2016.
Required:
Prepare Building A/c and Equipment A/c as well as separate ledgers for accumulated depreciation for the year
ended 31-12-2015 and 31-12-2016.
Question 5
The following information pertains to Sherdil Limited (SL):
(i) Buildings and equipment were acquired on 1 January 2014 for Rs. 450 million and Rs. 50 million
respectively.

18 Page 18 of 43
(ii) The relevant information relating to both assets is summarised below:

Subsequent
Assets Depreciation method Life/rate
measurement
Buildings Straight line 20 years Annual revaluation
Equipment Reducing balance 10% Cost

SL transfers the maximum possible amount from revaluation surplus to retained earnings on an annual
basis.
(iii) The revalued amount of buildings as determined by Accurate Valuers (Private) Limited, an independent
valuation company, on 1 January 2015 and 2016 was Rs. 456 million and Rs. 378 million respectively.
(iv) Equipment costing Rs. 35 million was purchased on 1 August 2015. Half of the equipment purchased on 1
January 2014 was disposed off on 30 June 2016.
Required:
In accordance with International Financial Reporting Standards, prepare a note on ‘Property plant & equipment’
(including comparative figures) for inclusion in SL’s financial statements for the year ended 31 December 2016. (18)
Question No. 6
Abid Limited (AL) uses the revaluation model for subsequent measurement of its property, plant and equipment
and has a policy of revaluing its assets on an annual basis using the net replacement value method.
The following information pertains to AL’s buildings:
(i) Four buildings were acquired in same vicinity on 1 January 2012 at a cost of Rs. 300 million. The useful life
of the buildings on the date of acquisition was 20 years.

(ii) AL depreciates buildings on the straight line basis over their useful life.

(iii) The results of revaluations carried out during the last three years by premier Valuation Service, an
independent firm of valuers, are as follows:

Fair value
Revaluation date
Rs. In million
1 January 2013 323
1 January 2014 252
1 January 2015 272

(iv) On 30 June 2015, one of the buildings was sold for Rs. 80 million.

Required:
Prepare a note on “property, plant and equipment” (including comparative figures) for inclusion in AL’s financial
statements for the year ended 31 December 2015 in accordance with International Financial Reporting Standards.
(Ignore taxation) (13)

19 Page 19 of 43
Solution:
Answer No. 1
Omega Chemicals Limited
Accounting entries for the year ended 31 December 2014

Debit Credit
Date Description
Rs. ‘000’ Rs. ‘000’
Factory buildings
1-1-2014 Accumulated depreciation - Factory buildings 37.50
Factory buildings 37.50
(Reversal of accumulated depreciation on revaluation of factory
buildings on 31 December 2013)
1-1-2014 Revaluation loss [52 – (100 – 37.5) 10.50
Factory buildings 10.50
(2013 Revaluation loss of factory buildings accounted for in
2014)
31-Dec-2014 Depreciation expense (52 ÷ 12.5 W-1) 4.16
Accumulated depreciation – Factory buildings 4.16
(Depreciation expenses for the year ended 31 December 2014)
31-Dec-2014 Accumulated depreciation – Factory buildings 4.16
Factory buildings 4.16
(Reversal of accumulated depreciation on revaluation of factory
buildings on 31 December 2014)
31-Dec-2014 Factory buildings [64 – (52 – 4.16)] 16.16
Reversal of revaluation loss [10.5-(10.55÷12.5W-1)] 9.66
Revaluation surplus – factory buildings (Bal.) 6.50
(revaluation of factory buildings on 31 December 2014 and
reversal of previous revaluation loss).
Office buildings
1-1-2014 Accumulated depreciation 26.32
Office building 26.32
1-1-2014 Office building 11.76
Revaluation surplus [149.94 – (164.5 – 26.32)] 11.76
1-Jul-2014 Depreciation expense [31.92 5 ÷ 21 × 0.5] 0.76
Accumulated depreciation-Office buildings 0.76
(Depreciation expenses for the six months ended 1 July 2014 for
the office building block sold)
1-Jul-2014 Revaluation surplus [(31.92-27.72=4.2) 5 ÷ 21 × 0.5] 0.10
Retained earnings 0.10
(Transfer of incremental depreciation for the six months ended
31 December 2014 to retained earnings)

20 Page 20 of 43
1-Jul-2014 Bank 30.00
Accumulated depreciation 0.76
Loss on sale of Office building (bal) 1.16
Office buildings 31.92
(Sale of office building)
1-Jul-2014 Revaluation surplus 4.1
Retained earnings 4.1
(4.2 - 0.1)
31-Dec-2014 Depreciation expense (149.94 – 31.92) ÷ 21 5.62
Accumulated depreciation-Office buildings 5.62
(Depreciation expenses for the year ended 31 December 2014)
31-Dec-2014 Revaluation surplus-Office building [149.94-(164.5-26.32)- 0.36
4.21÷21
Retained earnings 0.36
(Transfer of incremental depreciation for the year ended 31
December 2014 to retained earnings)
(W-1: Remaining useful life of the buildings on the revaluation date of 31 December 2013

Years

100 37.5 
Factory buildings:   5;  7.5; 20  7.5  12.5 12.50
 20 5 

164.5 26.32 
Office buildings:   6.58;  4; 25  4  21 21.00
 25 6.58 
Answer No. 2
“Rs. In Million”
Date Particulars
Debit Credit
1-7-2010 Plant 500
Bank 500
10-6-2011 Depreciation 50
Accumulated depreciation (500 ÷ 10) 50
1-7-2011 Accumulated depreciation 50
Plant 50
1-7-2011 Plant 125
Revaluation Surplus 125
30-6-2012 Depreciation 63.89
Accumulated depreciation [575/9] = 63.89 63.89
30-6-2012 Revaluation Surplus 13.89
Retained Earnings [125/9] = 13.89 13.89
1-7-2012 Accumulated depreciation 63.89

21 Page 21 of 43
Plant 63.89
1-7-2012 Revaluation Surplus 111.11
Revaluation Loss 10
Plant 121.11
WDV = 575 – 63.89 = 511.11
FV = 390.00
30-6-2013 Depreciation 48.75
Accumulated depreciation [390÷8] = 48.75 48.75
1-7-2013 Accumulated depreciation 48.75
Plant 48.75
1-7-2013 Plant 38.75
Reversal of Loss [10 – 1.25 (10/8)] 8.75
Rev. Surplus 30.00
WDV = 390 – 48.75 = 341.25
FV = 380.00
38.75
30-6-2014 Depreciation 54.29
Accumulated depreciation [380÷7] = 54.29 54.29
30-6-2014 Rev. Surplus 4.29
Retained Earnings [30÷7] = 4.29 4.29

Answer No. 3
PQR
Journal Entries for the year ended 30-6-2014
Office Buildings:

30-6-14 Depreciation 500 (6,000 ÷ 12)


Accumulated depreciation 500
30-6-14 Accumulated depreciation 1,000 (500 + 500)
Office building 1,000
30-6-14 Office building 750
R. Surplus 750
WDV = 5,500 – 500 = 5,000
FV = 5,750
Factory Buildings:
30-6-14 Depreciation 440 (4,400 ÷ 10)
Accumulated depreciation 440
30-6-14 Accumulated depreciation 880 (440 + 440)
Factory building 880
30-6-14 Revaluation Loss 200
Factory building 200

22 Page 22 of 43
WDV = 3,960 – 440 = 3,520
FV = 3,320
200
Warehouse:
30-6-14 Depreciation 450 (4,500 ÷ 10)
Accumulated depreciation 450
30-6-14 Accumulated depreciation 900 (450 + 450)
Warehouse 900
30-6-14 Revaluation Loss 250
Warehouse 250
WDV = 4,050 – 450 = 3,600
FV = 3,350
250
Journal Entries for the year ended 30-6-2015
Office Buildings:

30-6-15 Depreciation 718.75 (5,750 ÷ 8)


Accumulated depreciation 718.75
30-6-15 Revaluation Surplus 93.75 (750 ÷ 8)
Retained Earnings 93.75
Factory Buildings:
30-6-15 Depreciation 369 (3,320 ÷ 9)
Accumulated depreciation 369
Warehouse:
30-6-15 Depreciation 419 (3,350 ÷ 8)
Accumulated depreciation 419

Ans. 4
Buildings
1-1-2014 Cash 450
c/d 450
1-1-2015 b/d 450 1.1.2015 Acc. Depreciation 22.5
1-1-2015 R.S 28.5
c/d 456
1-1-2016 b/d 456 1.1.2016 Acc. Depreciation 24
1.1.2016 R.S 27
1.1.2016 R.L 27
c/d 378

23 Page 23 of 43
Accumulated Depreciation
31-12-2014 Depreciation (450÷20) 22.5
c/d 22.5
1.1.2015 Building 22.5 1.1.2015 b/d 22.5
31.12.2015 Depreciation (456÷19) 24
c/d 24
1.1.2016 Building 24 1.1.2016 b/d 24
31.12.2016 Depreciation (378÷18) 21
c/d 21

R.S
31.12.2015 Retained earnings (28.5÷19) 1.5
1.1.2015 Building 28.5
c/d 27
1.1.2016 Building 27 1.1.2016 b/d 27
c/d -
(W-1) 1-1-2015
WDV = 450 – 22.5 = 427.5
FV = 456
R. Surplus = 28.5
1-1-2016
WDV = 456 – 24 = 432
FV = 378
R. Loss = 54

R. Surplus 27
R. Loss (balance) 27
Building 54

(b) Equipment

1-1-2014 Cash 50
31-12-2014 c/d 50
b/d 50
1-8-2015 Cash 35
31-12-2015 c/d 85
b/d 85 30-6-2016 Disposal 25
c/d 60

Accumulated Depreciation

31-12-2014 Depreciation (50×10%) 5


c/d 5
01.01.2015 b/d 5

24 Page 24 of 43
31.12.2015 Depreciation 5.96
(50-5)×10% + 35 × 10% ×
5/12
c/d 10.96
01.01.2016 b/d 10.96
30.06.2016 Disposal 5.7625 31.12.2016 Depreciation 6.3915
[25×10%] = 2.5 [85-10.96-20.25]×10%
[25-2.5]×10% = 2.25 + 20.25×10%×6/12
[25-2.5-2.25]×10%× 6/12 = [5.379 + 1.0125]
1.0125
c/d 11.58

Answer 5
Property, plant and equipment

2016
Building Equipment Total
-------- Rs. In million ---------
Gross carrying amount
Opening 456.00 85.00 541.00
Elimination (24.00) (24.00)
Revaluation surplus [28.5 – (28.5 ÷ 19)] (27.00) (27.00)
Revaluation loss (27.00) (27.00)
Disposal (25.00) (25.00)
Closing 378.00 60.00 438
Accumulated depreciation
Opening 24.00 10.96 34.96
Elimination (24.00) (24.00)
Disposal [2.5 + 2.25 + 1.01] (5.76) (5.76)
Depreciation
(378÷18) 21.00 21.00
[(74.04-20.25)×10% + (20.25×10%×6÷12)] 6.39 6.39
Closing 21.00 11.59 32.59
Carrying amount [378 – 21] 357.00 48.41 405.41
[60-11.59]

25 Page 25 of 43
Property, plant and equipment
2015
Building Equipment Total
-------- Rs. In million ---------
Gross carrying amount
Opening 450.00 50.00 500.00
Elimination (22.50) (22.50)
Revaluation 28.50 28.50
Addition 35.00 35.00
Closing Cost 456.00 85.00 541.00
Accumulated depreciation
Opening 22.50 5.00 27.50
Elimination (22.50) (22.50)
Disposal
Depreciation
(456÷19) 24.00 24.00
[(45×10%) + (35×10%×5÷12)] 5.96 5.96
Closing 24.00 10.96 35.96
Closing amount (456 – 24] 432.00 74.04 506.04
[85-10.96]

Disclosures related to property, plant and equipment:


Buildings Equipment
Measurement base Revaluation model Cost model
Useful life/depreciation rate 20 years 10%
Depreciation method Straight line Reducing balance
Disclosures regarding revaluation of buildings:
1.The last revaluation was performed on 1 July 2016.
2.The revaluation was performed by Accurate Valuers (private) Limited, an independent
firm of valuers.
3.Carrying value had the cost model been used.
2016 2015
Buildings 382.50 405.00
[450 – (22.5 x 2)] [450 – (22.5 x 3)]
4. Change in revaluation surplus:
2016 2015
Opening balance 27 -
Revaluation (27) 28.5
Transfer of surplus - (1.5)
Closing balance - 27
There is restriction on distribution of revaluation surplus as dividend to shareholders.

26 Page 26 of 43
Detail of Disposal of Equipment:

Gross carrying amount 25


Accumulated Depreciation (5.7625)
Carrying Amount 19.2375
Sale proceeds Not Available
Mode of Disposal Not Available
Particulars of buyer Not Available

Answer No. 6
Abid Limited
Note of Property, Plant & Equipment :
For the year ended 31-12-2015
Rs. In Millions
2015 2014
Gross carrying amount:
Opening balance 252 323
Addition/Disposal (68) --
Elimination (14) (17)
Revaluation 34 (54)
Closing balance 204 252
Accumulated Depreciation
Opening balance 14 17
Elimination (14) (17)
For the year 14 14
Disposal (2) --
Closing balance 12 14
Carrying amount 192 238
Useful life 20 years 20 years

Disclosures of Revaluation:
The revaluation was performed on 1-1-2015 by M/s Premies Valuatin Services, an independent firm of valuers.
Revaluations are performed annually. The carrying amount of buildings had they were carried at cost model would
have been:
31-12-2014 = 300 – (15 × 3) = 255
31-12-2015 = 225* - 45** = 180
* [300 – 75] = 225
** [225 ÷ 20 × 4] = 45

Detail of Disposal of building:


Gross carrying amount (272 ÷ 4) 68
Accumulated Depreciation (68 ÷ 17 × 6/12) = 2
Carrying Amount 66
Sale proceeds 80
Mode of Disposal Not Available

27 Page 27 of 43
Particulars of buyer Not Available

Workings: (Ledgers; not required in exam but for calculation of figures)


Building A/c
2012
Cash 300
c/d 300
b/d 300 Accumulated depreciation 15
Revaluation surplus (285 – 323) 38
c/d 323
b/d 323 Accumulated depreciation 17
Revaluation surplus 36
R. Loss 18
c/d 252
b/d 252 Accumulated depreciation 14
Reversal of loss 17 Disposal (W-2) 68
Revaluation surplus (W-1) 17
c/d 204

Accumulated Depreciation
Depreciation (300 ÷ 20) 15
c/d 15
Building 15 b/d 15
Depreciation (323 ÷ 19) 17
c/d 17
Building 17 b/d 17
Depreciation (252 ÷ 18) 14
c/d 14
Building 14 b/d 14
Disposal 2 Depreciation (W-3) 14
c/d 12

Revaluation Surplus

Retained earnings (38 ÷ 19) 2 b/d --


Building 38
c/d 36
Building 36 b/d 36
c/d --
Retained earnings (W-4) 0.875 b/d --
Retained earnings (4.25 – 0.125) 4.125 Building 17

28 Page 28 of 43
c/d (4 × 3) 12

Workings:
1) 01.01.2015
WDV = 238
FV = 272

R. Surplus = 34

Reversal of loss 18 R. Surplus 17


Extra depreciation to be charged (15 x 1) (1)
Net reversal of loss 17
(2) FV of Building Disposed:

272/4 = 68
(3) Depreciation for the year:

[272 – 68] ÷ 17 = 12
[68 ÷ 17 × 6/12] = 2
(4) Transfer of Surplus:

17/4 = 4.25 per building; 17 - 4.25 = 12.75 ÷ 17 = 0.75; 4.25/17 x 6/12 = 0.125; total 0.75 + 0.125 =0.875

29 Page 29 of 43
ICAP Study Text
IAS 16: Property, plant and equipment
DISCLOSURE REQUIREMENTS OF IAS 16

IAS 16 Property, plant and equipment requires the following disclosures in the notes to the financial
statements, for each major class of property, plant and equipment.

The measurement bases used (cost or revaluation model)

The depreciation methods used

The useful lives or depreciation rates used

Gross carrying amount (means cost or revalued amount) and the accumulated depreciation at the beginning
and at the end of the period

A reconciliation between the opening and closing values for gross carrying amounts and accumulated
depreciation, showing:
i. additions during the year
ii. disposals during the year
iii. depreciation charge for the year
iv. impairment losses
v. the effect of revaluations.

An entity must also disclose:



the existence and amounts of restrictions on title, and property, plant and equipment pledged as
security for liabilities;

the amount of expenditures recognised in the carrying amount of an item of property, plant and
equipment in the course of its construction;

the amount of contractual commitments for the acquisition of property, plant and equipment;
and

if it is not disclosed separately in the statement of comprehensive income, the amount of compensation
from third parties for items of property, plant and equipment that were impaired, lost or given up that is
included in profit or loss.

Disclosures for assets stated at revalued amounts


When items of property, plant and equipment are stated at revalued amounts the following must be disclosed:
I. the effective date of the revaluation;
II. whether an independent valuer was involved;

III. for each revalued class of property, plant and equipment, the carrying amount that would have been
recognised had the assets been carried under the cost model; and
IV. the revaluation surplus, indicating the change for the period and any restrictions on the distribution of
the balance to shareholders.

Additional disclosures encouraged by IAS 16


IAS 16 encourages disclosure of the following information as users of financial statements might find it to be
useful.
I. the carrying amount of temporarily idle property, plant and equipment;
II. the gross carrying amount of any fully depreciated property, plant and equipment that is still in use;

III. the carrying amount of property, plant and equipment retired from active use and held for disposal; and

30 Page 30 of 43
IV. when the cost model is used, the fair value of property, plant and equipment when this is materially different
from the carrying amount.

Further Practice:
Example 01: ALI LIMITED

Question: Ali Limited (AL) uses the revaluation model for subsequent measurement of its property, plant and
equipment and has a policy of revaluing its assets on an annual basis using the net replacement value method.
The following information pertains to AL’s building:
i. The building was purchased on 01 January 2010 for Rs. 2 hundred million with expected useful life of ten
years.
ii. AL depreciates buildings on the straight line basis over their useful life.
iii. The results of revaluations carried out during the last three years by Standard Valuation Service, an
independent firm of values, are as follows:

Revaluation date Fair value Rs. in million


1 January 2011 280
1 January 2012 170
1 January 2013 180
Required:
The journal entries relating to the above transactions including revaluations for the year ended December 31,
2010, 2011, 2012 and 2013.

Answer:

Debit Credit
Date - Property, plant and equipment
---- Rs. in million ----
January Building Account 200
01,2010 Payable 200
December Depreciation (200÷10) 20
31,2010 Accumulated depreciation 20
January Accumulated depreciation 20
01,2011 Building 20
January Building (280-(200-20)) 100
01,2011 Revaluation surplus (OCI) 100
January Depreciation (200÷9) 31
31,2011 Accumulated depreciation 31
January Revaluation surplus (100/9) 11
31,2011 Retained earnings 11
January Accumulated depreciation 31
01,2012 Building 31
January Revaluation surplus (SOCI) (280-31)-170 79
01,2012 Building 79

31 Page 31 of 43
December Depreciation (170÷8) 21.25
31,2012 Accumulated depreciation 21.25
December Revaluation surplus 1.25
31,2012 Retained earnings 1.25
(100-11-79)=10/8 =1.25
January Accumulated depreciation 21.25
01,2013 Building 21.25
January Building 31.25
01,2013 Revaluation surplus (180-(170-21.25) 31.25
December Depreciation (180÷07) 25.7
31,2013 Accumulated depreciation 25.7
December Revaluation surplus 5.7
31,2013 Retained earnings 5.7
(10-1.25+31.25)/7

Example 02: SUNDRY QUESTION 1


Question: The following are two independent scenarios:
1. A business purchased some land and buildings on 1 January 2011 for Rs.800million (land Rs.250 million and
buildings Rs.550 million). The buildings are to be depreciated over a period of 50 years.
On 1 January 2015 the land and buildings were revalued to Rs. 1,500 million (land Rs.400 million and buildings Rs.
1,100 million). At this date the buildings were believed to have a remaining useful life of 40 years.

Required:
What is the original depreciation charge for the buildings and the revised charge from 1January 2015?

Answer:
The original depreciation charge for the buildings and the revised charge from 1 January 2015 are calculated as
under:
Original depreciation = Rs.550 million /50 years = Rs.11 million Revised depreciation = Rs.1,100 million /40 years =
Rs.27 million

2.A business purchased land for Rs.250 million and buildings for Rs.400 million on 1 January2011. The buildings were
to be depreciated over a period of 50 years. On 1 January 2015 the land was revalued to Rs.520 million and the
buildings were revalued at Rs.750 million.

Required:
What amount is to be taken to the revaluation reserve on 1 January 2015?

Answer:

Rs.m Rs.m
Land (520 - 250) 270
Buildings – cost 400
depreciation (400/50 x 4 years) (32)

32 Page 32 of 43
Net book value 368
Revaluation 750
Transfer to revaluation reserve 382
Total revaluation surplus 652

Example 03: EHTISHAM


Question:
The following information relates to the financial statements of Ehtisham for the year to 31 March2015.
The head office of Ehtisham was acquired on 1 April 2012 for Rs. 1 million. Ehtisham intend to occupy the building for
25 years. On 31 March 2014 it was revalued to Rs. 1.15 million. On 31 March 2015,a surplus of vacant commercial
property in the area had led to a fall in property prices and the fair value was now only Rs. 0.8 million.
Required:
Explain the correct accounting treatment for the above (with calculations).
Answer:
IAS 16 permits assets to be carried at cost or revaluation. Where the latter is chosen, the asset must be stated at its
fair value.

The original depreciation was Rs. 40,000 (Rs. 1,000,000/25 years) per annum.

On 31st March 2014 the asset is two years old. Its carrying value before revaluation was therefore Rs.1million less
accumulated depreciation of Rs.80,000 (40,000 x 2).

Rs.
Cost/valuation 1,000,000
Accumulated depreciation (80,000)
Net book value 920,000
In order to effect the revaluation, the cost is uplifted to fair value of Rs.1.15m, the accumulated
depreciation is eliminated, and the uplift to the net book value is credited to a revaluation surplus
account.
Debit Credit
150,000
Building
Accumulated depreciation 80,000
Revaluation surplus 230,000

The impact of the journal is as follows:

Before Adjustment After


Cost/valuation 1,000,000 150,000 1,150,000
Accumulated depreciation (80,000) 80,000 nil
Carrying amount 920,000 1,150,000

33 Page 33 of 43
The asset is depreciated over its remaining useful economic life of 23 years giving a charge of Rs.50,000
(Rs. 1,150,000/23 years) per annum in the year to 31st March 2015.
Debit Credit
Depreciation 50,000
Accumulated depreciation 50,000
This results in a carrying value as at 31st March 2015 of:
Rs.

Cost/valuation 1,150,000
Accumulated depreciation (50,000)
Carrying amount 1,100,000

Transfer from revaluation surplus to retained earnings

As a result of the revaluation, the annual depreciation has increased from Rs.40,000 to Rs.50,000.
This extra depreciation of Rs.10,000 is transferred from the revaluation reserve to accumulated
profits each year.(or 230,000/23)
Debit Credit
Revaluation surplus 10,000
Retained earnings 10,000

By the 31st March 2015, the balance remaining on the revaluation reserve will be Rs.220,000.

Rs.
Surplus recognised at 31 March 2014 230,000
Transfer to accumulated profits (10,000)
Carrying amount 220,000

The fall in property values at the year-end. The asset must be revalued downwards to Rs.0.8million,a
write-down of Rs.300,000.[1,100,000-800,000]

Rs.220,000 of this is charged against the revaluation reserve relating to this asset, and the
remaining Rs.80,000 must be charged against profits.

The reduction of the carrying amount of the asset is achieved by removing the accumulated
depreciation and adjusting the asset account by the balance.

Debit Credit
Revaluation surplus 220,000
Revaluation Loss - Statement of profit or loss (Working - 1) 80,000
Asset at valuation 350,000
Accumulated depreciation 50,000

The impact of the journal is as follows:


Before Adjustment After

34 Page 34 of 43
Cost/valuation 1,150,000 350,000 800,000
Accumulated depreciation (50,000) 50,000 Nil
Carrying amount 1,100,000 800,000

Example 04: CARLY

Question: The following is an extract from the financial statements of Carly on 31 December 2014.Property, plant
and equipment

Land and Plant and Computer Total


buildings machinery equipment

Rs. Rs. Rs
Rs.

Cost
On 31 December 2014 1,500,000 340,500 617,800 2,458,300

Accumulated depreciation
On 31 December 2014 600,000 125,900 505,800 1,231,700

Carrying amount
On 31 December 2014 900,000 214,600 112,000 1,226,600

Accounting policies
Depreciation

Depreciation is provided at the following rates.


On land and buildings Over 50 years on straight line basis on buildings only
On plant and equipment 25% reducing balance
On computers 33.33% per annum straight line
During 2015 the following transactions took place.
(1) On 31 December the land and buildings were revalued to Rs. 1,750,000. Of this amount, Rs. 650,000 related
to the land (which had originally cost Rs. 500,000). The remaining useful lifeof the buildings was assessed as
40 years.
(2) A machine which had cost Rs. 80,000 and had accumulated depreciation of Rs. 57,000 at thestart of the year
was sold for Rs. 25,000 at the start of the year.
(3) A new machine was purchased on 30 April 2015. The following costs were incurred:
Rs.
Purchase price, before discount, inclusive of reclaimable sales tax of Rs.3,000 20,000
Trade Discount 1,000
Delivery costs 500
Installation costs 750
Interest on loan taken out to finance the purchase 300
(4) On 1 January it was decided to change the method of providing depreciation on computer

35 Page 35 of 43
equipment from the existing method to 40% reducing balance.
Required:
Produce the analysis of property, plant and equipment as it would appear in the financial statementsof
Carly for the year ended 31 December 2015.
Answer:
Property, plant and Land and Plant and Computer Total
equipment buildings machinery equipment

Rs. Rs. Rs
Rs.

Gross Carrying amount


At 1 January 2015 1,500,000 340,500 617,800 2,458,300
Cancelation (620,000) (620,000)
Revaluation 870,000 - - 870,000
Additions (W2) - 17,250 - 17,150
Disposals - (80,000) - (80,000)
---------------------- ----------------------- ----------------------- -------------------
At 31 December 2015 1,750,000 277,750 617,800 2,645,550
----------------- ------------------- ------------------ ----------------
Accumulated depreciation
At 1 January 2015 600,000 125,900 505,800 1,231,700
Charge for the year (W1) 20,000 50,775 44,800 115,575
Cancelation (620,000) - - (620,000)
Disposals - (57,000) - (57,000)
--------------------- ----------------------- ---------------------- ------------------
At 31 December 2015 Nil 119,675 550,600 670,275
---------------- ------------------ -------------------- ----------------
Carrying amount
At 31 December 2014 900,000 214,600 112,000 1,226,600
------------------- ----------------------- --------------------- --------------------
At 31 December 2015 1,750,000 158,075 67,200 1,975,275
------------------- ---------------------- --------------------- -------------------

Workings:

(1) Depreciation charges


Buildings = (1,500,000 – 500,000) / 50 years = 20,000.

Plant and machinery:

New machine 25% 8/12) 2,875


Existing plant (((340,500 – 80,000) – (125,900 – 57,000)) 25%) 47,900
Total 50,775
Computer equipment = 40% = Rs.44,800

36 Page 36 of 43
(2) Cost of new machine

Purchase price (20,000 – 3,000 – 1,000) 16,000


Delivery costs 500
Installation costs 750
TOTAL 17,250
It is not a qualifying asset so interest is not capitalized.

Example 05 : ADJUSTMENTS LIMITED


Question: Adjustments Limited has carried out a review of its non-current assets.
(a) A grinder was purchased on 1 January 2012 for Rs. 100,000. The plant had an estimated useful life of ten
years and a residual value of nil. Depreciation is charged on the straight line basis. On 1 January 2015, when
the asset’s net book value is Rs. 70,000, the directors decide that it would be more appropriate to depreciate
this asset using the sum of digits’ approach. The remaining useful life is unchanged.
(b) The company purchased a fifty year leased property some years ago for Rs. 1,000,000. This was being
depreciated over its life on a straight line basis. On 1 January 2015, when the net book value is Rs. 480,000
and twenty-four years of the lease are remaining, the asset is revalued to Rs. 1,500,000. This revised value
is being incorporated into the accounts.
Required:
Explain the effects of these changes on the depreciation for the year to 31 December 2015.
Answer:
(b) The grinder was purchased in 2012 and was originally being depreciated on a straight line basis. It
has now been decided to depreciate this on the sum of digits basis.
(c)
IAS 16 requires that depreciation methods be reviewed periodically and if there is a significant change in the
expected pattern of economic benefits, the method should be changed. Depreciation adjustments should be
made in current and future periods. This change might be appropriate if, for instance, usage of the machine is
greater in the early years of an asset’s life when it is still new and consequently it is appropriate to have a higher
depreciation charge.

If the change is implemented, the net book value of the asset should be depreciated over the remaining useful life
commencing with the period in which the change is made.
The depreciation charge for the remaining life of the asset will therefore be as follows.

Year Digits Depreciation


Rs.
2015 7 7/28 x Rs.70,000 17,500
2016 6 6/28 x Rs.70,000 15,000
2017 5 12,500
2018 4 10,000
2019 3 7,500
2020 2 5,000
2021 1 2,500
— ———–
7 (7 + 1)/2 — Rs. 70,000
28 ———–

37 Page 37 of 43
Disclosure will need to be made in the accounts of the details of the change, including the effect on the charge in
the year.
The reassessment of the depreciation method is not a change in accounting policy and neither rectification of a
fundamental error so the effects of the change will not affect the previously reportedfinancial statements
(b) Leasehold Property:
IAS 16 requires that the subsequent charge for depreciation should be based on the revalued amount. The annual
depreciation will therefore be Rs. 62,500, i.e. Rs.1,500,000 divided by the 24 years of remaining life.
There will then be a difference between the revalued depreciation charge and the historical depreciation charge.
The resulting excess depreciation may be dealt with by a movement in reserves, i.e. by transferring from the
revaluation reserve to retained earnings a figure equal to the depreciation charged on the revaluation surplus
each year.

Example 07: SHAHWEZ LIMITED


Question: Shahwez Limited (SL) revalued its property on 1 April 2011 to Rs.20 million (Rs.8 million for the land). The
property originally cost Rs.10 million (Rs.2 million for the land) 10 years ago. The original useful life of 40 years is
unchanged. SL’s policy is to make a transfer to realised profits in respect of excess depreciation.

Required:

How will the property be reflected in the financial statements accounted for in the year ended 31 March 2012?

Answer:

Statement of comprehensive income extract for the year ended 31 March 2012

Depreciation expense
400
Other comprehensive income:

Revaluation gain 12,000

Statement of financial position extract as at 31 March 2012

Rs. In 000

Non-current assets Property 19,600


(20,000 – 400)

Equity Revaluation reserve 11,800

(12,000 – 200)

Statement of changes in equity extracts

Revaluation reserve Retained earnings


Revaluation surplus 12,000
Transfer of surplus (200) 200
Workings:

38 Page 38 of 43
Gain on revaluation: Rs. In 000
Carrying value at revaluation date
(10,000 – ((10,000 – 2,000)/40 years x 10 years)) 8,000
Fair value 20,000
Gain on revaluation 12,000
Double entry:
Dr Property 10,000
(20,000 – 10,000)
Dr Accumulated depreciation 2,000
((10,000 – 2,000)/40 years x 10 years)
Cr Revaluation reserve 12,000
Depreciation charge for year to 31 March 2012:
Dr depreciation expense 400
((20,000 – 8,000)/30 years)
Cr Accumulated depreciation 400
Transfer of surplus:
Historical cost depreciation charge 200
((10,000 – 2,000)/40 years)
Revaluation depreciation charge 400
Excess depreciation to be transferred 200
Dr Revaluation reserve 200
Cr Retained earnings 200

Example 07: HAMZA LIMITED


Question: Hamza Limited (HL) acquired a building on 1 April 2011 for Rs. 100,000 at which point it was
considered to have a useful life of 40 years. At the year-end 31 March 2016, HL decided to revalue the building
to its current value of Rs. 98,000.
Required:
How will the building be accounted for (means extracts of financial statements) in the year ended 31 March 2016?
Answer:

Statement of comprehensive income extract 31 March


2016
Depreciation charge (100,000/40) 2,500
Other comprehensive income:
10,500
Revaluation surplus

39 Page 39 of 43
Statement of financial position extract 31 March 2016
98,000
Building at valuation

Statement of changes in equity extract


Revaluation gain

Revaluation reserve 10,500


Notes:
Revaluation takes place at year end, therefore a full year of depreciation must first be charged.
(W1) Depreciation year ended 31 March 20X6
100,0000 / 40 years = Rs.2,500 per annum

(W2) Revaluation as at 31 March 20X6


Carrying value of non-current asset at revaluation date
(100,000 – (100,000/40 years x 5 years)) 87,500
Valuation of non-current asset 98,000
Gain on revaluation 10,500
Double entry: Debit Credit
Accumulated depreciation 12,500
Building 2,500
Revaluation reserve 10,500
Example 09: SUNDRY QUESTION 2
Question:
(a) When the asset should be derecognised from the balance sheet? What is the accountingtreatment for
gain or loss on disposal for revaluation model?
(b) What is the disclosure requirement of asset carried at revalued amount???
Answer:
(a). Property, Plant and Equipment should be derecognised (removed from PPE) either;

i. on disposal (sold or exchanged etc. by cash for asset given up) or

ii. when it is withdrawn from use and no future economic benefits are expected from the asset (in
other words, it is effectively scrapped).
A gain or loss on disposal is recognised as the difference between the disposal proceeds (gross proceeds received
minus cost of making sale) and the carrying value of the asset (using the cost or revaluation model) at the date of
disposal. This net gain is included in thestatement of profit or loss. The sales proceeds should not be recognised as
revenue.

Where assets are measured using the revaluation model, any remaining balance in the revaluation reserve relating
to the asset disposed of is transferred directly to retained earnings. No recycling of this balance into the statement
of profit or loss is permitted.

(b) General disclosures

The financial statements shall disclose, for each class of property, plant and equipment:

40 Page 40 of 43
a) the measurement bases used for determining the gross carrying amount;
b) the depreciation methods used;
c) the useful lives or the depreciation rates used;
d) the gross carrying amount and the accumulated depreciation (aggregated with accumulated
impairment losses) at the beginning and end of the period; and
e) a reconciliation of the carrying amount at the beginning and end of the period showing increases or
decreases resulting from revaluations from comparing its revalued amount to the book value and
recognize in other comprehensive income and accumulated in equity under the heading of revaluation
surplus. However, the revaluation increase shall be recognised in profit or loss to the extent that it
reverses arevaluation decrease of the same asset previously recognised in profit or loss.
Specific disclosures
If items of property, plant and equipment are stated at revalued amounts, the followingshall be disclosed:
i. the effective date of the revaluation;
ii. whether an independent valuer was involved;
iii. for each revalued class of property, plant and equipment, the carrying amount that would have been
recognised had the assets been carried under the cost model; and
iv. the revaluation surplus, indicating the change for the period and any restrictions on the distribution of
the balance to shareholders.

Example 10: INDIGO LIMITED


Question: Indigo Limited (IL) purchased a building on 1 April 2015 for Rs.10 million at which point it was
considered to have a useful life of 40 years. At the year ended 31 March 2020, IL decided to revalue the building
to its market value of Rs.9.8 million.
Required:
How the above building shall be accounted for in the books(means extracts of financial statements) of IL for the
year ended 31 March 2020?
Answer:
Statement of comprehensive income extract for the year ended 31 March 2020

Profit or loss account:

Depreciation charge = Rs.0.25 million (W1) Other


comprehensive income:

Revaluation surplus = Rs.1.05 million (W2)

Statement of financial position extract as at 31 March 2020

Building at valuation = Rs.9.8 million


Statement of changes in equity extract for the year ended 31 March 2020

Revaluation reserve
Revaluation surplus = Rs.1.05 million (W2)

41 Page 41 of 43
Workings:
Revaluation takes place at year end, therefore a full year of depreciation must first be charged.

(W1) Depreciation for the year ended 31 March 2020


Rs.10 million / 40 years = Rs.0.25 million
(W2) Revaluation
Carrying value of building at revaluation date (10 m – (10 m/40 years x 5 years)) = Rs.8.75 m
Revaluation of building = Rs.9.8 m

Gain on revaluation = Rs.1.05 m


Example 11:
Question: At 1 April 2019, Glacier Limited (GL) carried its office at its original cost of Rs.20 million and depreciation
of Rs.4 million (based on straight line method to its original life of 50 years). GL decided to revalue the office on 1
October 2019 to its current value of Rs.22 million. The useful liferemaining was reassessed at the time of valuation
and is considered to be 40 years at this date. It is the company’s policy to charge depreciation proportionally.
Required:
How the office will be accounted for in the financial statements of GL for the year ended 31 March 2020?
Answer:
Statement of comprehensive income extract for the year ended 31 March 2020 Depreciation charge (200,000
(W1) + 275,000 (W3) = Rs. 475,000
Other comprehensive income:
Revaluation gain (W2) = Rs.6.2 million
Statement of financial position extract as at 31 March 2020

Revaluation (office) = Rs. 22 million


Subsequent depreciation (W3) = (Rs. 275,000)
Carrying value = Rs. 21,725,000
Statement of changes in equity extract for the year 31 March 2020
Revaluation reserve (gain) = 6,122,500 [Rs. 6,200,000 (W2)-(6,200,000/40 x 6/12)]
Workings:
Revaluation takes place part way through the year and therefore depreciation must first be charged for the period 1
April 2019 – 30 September 2019, then the revaluation occurs so the depreciation needs to be charged for the
period 1 October 2019 – 31 March 2020 on current market value.
(W1) Depreciation
1 April – 30 September 2019 = Rs.20 million / 50 years x 6/12 = Rs. 200,000

(W2) Revaluation

The carrying value of the asset at 1 October 2019 can now be found and revalued.

42 Page 42 of 43
Carrying value of office at revaluation date (20,000,000 – (4,000,000 + 200,000)) = Rs. 15,800,000Revaluation of office
= Rs.22 million
Gain on revaluation = Rs. 6,200,000

(W3) Depreciation
1 October 2019 – 31 March 2020 => Rs. 22,000,000 / 40 years x 6/12 = Rs. 275,000

43 Page 43 of 43
Statement of Changes in Equity:
Components of financial statements
 Statement of financial position (balance sheet)
 Statement of comprehensive income (income statement)
 Statement of changes in equity
 Statement of cash flow
 Notes to the financial statements.

Introduction of business in the form of company:


Company: An association of persons which is registered under the Companies Act 2017.

Shareholder: Person who owns shares in the company is called as shareholder.

Share certificate: Evidence of ownership in the company.

Shareholders elect directors to manage the affairs of the company on their behalf.
Capital of the company is divided into shares; therefore called as share capital.

Types of share capital:

Authorized Capital: Maximum number of shares that a company can issue (No accounting entry is made for
authorized share capital)

Issued Capital/Paid-up Capital: Actual number of shares in issue at any point of time.
Every share has a registered price called as face value, nominal value or par value. In Pakistan, this registered price
is normally Rs. 10 per share.
(I) Issue of shares at nominal value/face value/par value

100,000 shares of Rs. 10 each issued at Rs 10 each.


Bank 1,000,000
Share capital 1,000,000

(II) Issue of shares at a premium


100,000 shares of Rs. 10 each issued at Rs 27 each.
Bank 2,700,000
Share capital 1,000,000
Share premium 1,700,000

(III) Issue of shares at a discount


100,000 shares of Rs. 10 each issued at Rs 7 each.
Bank 700,000
Discount on shares 300,000
Share capital 1,000,000
Discount on issue of shares is adjusted from share premium and if not available then from retained earnings in
statement of changes in equity.

Right shares: Shares issued to existing shareholders in proportion to existing number of shares against
consideration (normally cash). Its entry will be same as discussed above.

Page 1 of 13
44
Illustration: Cost of goods manufactured statement

XYZ Entity
Cost of goods manufactured statement
For the year ended 31 December 20XX

Rs. Rs.
Raw materials
Opening inventory 25,000
Purchases 150,000
175,000
Less: Closing inventory (20,000)
Raw materials consumed 155,000
Manufacturing wages 100,000
Prime cost 255,000

Factory Overheads
Light and power 72,000
Depreciation of production machinery 40,000
Depreciation of factory 50,000 162,000
Manufacturing costs/Factory cost 417,000

Opening work in progress 85,000


Closing work in progress (95,000)
Cost of goods made/Manufactured 407,000

Illustration: Statement of financial position


XYZ Entity
Statement of financial position
As at 31 December 20XX

Rs. M Rs. M
Assets
Non-current assets
Property, plant and equipment 205.1
Intangible assets 10.7
Investments 6.8 222.6

Current assets
Inventories 17.8
Trade and other receivables 15.3
Cash and cash equivalents 0.7 33.8
Total assets 256.4

Rs. m Rs. m

Page 2 of 13
45
Equity and liabilities
Equity:
Share capital 50.0
Share premium 31.9
Retained earnings (accumulated profits) 60.6 142.5

Non-current liabilities
Long-term borrowings 34.5 34.5
Current liabilities
Trade and other payables 67.1
Short-term borrowings (bank overdraft) 3.2
Current portion of long-term borrowing 5.0
Current tax payable 4.1 79.4
Total equity and liabilities 256.4

Example: statement of comprehensive income

XYZ Entity
Statement of comprehensive income
For the year ended 31 December 20XX
000
Revenue 678
Cost of sales (250)
Gross profit 428
Other income 12
Distribution costs (98)
Administrative expenses (61)
Other expenses (18)
Finance costs (24)

Profit before tax 271


Taxation (50)
––––––
Profit after tax 221

Taxation: tax on profits of the current period is called as current tax. It is expense for the business. Its accounting
entries are:
Current tax xxx
Current tax payable xxx
(when accrued at the period end)
Current tax payable xxx
Cash/ bank xxx
(when paid)

Dividend: It is the distribution of profits to shareholders.


Dividend* xxx
Dividend payable xxx

Page 3 of 13
46
(when declared)
Dividend payable xxx
Cash xxx
(when paid)
*Dividend is recorded when it is declared.

Dividend
Interim dividend Final dividend
Dividend on the basis of profits of less than one year Dividend on the basis of profits of full year

Dividend is recorded when it is declared. If dividend is declared after the reporting period but before the date of
authorization of financial statements, then the dividend declaration is disclosed together with the amount per
share in the notes to the financial statements of the period to which it relates.
Preparation of financial statements is the responsibility of the management of the company. Authorization of
financial statements means directors have signed the financial statements after their finalization.

Issue of Shares against Cash


Cash XX
*Share Capital XX

Difference can either be share premium or discount on issue of shares.


*Share capital and share premium are presented in equity.

Dividend to Shareholders:
It is distribution of profits to shareholders.

a)
*Dividend XX
Dividend Payable XX
(When the dividend is declared)
*Dividend is not an expense instead it is deducted from retained earnings just like drawings (which are deducted
from capital).
b)
Dividend Payable XX
Cash/Bank XX

(When the dividend is paid)

Dividend

Shares
Cash

Interim Final
Interim Final

Page 4 of 13
47
Bonus shares/ Stock dividend / Capitalization Issue
It is a situation in which company issues new shares to existing shareholders in proportion to existing shares
without any consideration.
Dividend xxxx
Share capital xxxx
Entry will be made at the amount of nominal value of shares. Therefore there will be no premium or discount in
this entry..

Summary of discussion about dividend


It is a distribution of profits to owners (shareholder). If dividend Is given as a percentage then multiply the rate
with the amount of share capital (nominal value of share capital) which is at the date of declaration of dividend to
calculate the amount of dividend. A company may pay dividend either in cash or in form of shares.
Dividend
Cash Dividend (Either Final or Interim) Bonus Shares (Either Final or Interim)
When Declared Dividend XXX
Dividend(Retained Earnings) XXX Share Capital A/c XXX
Dividend Payable A/c XXX

When Paid
Dividend Payable A/c XXX
Cash/ Bank A/c XXX

 Bonus shares are distributed from share premium and if the balance of premium is not available then from
retained earnings.
 Dividend is recognized on the date of declaration.
 If the dividend is declared after reporting date but before the authorization of the financial statements it is
disclosed in the financial statement to which it relates.
 If the date of declaration is not given, then:
1. For interim dividend assume that dividend is declared during the accounting period.
2. For final dividend assume declared after the reporting date.
 If nature of dividend (means whether final or interim) is not given then assume final dividend.
 If type of dividend (means cash or bonus) is not available then assume cash dividend.

Q.1 SCCL, a company listed on Karachi and Lahore stock exchange is in process of finalization of its accounts for the year
ended 31-12-2012. The following information is available

i. Shareholders equity as at 31-12-2011 and 2010 consist of


Rs in million

2011 2010

Share capital (Rs 10 each) 10,340 7,833

Unappropriated Profits 6,945 4,508

ii. Profit after tax for the years ended 31-12-2010, 2011 and 2012 (unaudited) was Rs 4,240 million, Rs
4,944 million and Rs 5,090 million respectively.

iii. Cash dividends and bonus declared/paid during the last three years were as follows:

Page 5 of 13
48
Cash Dividend Bonus Shares

*Interim Final *Interim Final

For the year ended 31-12-2010 10% - - 20%

For the year ended 31-12-2011 - 15% 10% 10%

For the year ended 31-12-2012 - 10% 5% 5%

*Interim dividend/bonus was declared at time of announcement of half-yearly financial results.

iv. Right shares were issued on 30-11-2012 in the ratio of 4 right shares for every 5 shares held by the
shareholders of the company. The right issue was made at Rs 18/share.
v. Applicable tax rate is 35%.

Required: Prepare statement of changes in equity for the year ended 31-12-2012.

Note: If more than one dividends are declared on the same date then for both dividends base share capital is
same.

In the question of statement of changes in equity even if the question is silent prepare on year comparative figures
if there is information.

Reserves: Reserves are profits retained in the business (not available for distribution to its shareholders as
dividend) to meet any future business requirements. There are two types of reserves:

1. Specific reserve (reserve created to meet any defined future business requirement e.g. any future capital
expenditure or against any court case).
2. General reserve (reserve created to meet any undefined future business requirement).

These reserves are also called as revenue reserves.


Accounting entry to create reserves:
Retained earnings xxx
Reserves xxx
(whether general or specific)

Transfer of incremental depreciation means amount of revaluation surplus transferred to retained earnings.

Question-2: For the purpose of preparation of statement of changes in equity for the year ended 31 December
2017, Daffodil Limited (DL) has extracted the following information:

2017 2016 2015


Draft Audited Audited
Rs. in million
Net profit 650 318 214
Transfer to general reserves 112 - 141
Transfer of incremental - 49 55
depreciation
Final cash dividend - - 7.5%

Page 6 of 13
49
Additional information:
i) Details of share issues:
o 25% right shares were issued on 1 May 2016 at Rs. 18 per share. The market price per share
immediately before the entitlement date was also Rs. 18 per share.
o A bonus issue of 10% was made on 1 April 2017 as final dividend for 2016.
o 50 million right shares were issued on 1 July 2017 at Rs. 15 per share. The market price per share
immediately before the entitlement date was Rs. 25 per share.
o A bonus issue of 15% was made on 1 September 2017 as interim dividend.
ii) Share capital and reserves as at 31 December:

2015 2014
------ Rs. in million ------
Ordinary share capital (Rs. 10 each) 1,600 1,600
General reserves 1,850 1,709
Retained earnings 1,430 1,302
Revaluation surplus 100 -

Required: Prepare DL’s statement of changes in equity for the year ended 31 December 2017 including
comparatives. (Ignore taxation)

Redemption of shares[means repurchase of ordinary shares at the discretion of management of the entity]
Debit Credit
Share capital Xxx
Bank Xxx

A company may redeem its shares for a number of reasons such as to buy out certain shareholders or to provide
an exit strategy to third party investors. However an equal amount needs to be transferred from retained earnings
to capital repurchase reserve to safeguard the interest of creditors and other loan providers. This reserve will
remain un distributable until winding up.

Q.3 The following information pertains to a listed company, Fu-tech (Pakistan) Limited.
a) Shareholders’ equity as at 1 January 2013:
Share capital (Rs. 10 each) Rs. 116 million
Retained earnings Rs. 58 million
b) The profit after tax for the year ended 31 December 2013 was Rs. 34 million (2012: Rs. 25).
c) Depreciation expense for the year ended 31 December 2013 and 31 december 2012 included
incremental depreciation amounting to Rs. 6.5 million and 7 million on account of revaluation surplus
(means transfer the surplus to retained earnings).
d) Right shares were issued on 15 September 2013 at Rs. 12 per share in the ratio of 1 right share for every
4 shares held by the shareholders of the company.
e) Dividend information is as under:
2013 2012 2011
Cash dividend - Interim *18% - 10%
Cash dividend - Final 14% 15%
Bonus shares - Final - 16%
*interim dividend was announced before the issue of right shares.

Page 7 of 13
50
f) Balance of revaluation surplus as on 01.01.2012 was 30 million.
g) On 31.12.2013, there was redemption of 1 million shares at par. Transfer an equal amount to capital
repurchase reserve.

Required:
Prepare a statement of changes in equity for the year ended 31 December 2013 in accordance with the
requirements of the Companies Act, 2017. (Show comparative figures)

Answers:
A.1
SCCL
Statement of Changes in Equity
For the Year ended 31-12-2012
Rs in millions
Share Share Unappropriated Total
Capital Premium Profit
Balance as on 1-1-2011 7,833 - 4,508 12,341

Final Bonus Shares-2010 (20%) 1,567 - (1,567) -


(7,833x20%)
Interim Bonus Shares-2011(10%) 940 - (940) -
(7,833+1,567)x10%
Profit for the year - - 4,944 4,944
Balance as on 31-12-2011 10,340 - 6,945 17,285

Final Bonus Shares-2011 (10%) 1034 (1034) -


(10,340x10%)
Final Cash Dividend-2011 (15%) (1,551) (1,551)
(10,340x15%)
Interim bonus Shares-2012(5%) 569 (569) -
(10,340+1034) x 5%
*Issue of Right Shares 9,554 7,643 - 17,197
Profit for the year (W) 5,090 5,090
Balance as on 31-12-2012 21,497 7,643 8,881 38,021

*Right share=(10,340+1034+569)=11,943÷10 x 4/5 = 955.4 m shares


Share capital = 955.4 x 10 = 9,554
Share Premium = 955.4 x 8 =7,643

Page 8 of 13
51
Answer-2:

DAFFODIL LIMITED
Statement of changes in equity
For the year ended 31 December 2017
-------------------- Rs. in million --------------------
Share Capital Share General Retained Revaluation Total
Premium reserves Earnings Surplus
Balance as at 31
December 2015 1,600.00 1,850.00 1,430.0 100.00 4,980.0
Final cash dividend @
7.5% - 2015 (120.00) (120.00)
(1,600×7.5%)

Right issue @ 25% 400.00 320.00 720.00


1,600/10=160 x 25%=40 (40 x 10) (40 x 8)

Net profit - 2016 318 318


Transfer of surplus 49 (49) -
Balance as at 31
December 2016 - 2,000.00 320.00 1,850.00 1,677 51 5,898

Final bonus dividend @


10% - 200.00 (200.00) -
2016 (2,000×10%)

Right issue 500.00 250.00 750.00


(50×10) (50×5)

Interim bonus dividend


@ 15% - 405.00 (370) (35) -
2017 (2,700×15%)

Net profit - 2017 650 650

Transfer to general
reserves 112.00 (112.00) -

Balance as at 31
December 2017 3,105.00 - 1,962.00 2,180 51 7,298

Page 9 of 13
52
Answer-3:

Fu Tech (Pakistan) Limited


Statement of Changes in Equity
For the Year ended 31-12-2013
Rs in
millions
Share Share Retained Revaluation Capital Total
Capital Premium Earnings Surplus repurchase
reserve
Balance as at 1-1-2012 100 42 30 - 172
Final Bonus shares at Rs. 1.6 16 (16) -
per share
Profit for the year 25.0 25.0
Transfer of surplus 7 (7) -
Balance as at 1-1-2013 116 58 23 - 197
Final Cash dividend at 15% (17.4) (17.4)
(116 x 15%)
Interim Cash dividend at 18% (20.88) (20.88)
for 2013 (116 x 18%)
Issue of right shares (W 5) 29 5.8 34.80
Profit for the year ended 31- 34.0 23.242
12-2012 (W 4)
Transfer of Surplus 6.5 (6.5) -

Redemption and transfer (10) (10) 10 (10)


Balance as at 31-12-2013 135 5.8 50.22 16.5 10 206.762

(W-1)
Share Capital
b/d (bal) 100
Dividend 16
c/d 116

Retained Earnings
Dividend 16 b/d (bal) 42
(116/116 x16) Profit after tax 25
c/d 58 Rev. surplus 7

(W-2)
Right shares 116÷10 = 11.6 x 1/4 = 2.9 m shares
Share capital = 2.9 x 10 = 29
Share Premium = 2.9 x 2 = 5.8
ICAP study text

STATEMENT OF CHANGES IN EQUITY (SOCIE)


Overview
A change in equity is simply the increase or decrease in the net assets (Total assets – Total liabilities) of the equity.
The statement of changes in equity separates owner and non-owner changes in equity in the following
manner:

transactions with owners; and

Page 10 of 13
53

non-owner changes in equity (e.g profit for the year, revaluation surplus), referred to as total
comprehensive income.

IAS 1 Preparation of Financial Statements


IAS 1 prescribes the basis for presentation of financial statements for the comparability of company’s financial
position to last year’s performance or with other entities. It sets the requirement, guidelines for structure and
presentation for the preparation of following statements
A complete set of financial statements consists of:

a statement of financial position (balance sheet) as at the end of the period;

a statement of comprehensive income for the period;

a statement of changes in equity for the period;

a statement of cash flows for the period

notes to these statements

Purpose and Importance of statement of changes in euity:

Statement of changes in equity helps users of financial statement to identify the factors that cause a change
in the owners' equity over the accounting periods.
It is important to note that the Statement of financial position provides information about the financial position
of the business by presenting total assets, liabilities and equity. Income statement on the other hand provides
information about how business has performed during the year and how much income has been generated
against the expenses and liabilities incurred.
But shareholders are interested in knowing how the business’ financial position and financial performance has
impacted their interest in the business. And this is not particularly addressed by Statement of Financial Position or
Income Statement.
Thus statement of changes in equity helps the users of financial statements to identify the factors that cause a
change in owner’s equity over the accounting periods.
Examples of the information provided in the statement of changes in equity include share capital issue and
redemption during the period, gains and losses recognised outside profit or loss (e.g. revaluation surplus),
dividends and bonus shares issued during the period.

The elements of a statement of changes in equity

A statement of changes in equity shows for each component of equity the amount at the beginning of the
period, changes during the period, and its amount at the end of the period.

A. Transactions with owners


It includes:

Share capital

Share premium

Redemption (it is the re acquisition of the entity’s own shares)

Dividend

Bonus shares

Right issue

B. Total comprehensive income


It includes:

Profit or loss for the year; and

revaluation surplus

Page 11 of 13
54
Types of Share Capital
Following are the different types of share capital:
1. Authorised share capital
2. Issued share capital
3. Subscribed share capital
4. Paid up share capital

Authorized Share Capital


It is the maximum amount of share capital that a company is authorized to issue/raise.
Issued Share Capital
A company may elect to issue a portion of the authorized share capital with the plan of issuing more shares at a
later date. Issued share capital is the total value of the shares a company offers for sale. The issued share capital
of a company is the par value of the shares that have actually been issued to shareholders.
Subscribed Share Capital
When a company issues its share capital, some part of it may not be subscribed by the investors. Subscribed
share capital is the monetary value of all the shares that the investors have committed to buy.
Paid up Share Capital
Paid-up share capital is the amount of money a company has received from shareholders in exchange for its
shares.

Revaluation surplus
The result of an upward revaluation of a non-current asset is a 'revaluation surplus'. The amount accumulated in
revaluation surplus is non-distributable, as it represents un-realised profits on the revalued assets. It is another
capital reserve. The revaluation surplus can only become realised if the asset is sold or as the asset is used by the
entity. The revaluation surplus may diminish if an asset which had previously been revalued upwards is devalued
later.

Summary of Journal Entries


Following journal entries are made in order to record the effects of changes in elements of equity during the
period.

Ref Account / Description Debit Credit


1 Bank X
Share Capital X
Issue of new shares/ right shares at par value
2 Bank X
Share capital X
Share Premium X
Issue of new shares/ right shares at premium
3 Dividend (Retained earnings) X
Share capital X
Issue of bonus shares
4 Dividend (Share premium) X
Share capital X

Page 12 of 13
55
Issue of bonus share
(If share premium balance is available)
5 Dividend (Retained earnings) X
Dividend payable X
On approval of interim / final dividend
6 Dividend payable X
Bank X
Payment of interim/final dividend

7 Retained earnings X
General reserves X
Transfer of balance to general reserves
8 Revaluation surplus X
Retained earnings X
Transfer of balance in case of incremental depreciation of
revalued assets
9 Revaluation surplus X
Retained earnings X
Transfer of balance in case of disposal of revalued asset
10 Share capital X
Bank X
Payment of cash for redemption of shares (at par)
11 Retained earnings X
Capital repurchase reserve account X
Transfer from distributable reserves to capital repurchase reserve account on
redemption of repurchased shares.

Page 13 of 13
56
Without Allah We are nothing. We surrender to the will of Allah

IAS 23 BORROWING COST


BASIC DISCUSSION

LONG TERM LOANS AND RELATED ISSUES:

A. A loan is obtained from MCB on 1-1-2012 against a sanctioned limit of 120 M. It carries markup @ 12% p.a.
This loan is repayable in semi-annual installments of Rs 5 M starting from 1-7-2012; along with interest.

Required:
i.) Calculate the amount of interest expense for the year ended 30-6-2013
ii.) Prepare relevant extracts from statements of financial position as on 30-6-2013

Note
i) Sanctioned limit means maximum amount of loan agreed with the bank.
ii) Instalment of loan comprises of principal amount only unless there is any indication.

B. A loan is obtained from Citibank Limited on 1-1-2013 for Rs 5,000,000. It carries interest @ Rs 0.23 per
thousand per day (means 8.395% p.a)

It is repayable in monthly instalments of Rs 100,000 starting from 1-4-2013, along with interest.

Required:
i.) Calculate the amount of interest expense for the year ended 30-6-2013
ii.) Prepare relevant extracts from statements of financial position as on 30-6-2013

C. A loan is obtained on 1-8-2012 for Rs 25 M. It carries markup @ 15% p.a. It is repayable in quarterly
instalments of Rs 5 M starting from 1-11-2012; along with interest.

Required:
i.) Calculate the amount of interest expense for the year ended 30-6-2013
ii.) Prepare relevant extracts from statements of financial position as on 30-6-2013

Example A
If we want to calculate interest expense; then identify the following figures:

Outstanding Balances of loan x Rate x Time period

(It means balance which remained constant for a given period of time)

Page 1 of 46
57
i) Calculation of interest expense:
Figures in millions

115 M x 12% x 6/12 (July to Dec) = 6.9


110M x 12% x 6/12 (Jan to June) = 6.6
13.5

Workings
1-1-2012 Loan Obtained Rs 120

30-6-2012 Closing Balance 120


1-7-2012 Repayment of Loan (5)
115
1-1-2013 Repayment of Loan (5)
110

30-6-2013 Closing Balance 110


1-7-2013 Repayment of Loan 5
1-1-2014 Repayment of Loan 5
Current Portion 10
Non-current Portion 100
ii)

Extracts from statement of Financial Positions


As on 30-6-2013
2013
Equity & Liabilities
Non-current liabilities
Long term Loan 100

Current Liabilities
Current Portion of Loan 10
Accrued Interest 6.6

Note:
In a question in which we are preparing an extract of statement of financial position or statement of
comprehensive income then there is no need of any total.

Page 2 of 46
58
May Allah guide us and forgive us for hurting other ones knowingly or unknowingly

Example B

i) Calculation of interest Expense


5,000,000 x 8.395% x 3/12 = 104,938
4,900,000 x 8.395% x 1/12 = 34,280
4,800,000 x 8.935% x 1/12 = 33,580
4,700,000 x 8.935% x 1/12 = 32,880
205,678

Workings
1-1-2013 Loan Obtained 5,000,000
1-4-2013 Loan repaid (100,000)
4,900,000

1-5-2013 Loan repaid (100,000)


4,800,000

1-6-2013 Loan repaid (100,000)


4,700,000
30-6-2013 Closing Balance 4,700,000

Current portion of loan 1,200,000


(100,000 x 12)
Non-current Portion 3,500,000

ii)
Extracts from statement of Financial Position as on 30-6-2013
2013
Non-current Liabilities
Long term loan 3,500,000

Current Liabilities
Current Portion of Loan 1,200,000
Accrued Interest 32,880

Example C
i) Calculation of Interest expense for the period
Figures in millions

25M x 15% x 3/12 Aug to Oct = 0.9375


20M x 15% x 3/12 Nov to Jan = 0.75
15M x 15% x 3/12 Feb to April = 0.5625
10M x 15% x 2/12 May & June = 0.25

Page 3 of 46
59
2.5
Workings
1-8-2012 Loan Obtained Rs 25
1-11-2012 Repayment made (5)
20

1-2-2013 Repayment made (5)


15

1-5-2013 Repayment made (5)


10

30-6-2013 Closing Balance 10

Current portion (5M x 2) 10

Non-current Portion Nil

ii) Extracts from statement of Financial Position as on 30-6-2013


2013
Non-current Liabilities
Long term loan --

Current Liabilities
Current Portion of Loan 10
Accrued Interest 0.25

Page 4 of 46
60
Quran takes us out of darkness and puts us into the light so don’t forget to read Quran.

Borrowing Cost (IAS-23)

Borrowing Costs
Interest and other related costs that an entity incurs in connection with borrowing of funds.
Other related costs includes bank charges deducted by bank at the time of disbursement of loan.
 Different names used for bank charges in exam questions.
 Arrangement fee
 Loan processing charges
 Commitment fee

 Different types of Loans/Borrowings:


 Long term Loans
 Short term Loans
 Short term running finance/Bank overdraft.

 Some important accounting entries


Issue of Shares against Cash
Cash XX
*Share Capital XX

*Share capital is presented in equity; it is not a borrowing.

Dividend to Shareholders:
It is distribution of profits to shareholders.

a)

*Dividend XX
Dividend Payable XX
(When the dividend is declared)

*Dividend is not an expense instead it is deducted from retained earnings just like drawings (which are deducted
from capital).
b)

Dividend Payable XX
Cash/Bank XX
(When the dividend is paid)
 As the share capital is not borrowing therefore dividend to shareholders is not a borrowing cost.
Treatment of borrowing cost: [Para 8]
Borrowing cost is recognized as an expense, when incurred;
Interest Expense XX
Interest expense payable XX
Other common words used for interest is finance cost or finance charge.

Page 5 of 46
61
Unless; borrowing cost related to a qualifying asset. In that case borrowing cost is Capitalized (added) to the cost
of a qualifying asset.

Qualifying Asset:
An asset which takes substantial time period to get ready for its intended use or sale.
 Asset for use
 Tangible Assets (IAS-16)
 Intangible Asset (IAS-38)

 Assets for sale


 Inventories (IAS-2)
Examples of qualifying assets:
Depending upon the circumstances, any of the following may be a qualifying asset:
a) Construction of factory building/Head office building
b) Purchase and installation of plant & machinery especially if imported.
c) Power generation facilities.
 Assets that are ready for their intended use or sale when acquired are not qualifying assets e.g furniture,
computers, vehicles etc)
 Similarly inventories that are manufactured over a short period of time are also not a qualifying asset.
General Rule of Capitalization:
All amounts incurred to bring an asset into working condition as intended by management are Capitalized (added
to cost of an asset).
Conclusion:
Borrowing cost is recognized as an expense when incurred unless it relates to a qualifying asset. In that case, it is
capitalized.

Types of Borrowings with respect to Capitalization of borrowing cost:

Specific Borrowings General Borrowings

Specific Borrowings
Borrowings which are taken for the sole purpose of construction, acquisition or production of qualifying asset.
Amount of borrowing cost to be Capitalized:
In case of specific borrowings, the amount to be capitalized is calculated as follows:

Amount of interest incurred on outstanding balance of loan Xxxx


Less: investment income (if any) on temporary investment of surplus funds (xxx)
Amount of interest to be capitalized Xxx

Page 6 of 46
62
Allah’s plans are better than our dreams

Commencement of Capitalization
As per IAS-23, entity shall commence capitalization of borrowing cost only; when
(i) Entity incurs borrowing cost; and
(ii) Entity incurs expenditures on qualifying asset and activities necessary to prepare the asset have started.
For example
a) A loan obtained on 1-8-2014
Construction started on 1-9-2014
 Commencement date should be 1-9-2014

b) Construction started on 1-7-2014 with owned funds.


On 15-12-2014 entity obtained a loan for further expenditures on qualifying asset.
 Commencement date should be 15-12-2014
Cessation of Capitalization
Entity shall cease capitalizing borrowing costs when the asset is substantially complete i.e. when the asset is
available for its intended use or sale.

Important points to remember


a) If a non-current asset is under process of construction or installation then it is called as Capital Work in
Progress (CWIP). If that CWIP is being prepared by borrowings then it is also called as qualifying asset.
b) While calculating investment income to be deducted from interest incurred, we will consider only that time
period for which interest incurred is capitalized.

Q1. Money Limited began the construction of a new building on the 1 February 20X5. Construction costs
incurred in 20X5 were paid for as follows:
Rupees
On 1 February 500,000
On 1 July 600,000
On 1 November 800,000

The construction of the building ended on the 1 December 20X5 when the building was complete and ready for its
intended use. This building is to be depreciated over 10 years to a nil residual value using the straight-line method.

The construction was financed by a loan of Rs. 1,900,000 from Citi Bank Limited. The loan was raised on 1 January
20X5 specifically to facilitate the construction of the building. The interest rate is 25% per annum. There were no
capital repayments during the year. Surplus funds were invested at 20% per annum.

The building is a qualifying asset for the purposes of IAS-23.

Required:
a) Calculate the amount of borrowing costs that are eligible for capitalization during the year ended 31
December 20X5.
b) Calculate the depreciation for the year ended 31 December 20X5.
c) Calculate the carrying amount of the buildings as at 31 December 20X5.
d) Calculate the amount of interest income to be recognized in statement of profit or loss.

Q2. Loans raised specifically to fund the construction of a building (a qualifying asset} :
Loan % Raised on Rs
A 10% 1 January 2005 500,000
B 15% 1 June 2005 400,000

Page 7 of 46
63
Rs 100,000 of the loan B capital was repaid on 31 July 2015. No other loan capital was repaid.

The only interest income earned during the year was interest income earned on the investment of surplus funds
from the specific loans in a 6% interest account.

Construction costs paid for as follows:


Date Rs
1st March 2005 300,000
30th April 2005 100,000
31st July 2005 220,000

Commencement date: 1 March 2015


Cessation date: 31 August 2005.

Required:
a) Calculate the amount of borrowing costs that must be capitalized in terms of IAS-23.
b) Calculate carrying amount of asset as on 31-12-2005, assuming that useful life is 5 years.

General Borrowings:
If a loan is for more than one purpose including a qualifying asset; then loan is called as General Purpose
Borrowing. These funds may be utilized for buying inventories, paying off expenses and multitude of other
purposes in addition to acquisition, construction or production of a qualifying asset.
E.g A loan from MCB for Rs 20M @ 12%.

Suppose:
(i) 5M used for factory expenses.
(ii) 2M used for vehicles; and
(iii) 10M used for construction of H.O building (a qualifying asset).

 In such a case all interest incurred on the loan is not related to this qualifying asset.
 In case of General Borrowings, amount of borrowing cost to be capitalized is calculated as follows;

Expenditure on qualifying asset x capitalization rate x time period

Capitalization Rate:
(i) If there is only one general loan then its rate is called as Capitalization Rate.
(ii) If there is more than one general loan having different interest rates, then we calculate an average rate of
interest called as capitalization rate as follows:

borrowing cost incurred


Capitalization rate = × 100
weighted Borrowing outstanding

Q3. MCQ (Private) Limited has the following general loans outstanding as at December 31 20x5.
Rs.
Loan -1 @ 6% 300,000
Loan -2 @8% 200,000
Loan -3 @9% 150,000

Page 8 of 46
64
Send blessings on the Messenger of Allah, (Peace Be Upon him) and seek forgiveness from Allah.

All the three loans were brought forward from previous year. Neither loan is taken during the year nor is repaid.

The company spent following amounts on construction of an asset.


January 31,20x5 70,000
April,01,20x5 80,000
December,01,20x5 10,000
The asset is not yet available for use as on 31-12-2005

Required:
a) Calculate Capitalization rate
b) Calculate Borrowing cost eligible for capitalization.
c) Calculate the carrying amount of asset as at 31-12-2005
d) Calculate the amount of borrowing cost to be charged to statement of comprehensive income as an expense
during the year ended 31-12-2005

Q4. Sublime Sports Limited is currently manufacturing its power plants. Up to December 31, 20x3, the
company has incurred costs totaling Rs. 500,000 on construction of one of its plants.

The following general loans are outstanding:


Rs.
Loan from MCB @ 9% 500,000
Loan from HBL @ 10% 625,000
Loan from UBL @11% 375,000

Loan from HBL was taken on July 1, 20X3 while other loans were brought forward from previous year.
Expenditures on plant were incurred as follows:
May 31, 2003 Rs. 300,000
July 31,2003 Rs. 200,000
Plant is not yet available for use as on Dec 31, 2003

Required:
You are required to calculate:
a) Capitalization rate of the company;
b) Total borrowing cost to be capitalized for the year 20X3.
c) Calculate the carrying amount as on 31 Dec, 2003.
d) Calculate the borrowing cost to be charged to statement of comprehensive income.

Q5. Soccer Limited began the construction of a new stadium on the 1 January 20X5. Details of the progress
payments made during 20X5 are as follows:
Rupees
On 1 January 300,000
On 1 April 200,000
On 1 July 250,000
On 1 September 150,000

Page 9 of 46
65
On 1 October 200,000

The stadium was still under construction at 31 December 20X5.

The construction was financed by general borrowings within the company. General loans outstanding at any one
time during 20X5 averaged Rs. 20,000,000. The interest expense incurred on these loans during 20X5 was Rs
2,600,000.

The stadium is a qualifying asset as defined by IAS-23.

Required:
a) Calculate the amount of borrowing costs that are eligible for capitalization to the stadium during the year
ended 31 December 20X5.
b) Calculate the depreciation for the year ended 31 December 20X5.
c) Calculate the carrying amount of the stadium as at 31 December 20X5.
 Progress Payments means payments made to contractor by customers as the work is being completed.

Note: In case of general borrowings, interest income is not to be considered while calculating the borrowing cost
to be capitalized.

Note: the amount of borrowing cost capitalized cannot exceed the amount of borrowing cost it incurred during the
period.

Page 10 of 46
66
May Allah accept our dua, forgive us and make us wise in doing good deeds and our daily works.

Important Journal Entries

Company Investors

Company issued Shares:

Bank XX Investment in Shares XX


Share Capital XX Bank XX

Company issued Debentures (It is simply a loan)

Bank XX Investment in Debentures XX


Debentures XX Bank XX

(Normally it is a non-current loan) (Normally it is a Non-current investment in


assets)

Q6. An entity constructs a factory building which is a qualifying asset. Funds are provided from entity's general
borrowings. Capitalization of borrowing cost commences on 1-7-2009 and continues throughout the year
until the expected completion on 31-12-2010.

The detail of expenditures on the qualifying asset is as follows:


1 July 2009 2,050,000
15th September 2009 1,000,000
15th April 2010 500,000
30 June 2010 400,000

The entity's general borrowings in the period from 1st July 2009 to 30th June 2010 consist of the followings:
1) Rs. 5 million Debentures issued on 1-1-2009 with a fixed interest rate of 10%. The full amount of debentures is
payable in 2015.
2) Bank loan of 2 million taken on 1-7-2009 with a floating interest rate that is adjusted semi- annually by
changes in KIBOR. At 1-7-2009 interest rate was set at 8.75% which increased to 9.00% at 1 January 2010. The
loan is paid off in half-yearly installments of Rs. 200,000.
3) Bank overdraft with a floating interest rate that is adjusted by changes in KIBOR. Interest rate at 1-7-2009 is
14% which was adjusted to 14.25% at 1 Jan 2010. The overdraft at 1 July 2009 was 300,000 which increased to
700,000 at 1 Jan 2010, which remained constant till the year end.

Required:
Calculate the carrying amount of the factory building as on 30-06-2010 in accordance with relevant IFRS.

KIBOR: Karachi Inter Bank offered rate (It is an interest rate market).

Suspension of Capitalization

Sometimes after commencement of capitalization and before cessation of capitalization, there may be a
temporary stoppage of active development. In that case according to IAS-23:

Page 11 of 46
67
An entity shall suspend capitalization of borrowing costs during extended periods of time in which it suspends
active development of a qualifying asset unless temporary delay is a necessary part of the process of getting the
asset ready for its intended use or sale.

Example: Delays in Construction (Suspension Period)


A hotel was under construction in 2005. Borrowing cost of Rs. 300,000 is incurred on a loan during 2005. The loan
was specifically raised in January 1, 2005 for the sole purpose of construction of hotel.

Required:
Discuss how much of borrowing cost may be capitalized in the following two independent scenarios assuming that:
a) Workers went on a strike for a period of two months during which no progress was made.
b) The builder of the hotel had to wait for five days for cement in the foundation to dry.

Solution:
a) During these two months, the interest incurred may not be capitalized to the asset as it is an unnecessary
interruption to the construction process.
b) The borrowing costs must still be capitalized as it is merely a temporary delay and is a normal part of the
construction process.

Q7. Hockey Limited borrowed 2,000,000 (at an interest rate of 14%) from the Bank on 1 January 20X5. These
funds have been borrowed in order to build a hockey stadium.

Progress payments made in 20X5 are as follows:

Rupees
On 1 January 600,000
On 1 July 1,200,000
On 1 September 200,000

The surplus funds were invested in a fixed deposit earning interest at 10% per annum.

Construction began on 1 January 20X5 and was still incomplete on 31 December 20X5. Between 1 June and 20
June, construction suspended while concrete cured (a necessary part of the construction process).

The stadium is a qualifying asset as defined by IAS-23.

Required:
a) Calculate the amount of borrowing costs that may be capitalized to the hockey stadium cost account in the
year ended 31 December 20X5.
b) Calculate the amount of borrowing costs that may be capitalized to the hockey stadium cost account in the
year ended 31 December 20X5 assuming that construction could not begin due to building plans not meeting
municipal standards. The plans have been resubmitted and it is expected that the municipality will give the go-
ahead to begin construction in early 20X6.

Expenditure on Qualifying Asset:


If an entity has funds available from its owned resources, specific borrowings and general borrowings at any given
date then assume that:
 First; own funds are utilized
 Second; funds from specific borrowing ;and
 Then funds from general borrowings

Page 12 of 46
68
O Muslims, Fear Allah and don’t deal with riba (usury) or interest.

Repayment of Principal and Interest of Specific Borrowing:


If an entity has funds available from:
a) Owned resources
b) Specific borrowings; and
c) General Borrowings

At the date of repayment of principal and interest then assume that funds were utilized in above order.

Q8. On 1-3-2010, granite corporation (GC) started the construction of a new plant to meet the growing
demand for its products. The new plant was completed on 31 may 2011.

GC financed the cost of the project from the following sources:


i. On 1-3-2010, a 7 year loan of 70 million was obtained specifically for the construction of the plant.
The loan carried markup @ 13% p.a. payable semi-annually. An arrangement fee @ 1% of the loan
amount was paid to the bank. Two installments each comprising of repayment of principal of 5
million along with interest were paid on 31-08-2010 and 28-2-2011.
ii. GC also has a running finance facility of RS 100 million carrying markup @14% p.a. Any additional
amount required for the project was provided through this general purpose facility.
iii. Surplus fund were invested in saving accounts @ 8% P.a.

Payments made to contractor were as follows:

Payment date Rs in million


1-3-2010 25
31-1-2011 65
30-09-2011 10

The construction work was suspended from 1-02-2011 to 28-02-2011. The suspension was caused due to delay in
shipment of essential component for the installation of plant.

Required:
Calculate the amount of borrowing cost that may be capitalized during the year ended 30-06-2010 and 30-06-
2011.

Q9. Amjad Ltd started construction of a factory building on 1st April 2011. Construction took seven months to
complete. A contractor was hired for this work and total contract price was agreed to be Rs 20 million.

Progress payment were made as follows:


1st April 10%
31st July 45%
30th September 15%
31st October 30%

Construction was financed partly by own savings and partly by loans. First payment to contractor was made out of
own funds. Afterwards all payments were made from borrowed funds.

Page 13 of 46
69
A loan of Rs 8 million was obtained on 31st May 2011 specifically for this construction. This loan carried a markup
of 18%.Loan is to be repaid by the end of 2012.Unutilized portion of loan was kept in a deposit account yielding an
interest rate 4%.
Any shortage of funds was fulfilled from existing pool of loans:
Bank Loan Amount Interest
Rs
Aay 50,000,000 10%
Bee 25,000,000 14.5%
Cee 10,000,000 15%

All these loans remained outstanding throughout the year.

Factory building has useful life of 10 years. After completion of construction, due to a delay in purchase of
furniture, building was brought into use from 1st January 2012.

Required:
a) Calculate the amount of borrowing cost eligible for capitalization.
b) Calculate the carrying amount of building as on 31-12-2011.
c) Calculate the amount of borrowing cost to be recognized in income statement in the year ended
31-12-2011.

Note: while calculating the capitalization rate, only consider the outstanding balance of loans during the period. It
means commencement date, cease date and suspension period is not considered while calculating the rate.

Page 14 of 46
70
Don’t be Muslim by just name but be Muslim by practicing Islam.

Some important Terminologies used in exam questions:


Progress Billing: Bills given by contractor to customer as the work is performed and complete.
Retention Money: An amount deducted by the customer from progress billings as a security against satisfactory
completion of work. It is treated as a liability unless refunded/adjusted.
Right Shares: Shares issued to existing shareholders against consideration (which is normally cash). First time issue
of share is not called as right shares. Its accounting entry is exactly similar to entry of share capital.

Any funds from shareholders are treated as owned funds in a question of borrowing cost.

Q10. On July 1, 2009, Qureshi Steel Limited (QSL) signed an agreement with Pak Construction Limited for
construction of a factory building at a cost of Rs. 100 million. It was agreed that the factory would be
ready for use from January 1, 2011. The terms of payments were agreed as under:
1) 10% advance payment would be made on signing of the agreement. The advance paid would be
adjusted at 10% of the quarterly progress bills.
2) 5% retention money would also be deducted from the progress bills. Retention money will be
refunded one year after completion of the factory building.
3) Progress bills will be raised on last day of each quarter and settled on 15th of the next month.

The under mentioned progress bills were received and settled by QSL as per the agreement:
Invoice date Amount (Rs.)
September 30, 2009 30 million
December 31, 2009 20 million
March 31, 2010 10 million
June 30, 2010 15 million

On April 30, 2010 an invoice of Rs. 1.5 million was raised by the contractor for damages sustained at the site, on
account of rains. After negotiations, QSL finally agreed to make additional payment of Rs. 1.0 million to
compensate the contractor. The amount was paid on May 15, 2010. It is expected that 75% of the payment would
be recovered from the insurance company.

The cost of the project has been financed through the following sources:
1) Issue of right shares amounting to Rs. 15 million, on September 1, 2009. The company has been following a
policy of paying dividend of 20% for the past many years.
2) Bank loan of Rs. 25 million obtained on December 1, 2009. The loan carries a markup of 13% per annum. The
principal is repayable in 5 half yearly equal installments of Rs.5 million each along with the interest,
commencing from May 31, 2010. Loan processing charges of Rs.0.5 million were deducted by the bank at the
time of disbursement of loan. Surplus funds, when available, were invested in short term deposits at 8% per
annum.
3) Cash withdrawals from the existing running finance facility provided by a bank. Average running finance
balance for the year was Rs. 60 million. Markup charged by the bank for the year was Rs. 9 million.

Required:
a) Compute cost of capital work in progress for the factory building as of June 30, 2010 in accordance with the
requirements of relevant IFRSs.
(Borrowing costs calculations should be based on number of months)
b) Prepare extracts from statement of Financial Position as on 30-6-2010

Page 15 of 46
71
Q11. On January 1, 2015, Imran Limited started the construction of its new factory. The construction period is
approximately 15 months and the cost is estimated at Rs. 80 million. The work has been divided into 5
phases and payment to contractor shall be made on completion of each phase.

In the year the company had the following sources of finance available.
(i) Rights i s s u e o f shares amounting to Rs. 15 million on January 1, 2015. The company usually pays a
dividend of 10% each year.
(ii) Bank loan of Rs. 32 million carrying a mark-up of 13% was raised on March 1, 2015. (This loan was
outstanding for 306 days in the year).
(iii) On August 1, 2015, Rs. 10 million were borrowed from the bank. Interest thereon, is payable at the
rate of 11%. (This loan was outstanding for 153 days in the year).
Investment income on temporary investment of the borrowings amounted to Rs. 0.5 million.
The details of bills submitted by the contractor, during the year are as follows:
Particulars Date of payment Rupees

On completion of 1st phase March 1, 2015 20,000,000


On completion of 2nd phase April 1,2015 18,000,000
On completion of 3rd phase October 1, 2015 16,000,000
On completion of 4th phase Payment not yet made 17,000,000

On June 1, 2015, the Building Control Authority issued instructions for stoppage of work on account of certain
discrepancies in the completion plan. The company filed a petition in the Court and the matter was decided in the
company’s favour on July 31, 2015. Work recommenced after a delay of 61 days.
The following periods may be relevant:
Period Days
March 1 to December 31 306
April 1 to December 31 275
August 1 to December 31 153
October 1 to December 31 92
Required
a) Assuming that the loans were taken specifically for the project, calculate the amount of borrowing costs that
s h o u l d be capitalised in the p e r i o d e n d i n g December 31, 2015 in accordance with the requirements of
IAS 23 Borrowing Costs.
b) Assuming that the loans constituted general finance, calculate the amount of borrowing costs that s h o u l d
be capitalised in the p e r i o d e n d i n g December 31, 2015 in accordance with the requirements of IAS 23
Borrowing Costs.
(Borrowing cost calculations should be based on number of days)

Disclosures
a) Amount of borrowing Cost capitalized during the period
b) Capitalization rate used in case of general borrowings is to determine the amount of borrowing cost eligible
for capitalization.

Page 16 of 46
72
Invite people to Islam even without words, by just your behavior of kindness and good deeds.

Self-Testing Questions
Q1. On September 1, 2008, Spin Industries Limited (SIL) started construction of its new office building and
completed it on May 31, 2009. The payments made to the contractor were as follows:

Date of Payment Rupees


September 1,2008 10,000,000
December 1,2008 15,000,000
February 1,2009 12,000,000
June 1,2009 9,000,000

In addition to the above payments, SIL paid a fee of Rs. 8 million on September 1, 2008 for obtaining a permit
allowing the construction of the building.

The project was financed through the following sources:


i. On August 1, 2008 a medium term loan of Rs. 25 million was obtained specifically for the construction of the
building. The loan carried mark up of 12% per annum payable semi-annually. A commitment fee @ 0.5% of the
amount of loan was charged by the bank.
Surplus funds were invested in savings account @ 8% per annum. On February 1, 2009 SIL paid the six monthly
Interest plus Rs. 5 million towards the principal.
ii. Existing running finance facilities of SIL
a. Running finance facility of Rs. 28 million from Bank A carrying mark up of 13% payable annually. The
average outstanding balance during the period of construction was Rs. 25 million.
b. Running finance facility of Rs. 25 million from Bank B. The mark up accrued during the period of
construction was Rs. 3 million and the average running finance balance during that period was Rs. 20
million.

Required:
Calculate the amount of borrowing costs to be capitalized on June 30, 2009 in accordance with the requirements
of International Accounting Standards. (Borrowing cost calculations should be based on number of months).

Q2. In the board meeting held on 15 December 2006, management of Power limited decided to construct a
qualifying asset amounting to Rs, 1,000,000. The management of Power limited approached to a bank to
finance the project but the bank agreed to finance only the 50% of the estimated cost I.e. Rs. 500,000 @
15% p.a. The loan was sanctioned on 28th December 2006 and was immediately put into the bank
account of company. Management of the company decided to utilize other general purpose loans to
finance the remaining balance of Rs, 500,000.

The construction of qualifying asset started on 1 January 2007. Following is the schedule of payments made by
Power limited In connection with the construction of qualifying asset.
Amount of Payment
Date of Installment
(Rs.)
January 1,2007 200,000
April 1, 2007 200,000
July 1, 2007 200,000
October 1,2007 200,000

Page 17 of 46
73
Management decided to invest the unutilized portion of specific loan in the government securities fetching an
income @ 6% p.a. There was an unnecessary stoppage in the work from 1 June 2007 to 31 July 2007.
The following is the information regarding all the loans available to Power limited:
Description Amount Rate Loan taken On
Specific loan 500,000 15% 28-12-2006
General purpose loan 1 600,000 12% 01-02-2007
General purpose loan 2 400,000 14% 01-03-2007
General purpose loan 3 300,000 14% 01-01-2007

The general purpose loan 2 was repaid in full on-30-11-2007 and general purpose loan 3 was repaid in full on 31-
10-2007. Remaining loans were outstanding till 31 December 2007.
The construction of qualifying asset was not completed, up to 31 December 2007.
Required:
a) Calculate the amount of borrowing cost and portion of borrowing cost eligible for capitalization for the year
ended December 2007.
b) Compute the amount of borrowing cost to be recognized as an expense during the year.

Note: repayment of general borrowings are not considered while preparing the detail of payments.

Q3. On, 01 January 2012, Marvelous Engineering Limited (MEL) started construction of its new factory. The
construction work was completed on 30 November 2012. The payment made to the contractor as follows:
Date of payment Rs. in million
01-Jan-12 100
01-Apr-12 310
15-Dec-12 90

The construction work was financed through the following sources:


Date Description Rs. in million
01-Jan-12 12% Redeemable preference shares 150
01-Apr-12 14% TFCs for four years 300
01-Jul-12 Issue of right shares (estimated return is 22%) 50

The following additional information is also available:


i. The preference shares would be redeemed on 31 December 2016.
ii. Surplus funds were invested in a savings scheme @ 9% per annum.
iii. Due to delay in supply of construction material, the construction work was suspended from, 01 June 2012 to
30 June 2012.

Required:
Calculate the amount of borrowing costs that may be capitalized during the year ended 31 December 2012 in
accordance with the requirements of International Financial Reporting Standards. (Assume that calculations of
borrowing costs are based on number of months)

Note: if nothing is mentioned then loans are assumed as specific.

Page 18 of 46
74
Follow the teaching of Quran and Sunnah and be friend of righteous people.

Q4. A socially responsible multinational corporation (MNC) decided to construct a tunnel that will link two
sides of the village that were separated by a natural disaster years ago. Realizing its role as a good
corporate citizen, the MNC has been in this village for a couple of years exploring oil and gas in the nearby
offshore area. The tunnel would take two years to build. It began construction on January 1, 2014. The
following payments were made during 2014:
Rs 000
January 31 200
March 31 450
June 30 100
October 31 200
November 30 250
The first payment on January 31 was funded from the entity's general borrowings. However, the entity succeeded
in raising a medium-term loan for an amount of Rs 800,000 on March 31, 2014, with simple interest of 9% per
year. These funds were specifically used for this construction. Excess funds were temporarily invested at 6 percent
per year. The general borrowings was again used for an Rs 200,000 payment on Nov 30, which could not be funded
from the medium-term loan. The construction project was temporarily halted for six weeks in May, June, when
substantial technical and administrative work was carried out which was necessary stoppage for the completion of
construction work.. The following amounts of debt were outstanding at the year end, December 31, 2014:
Rs
Medium-term loan (see description above) 800,000
Bank overdraft (The weighted average amount outstanding during the year was Rs 750,000, and 1,200,000
interest charged thereon by the bank amounted to Rs 33,800.)
A 10%, 7-year loan taken on October 1, 2007, repayable in full in December 2015 9,000,000
Required:
Calculate the amount of borrowing cost to be capitalized to the cost price of the Tunnel in 2014? (10 marks)
Question No.5
NRA Limited started construction of building on 01 January 2010. Building was completed in two years’ time with
cost of Rs. 30 million. Company financed this construction mainly from existing running finance facility which
carries mark up at 14% per annum. Running finance limit is 100 million.
NRA Limited obtained a loan of Rs. 10 million at 12% per annum for this building on 1 December 2010 but due to
some breach of contract with bank it had to return the total amount of loan with interest on 01 June 2011. Surplus
funds were invested in a saving account @ 14%.
Construction activities remained suspended for the month of August 2011 due to shortage of material.
Following is the detail of payments made to contractor for the construction of building:

Date Amount
01-March-2010 3,200,000
30-June-2010 4,400,000
16-Sep-2010 2,300,000
01Dec-2010 6,200,000
31-jan-2011 5,500,000
01-May-2011 4,400,000

Page 19 of 46
75
01-Oct-2011 2,500,000
01-Dec-2011 1,500,000

Required:
Calculate the cost of building at 31 Dec., 2010 and 31 Dec., 2011 after capitalizing borrowing cost where
applicable. (Marks 15)

Q.6 On 1 July 2014, Alpha Trading Limited (ATL) signed an agreement with Quality Builders Limited for construction
of an office building at a cost of Rs. 500 million. Construction commenced on 1 July 2014 and is planned to
complete on 30 June 2016. The payments made to the builders were as follows:

(Rs. in million)
Invoice Date Payment Date Description Net Payment
20-Jan-2014 1-Jul-2014 Advance 50.00
10-Sep-2014 31-Oct-2014 st
1 progress bill 79.90
30-Dec-2014 31-Jan-2015 2nd progress bill 100.30

The progress bills were paid after deduction of advance and retention money at 10% and 5% of the gross amount
of the bills respectively. Retention money is to be refunded on completion of warranty period of one year from the
date of completion of the building.

On 1 September 2014, the construction work was stopped for one month to resolve geological complications
pertaining to foundation of the building.

The construction cost has been financed from the following sources:
(i) Bank loan of Rs. 100 million was obtained on 1 July 2014. The loan carries a mark-up of 11% payable semi-
annually on 31 December and 30 June each year. The principal is repayable in four equal annual
installments, commencing from 1 April 2015.
(ii) Existing finance facility was used for balance payments. Average running finance balance for the year ended
31 December 2014 was Rs. 190 million. Mark-up charges for the year ended 31 December 2014 amounted
to Rs. 24.70 million.
(iii) Surplus funds available were invested in a saving account @ 7% per annum.
ATL computes finance cost on a monthly basis.

Required:
From the above information, compute the related amounts and disclose them under appropriate heads in
ATL’s Statement of Financial Position as at 31 December 2014 in accordance with the International Financial
Reporting Standards.
(12)

Question No. 7
Alex Technologies started construction of its office building in 2014. The building was expected to be constructed
at a total cost of Rs. 20 million in almost 1 year. To finance the construction, following sources were planned:
- Rs. 3 million from its own resources set aside for this purpose.
- Rs. 17 million from a loan from bank at an interest rate of 15%.
Construction work started on June 1, 2014. A contractor was hired for this construction. It was agreed to settle the
contractor’s invoices after 1 month of invoice. Following invoices were submitted by the contractor:

Page 20 of 46
76
Stop trying to reform others until you reform yourself first.

No. Date Value (Rs.)


1031 30-06-14 2,000,000
1032 30-09-14 4,500,000
1033 31-12-14 4,750,000
1034 31-03-15 6,500,000
1035 31-07-15 2,250,000

Due to some deficiency in documentation, sanction of loan got delayed. Finally bank loan was sanctioned on
January 1, 2015. Till the loan sanction and after utilization of own funds, contractor’s bills were paid out of existing
running finance facilities. Following facilities were in use:

Date of receipt of
Bank Loan Interest Date of repayment
loan
A 01-01-12 Rs. 25 m 18.00% 31-07-15
B 30-06-13 Rs. 35 m 15.00% 28-02-15
C 01-01-14 Rs. 10 m 20.00% 30-09-14

Construction was complete by July 31, 2015. However, work remained suspended unnecessarily for the month of
April 2015. During construction period, surplus funds were kept in a deposit account yielding an interest of 6%.
Required:
Compute cost of office building as at December 31, 2014 and 2015. (14)

Page 21 of 46
77
Solutions
A1.

Money Limited

a) Borrowing costs to be capitalized


Interest incurred during construction:
1,900,000 x 25% x 10 / 12 395,833
Interest earned
(1 900,000 – 500,000) x 20% x 5/12 116,667
(1 900,000 – 500,000 – 600,000) x 20% x 4/12 53,333
170,000
Borrowing cost to be capitalized 225,833

b) Depreciation in 20X5

Construction costs 500,000 + 600,000 + 800,000 1,900,000


Borrowing costs capitalized Part (a) 225,833
2,125,833
Depreciation:
= 2,125,833 / 10 years x 1 / 12 months = 17,715

c) Carrying amount at 31 December 20X5


Cost 2,125,833
Accumulated depreciation Part (b) (17,715)
Carrying amount 2,108,118

d) Interest income to be recognized in income statement

1,900,000 x 20% x 1/12 = 31,667 (Related to Jan 2005)

 Ledgers not required in question. For Additional Information


Loan A/c Bank A/c
1-Jan Bank 1,900,000 1-Jan Loan 1,900,000 1-Feb CWIP 500,000
1-July CWIP 600,000
C/D 1,900,000 1-Nov CWIP 800,000
1,900,000 1,900,000 1,900,000 1,900,000

A2.

a) Borrowing Cost to be Capitalized


Loan A
1-3 to 31-8 500,000 x 10% x 6/12 = 25,000

Loan B
1-June to 31-7 400,000 x 15% x 2/12 = 10,000
1-August to 31-8 300,000 x 15% x 1/12 = 3,750
38,750

Less: Investment Income

Page 22 of 46
78
Loan A Funds 1-1 500,000
Payment made 1-3 (300,000)
200,000 x6%x2/12 = 2,000
Payment made 30-4 (100,000)
100,000 x6%x1/12 = 500
Loan B Funds 1-6 400,000
500,000 x6%x2/12 = 5,000
Payment made 31-7 (220,000)
Repayment of Loan 31-7 (100,000)
180,000 X6%x1/12 = 900
(8,400)

Amount of borrowing cost to be capitalized 30,350

b) Carrying Amount of Building as on 31 -12-2005:


Construction Cost = 620,000
(300,000 + 100,000 + 220,000)
Borrowing Cost Capitalized = 30,350
Total Cost of Building = 650,350

Depreciation
650,350 x 4/12 = 43,357
5
Carrying Amount as on 31 Dec,2005 is 650,350 – 43,357=606,993

A3. a) Calculation of Capitalization Rate


Rupees
Loan 1 300,000 X 6% 18,000
Loan 2 200,000 X 8% 16,000
Loan 3 150,000 x 9% 13,500
650,000 47,500

47,500 x 100 = 7.31%


650,000
b) Borrowing cost to be capitalized;
Loan Capitalization rate Months Rupees
January 31, 20X5 70,000 7.31% 11/12 4,691
April,01,20X5 80,000 7.31% 9/12 4,386
December,01, 20X5 10,000 7.31% 1/12 61
9,138

c) Carrying amount: Amount at which asset or liability is presented in statement of financial position.
Construction Cost = 160,000
Borrowing Cost = 9,138
= 169,138
Less Acc Dep = ( -- )
Carrying Amount = 169,138

Page 23 of 46
79
d) Borrowing cost to be expensed out during the year:
Total Borrowing cost = 47,500
(18,000 + 16,000 + 13,500)
Less Capitalized = (9,138)
To be expensed out = 38,362
A4.

a) Calculation of Capitalization Rate


Rupees
Loan MCB 500,000 X 9% 45,000
Loan HBL 615,000 X 10% 6/12 31,250
Loan UBL 375,000 x 11% 41,250
1,490,000 A 117,500

Weighted average Rupees


Loan MCB 500,000
Loan HBL (625000X 6/12) 312,500
Loan UBL 375,000
B 1,187,500

Capitalization rate B/A 9.89%

b) Borrowing cost to be capitalised


Loan Capitalization rate Months Rupees
May 31, 20X3 300,000 9.89% 7/12 17,308
July 31, 20X3 200,000 9.89% 5/12 8,242
25,550

c) Carrying Amount
Total Construction cost = 500,000
(300,000 + 200,000)
Borrowing cost capitalized = 25,550
Total Cost 525,550
Less Acc Dep = (--)
Carrying Amount = 525,550
d) Borrowing cost recognized as an expense:
Total borrowing cost incurred = 117,500
Less: Capitalized borrowing cost = (25,550)
Borrowing cots recognized as an expense = 91,950
A5.

a) Borrowing costs to be capitalized


Capitalization rate (weighted average rate of interest):
2 600,000 / 20,000,000
=13 %
300,000 X 13% X 12/12 39,000
200,000 X 13% X 9/12 19,500
250,000 X 13% X 6/12 16,250
150,000 X 13% X 4/12 6,500
250,000 X 13% X 3/12 6,500
87,750

Page 24 of 46
80
I thank Allah for giving me birth as Muslim and blessing with Islam.
b) Depreciation
Since the asset is not available for use (Still under construction) so there is no depreciation for the year.

c) Carrying amount at 31 December 2005


Total Cost incurred = 1,100,000
Borrowing Cost capitalized = 87,750

Total Cost = 1,187,750


Less: Acc Dep = ( -- )
Carrying Amount = 1,187,750

A6.

Capitalization Rate:
Weighted
Borrowing Interest Period
borrowing
Debenture at 10% 5,000,000 500,000 12/12 5.000,000
Bank loan:
8.75% for six months 2,000,000 87,500 6/12 1,000,000
9% for six months 1,800,000 81,000 6/12 900,000
Bank overdraft
14% for six months 300,000 21,000 6/12 150.000
14.25% for six months 7006-,000 49,875 6/12 350,000

739,375 7,400,000

Capital rate = 739,375 x 100


7,400,000
= 9.992% p.a

Borrowing Cost to be Capitalized


1-7-2009 2,050,000 x 9.992% x 12/12 = 204,836
15-9-2009 1,000,000 x 9.992% x 9.5/12= 79,103
15-4-2010 500,000 x 9.992% x 2.5/12= 10,408
30-6-2010 400,000 x 9.992% x 0/12= -
3,950,000 294,347

Carrying Amount :
Total Cost (3950,000 + 294, 355) 4,244,347
Less *Acc Dep (--)
Carrying Amount 4,244,347
*As asset is not yet available for use, therefore no depreciation.

Page 25 of 46
81
A7.

a) Borrowing costs to be capitalized


Interest incurred:
2,000,000 x 14% x 1 year = 280,000

Interest earned:
(1 400,000 x 10% x 6 / 1 2 ) + (200,000 x 10% x 2/12) = 73,333

You may find it easier to understand the calculation of the interest earned by looking at the following tabular
calculation:
Total Available for Interest on
From Used working
borrowings investment investment

1-Jan 2,000,000 600,000 1,400,000 70,000 1,400,000 x 10% x 6/12


1-Jul 1,200,000 200,000 3,333 200,000 x 10% x 2/12
1-Sep 200,000 - -
73,333

Interest to be capitalised
Interest incurred during construction 280,000
Less investment income during construction (73,333)
206,667

Please note: capitalization of borrowing costs does not cease during a temporary delay in construction.

b) Since construction has not yet started (the activities have not yet begun), none of the interest incurred is
eligible for capitalization during the year ended 31 Dec, 2005 (i.e. the interest incurred should be expensed).

A8.
Borrowing cost to be capitalized
2011 2010
Commitment fee - 700,000
Specific Borrowing (W 2) 6,987,500 3,033,333
Investment income (W 3) (1,749,000) (1,181,333)
General Borrowing(W 4) 1,381,625 -
6,620,125 2,552,000
Workings:
(1) Borrowing cost to be capitalized
Detail of expenditure

Used from Used from


Date Payments amount
specific general
1-3-2010 1st payment 25,000,000 25,000,000 -
31-8-2010 1st repayment 5,000,000+4,550,000(70m x 13% x6/12) 9,550,000 -
31-01-2011 2nd payment 65,000,000 34,750,000 30,250,000
28-02-2011 2nd repayment 5,000,000+4,225,000(65m x 13% x6/12) - 9,225,000
69,300,000

Page 26 of 46
82
(2) Specific borrowing
Interest incurred :

up to 31 June 2010 70,000,000 13% 4/12 3,033,333


For upto 30 June 2010 3,033,333

July and august 70,000,000 13% 2/12 1,516,667


September to January 65,000,000 13% 5/12 3,520,833
March April May 65,000,000 13% 3/12 1,950,000
For Upto 30 June 2011 6,987,500

(3) Investment income:

up to 31 June 2010 44,300,000 8% 4/12 1,181,333


For upto 30 June 2010 1,181,333

44,300,000 8% 2/12 590,667


34,750,000 8% 5/12 1,158,333
For Upto 30 June 2011 1,749,010

(4) General Borrowing:

31-1-2011 30,250,000 14% 3/12 * 1,058,750


28-2-2011 9,225,000 14% 3/12 322,875
1,381,625
* one month suspended
A9.
a) Amount of borrowing cost to be Capitalized
Specific borrowings (W-1) 546,667
General borrowings (W-2) 59,550
606,217
1) Specific borrowing
Interest Incurred
8,000,000 x 18% x 5/12 = 600,000
Investment Income
8,000,000 x 4% x 2/12 = (53,333)
546,667

2) General Borrowings
1,000,000 x 11.91% x 3/12 = 29,775
3,000,000 x 11.91% x 1/12 = 29,775
6000,000 x 11.91% x 0/12 = -
59,550

Page 27 of 46
83
Workings

Capitalization Rate = 5,000,000 + 3,625,000 + 1,500,000 x 100 = 1,0125,000 x 100 = 11.91%


50,000,000 + 25,000,000 + 10,000,000
Details of Expenditures
Particulars Amount Owned Specific General
1-4-2011 10% Payment 2,000,000 2,000,000 - -
31-7-2011 45% Payment 9,000,000 - 8,000,000 1,000,000
30-9-2011 15% Payment 3,000,000 - - 3,000,000
31-10-2011 30% Payments 6,000,000 - - 6,000,000

b) Carrying Amount
Construction Cost 20,000,000
Borrowing Cost 606,217
Total Cost 20,606,217
Less: Acc. depreciation (342437)
(20,606,217 ÷ 10 x 2/12)
Carrying Amount 20,262,780

c) Interest Expense to be charged in income statement


Specific
8,000,000 x 18% x 2/12 240,000
Nov & Dec
General
Aay 50,000,000 x 10% = 5,000,000
Bee 25,000,000 x 14.5% = 3,625,000
Cee 10,000,000 x 15% = 1,500,000
10,125,000
Interest Capitalized (59,550)
To be expensed 10,065,450

Total Borrowing cost to be expensed out (240,000+10,065,450) 10,305,450

A10.

Capital work in progress - Factory building Rs. in ‘000'


Progress invoices received from the contractor (30,000+20,000+10,000+15,000) 75,000.00
(Rain damages paid would be chargeable to profit and loss account / insurance
claim)
Borrowing costs to be capitalised:
Loan processing charges 500.00)
Interest on bank loan W-1 1,841.67)
Interest on running finance W-2 2,730.00)
Interest income from surplus loan amount W-4 (395.00)
Capital work in progress - June 30, 2010 79,676.67)

Page 28 of 46
84
Islam is the most beautiful and perfect religion. Islam teaches us peace and the truth

W-1: Interest on bank loan (Specific):


Rupees in ‘000
Interest amount Outstanding loan
Months Interest at 13%
From To amount
01-12-2009 31-05-2010 6 25,000 1625.00
01-06-2010 30-06 -2010 1 20,000 216.67
1,841.67

W-2: Interest on running finance


Rupees in ‘000
Payments from Months Interest at
Payments
Payments Invoice outstanding 15% per
Description net of Right Bank Running
date amount up to 30-6- annum (W-
deductions Issue Loan finance
10 3)
01-07-09 Advanced 10,000 10,000 10,000 12.00 1,500
payment
15-10 -09 1st progress 30,000 25.500 15,000 10,500 8.50 1,116
bill
15-01 -10 2nti progress 20,000 17,000 17,000 - - -
bill
15-04 -10 3rd progress 10,000 8.500 7,500 1,000 2.50 31
bill
31-05 -10 Loan interest 1,625 1,625 1.00 20
31-05 -10 Loan 5,000 5,000 1.00 63
instalment
15,000 *24,500 29,125 2,730

*Loan amount of Rs. 25,000,000 less processing charges of Rs. 500,000


15%
W-3: Average rate of interest for running finance facility (9,000/60,000)

W-4: Interest income from surplus loan amounts:


Rupees in ‘000'
Interest income
From Interest income To Months Surplus loan amounts
at 8%
01-12-09 15-01-10 1.5 24,500 (245)
16-01-10 15-04-10 3.0 7,500 (150)
(395)

W-5 Calculation of Net Payments: (used in W-2)


1st Bill 2nd Bill 3rd Bill 4th Bill
Amount of Bill 30,000 20,000 10,000 15,000
Less 10% Advance (3,000) (2,000) (1,000) (1,500)
5% Retention Money (1500) (1,000) (500) (750)
Net Payment 25,500 17,000 8,500 12,750

Page 29 of 46
85
Accounting Entries for the year ended 30-6-2010
(Not required in question just for additional information)

1-7-09 Advance to Contractor 10,000


Bank 10,000

30-9-09 CWIP 30,000


Advance to contractor 3,000
Retention money payable 1,500
Payable to contractor 25,500

15-10-09 Payable 25,500


Bank 25,500

31-12-09 CWIP 20,000


Advance to contractor 2,000
Retention money payable 1,000
Payable to contractor 17,000

15-1-10 Payable 17,000


Bank 17,000

31-3-10 CWIP 10,000


Advance 1,000
Retention money payable 5,00
Payable to contractor 8,500

15-4-10 Payable 8,500


Bank 8,500

30-6-10 CWIP 15,000


Advance 1,500
Retention money Payable 750
Payable to contractor 12,750

b)
Qureshi Steel Limited
Statement of Financial Position
As on 30-6-2010
2010
(000)
Non-Current Assets
CWIP-Factory Building 79,677

Current Assets
Advance to contractor 2,500
(10,000-3,000-2,000-1,000-1,500)
Insurance claim Receivable 750

Equity & Liabilities:


Equity
Share Capital 15,000

Page 30 of 46
86
Non-current Liabilities
Bank Loan 10,000
Retention Money Payable (1500+1000+500+750) 3,750

Current Liabilities
Current Portion of loan (5M x 2) 10,000
Financial Charges payable (20M x 13% x 1/12) 217
Payable to contractor(related to June Quarter 12,750

Running Finance (Assumed as Closing as well) 60,000

A11.
a) Assuming borrowings as Specific Borrowing:
Interest Incurred:
32,000,000 x 13% x (306-61)/365 = 2,792,329
10,000,000 x 11% x 153/365 = 461,096
Less: Investment Income: (Given) = (500,000)
Borrowing Cost to be capitalized = 2,753,425

b) Assuming borrowings as General Borrowings:


5,000,000 x 12.73% x (306-61)/365 = 427,240
18,000,000 x 12.73% x (275-61)/365 = 1,343,451
16,000,000 x 12.73% x 92/365 = 513,385
Borrowing cost to be Capitalized 2,284,076

W-1) Calculation of Capitalization Rate:


= 3,948,658 X 100 = 12.73%
31,019,178

 Interest incurred on loans:


32,000,000 x 13% x 306/365 = 3,487,562
10,000,000 x 11% x 153/365 = 461,096
3,948,658

 Weighted Avg outstanding balance:


32,000,000 x 306/365 = 26,827,397
10,000,000 x 153/365 = 4,191,781
31,019,178

W-2) Details of Payments:


Funds used From
Date Particulars Amount Right Shares Loans
1-3-2015 First Payments 20,000,000 15,000,000 5,000,000
1-4-2015 Second Payments 18,000,000 - 18,000,000
1-10-2015 Third Payments 16,000,000 - 16,000,000

Page 31 of 46
87
Be righteous and prevent your nafs (soul) from evil desires.

Solutions of Self-TEST Questions


A1.

Spin Limited
Amount of borrowing cost to be Capitalized as on June 30,2009
Commitment Fee 125,000
Specific Borrowings(W-1) 1,912,500
General Borrowings (W-2) 1,419,808
3,457,308

Workings:
1) Specific Borrowings:
Interest Incurred:
25,000,000 x 12% x 5/12 1,250,000
20,000,000 x 12% x 4/12 800,000
2050,000
Less Investment Income:
6,875,000 x 8% x 3/12 137,500
1,912,500

2) General Borrowings
8,125,000 x 13.88% x 6/12 563,875
12,,000,000 x 13.88% x 4/12 555,200
6,500,000 x 13.88% x 4/12 300,733
1,419,808

3) Detail Of Payments

Particulars Amount Specific General Months for General Borrowing


1-9-2008 Permit 8,000,000 8,000,000
1-9-2008 1st Payment 10,000,000 10,000,000
1-12-2008 2nd Payment 15,000,000 6,875,000 8,125,000 6 months
1-2-2009 3rd Payment 12,000,000 - 12,000,000 4 months
1-2-2009 Repayment 6,500,000 - `6,500,000 4 months

4)Capitalization Rate
6,250,000 x 100 = 13.88%
4,5000,000
2.1 25,000,000 x 13% = 3,250,000
20,000,000 3,000,000

Page 32 of 46
88
45,000,000 6,250,000

A2. a)
i)Total Borrowing Cost
500,000 x 15% = 75,000
600,000 x 12% x 11/12 = 66,000
400,000 x 14% x 9/12 = 42,000
300,000 x 14% x 10/12 35,000
218,000

ii)Borrowing Cost Eligible for Capitalization:


Specific Borrowings:
i) Interest Incurred:
500,000 x 15% x 10/12 (J – M & A – D ) 62,500
Less Investment Income
300,000 x 6% x 3/12 (J-M) 4,500
100,000 x 6% x 2/12 (A-M) 1,000
(5,500)
57,000
General Borrowings
100,000 x 13% x 5/12 5,417
200,000 x 13% x 3/12 6,500
11,917
Total amount to be capitalized 68,917
(57,000 + 11,917)

W-1 Capitalization Rate


143,000 (W-1.1) x 100 = 13%
1,100,000 (W-1.2)

W-1.1 Interest incurred on loans


600,000 x 12% x 11/12 = 66,000
400,000 x 14% x 9/12 = 42,000
300,000 x 14% x 10/12 = 35,000
143,000

W-1.2 Weighted Average borrowings


600,000 x 11/12 = 550,000
400,000 x 9/12 = 300,000
300,000 x 10/12 = 250,000
1,100,000

W-2) Details of Payments


Funds used From
Date Particulars Amounts Specific General
1-1-07 1st Payment 200,000 200,000

Page 33 of 46
89
1-4-07 2nd Payment 200,000 200,000
1-7-07 3rd Payment 200,000 100,000 100,000
1-10-07 4th Payment 200,000 200,000

Page 34 of 46
90
Allah is one. No one is worth to worship except Allah.

b) Amount to be recognized in income statement


Specific Loan:
500,000 x 15% 75,000
Less Capitalized (62,500)
To be expensed out 12500

General Loans:
600,000 x 12% x 11/12 = 66,000
400,000 x 14% x 9/12 = 42,000
300,000 x 14% x 10/12 = 35,000
Less: Capitalized (11,917)

To be expensed out 131,083

Total interest expense to be recognized in income statement as an expense:


12,500 + 131,083 = 143,583

A3. Amount of borrowing cost to be capitalized:


Rs. in million
Interest on redeemable preference shares [150 x 12% x (11- 1)-12] 15.00
Interest on TFCs [300 x 14% x (8-1)-12] 24.50

39.50
Less: Interest income from surplus funds (W-1) (3.23)
Amount of borrowing cost to be capitalized: 36.27

Surplus funds available Surplus Amount Interest income @ 9%


From To Months RS. in million
01-01-2012 31-03-2012 3 50 1.13
01-04-2012 30-11-2012 7* 40 2.1
3.23
*One month suspension

Details of Expenditures
Date Particulars Payments Specific Right Shares
Preference Shares TFCs
1-1-2012 1st Payment 100,000 100,000
1-4-2012 2nd Payment 310,000 50,000 260,000
15-12-2012 3rd Payment 90,000 40,000 50,000
150,000 300,000 50,000

Page 35 of 46
91
A4. Amount of borrowing cost to be capitalized:

Rs
Specific Loan

Rs 800,000 x 9% x 9/12 54,000


Less: Interest earned on unutilized amount of loan:
April 1 to June 30 [(800,000 – 450,000) x 3/12 x 6%] (5,250)
July 1 to October 31 [(800,000 – 550,000) x 4/12 x 6%] (5,000)
November 1 to November 30 [(800,000 – 750,000) x 1/12 x 6%] (250)
A 43,500
General Pool of Funds

Capitalization rate is 9.58% (W-1)


Paid on January 31 (200,000 x 11/12 x 9.58%) 17,563
Paid on November 30 (200,000 x 1/12 x 9.58%) 1,597
B 19,160

Total Amount to be Capitalized 62,660

Note: Although the activities has been interrupted for administrative work during May and June 2014,
capitalization of borrowing cost is not suspended for this period according to IAS-23, because it is a necessary
stoppage for the completion of construction work.

W-1 Rs
Capitalization rate for pool of debt

Total interest paid on these borrowings:


Bank Overdraft 33,800
7-year note (9,000,000 x 10%) 900,000
933,800
Borrowings outstanding
Bank overdraft 750,000
7-year note 9,000,000
9,750,000
Capitalization rate = 933,800/9,750,000 9.58% (rounded)

W-2 Detail of Payments:


Particulars Specific General
31-1 1st Payment 200,000
31-3 2nd Payment 450,000
30-6 3rd Payment 100,000
31-10 4th Payment 200,000
30-11 5th Payment 50,000 200,000

Page 36 of 46
92
Answer No. 5
Cost of Capital work In Process: Building
As on 31-12-2010

Payments (3.2 + 4.4 + 2.3 + 6.2) 16,100,000


Add borrowing cost to be capitalised on Specific borrowings 55,667
Add borrowing cost to be capitalised on General borrowing 775,250
16,930,917

As on 31-12-2011

Opening balance 16,930,917


Add Payments (5.5 + 4.4 + 2.5 + 1.5) 13,900,000
Add borrowing cost to be capitalised on Specific borrowing 455,667
Add borrowing cost to be capitalised on General borrowing 2,675,166
33,961,750

(W-1) borrowing cost to be capitalised on Specific Borrowings:

2010
Interest incurred 10,000,000 × 12% × 1/12 = 100,000
Interest income 3,800,000 × 14% × 1/12 = (44,333)
55,667

2011
Interest incurred 10,000,000 × 12% × 5/12 = 500,000
Interest income 3,800,000 × 14% × 1/12 = (44,333)
455,667

(W-2) Borrowing cost to be capitalised in case of General Borrowings:

2010 3,200,000 × 14% × 10/12 = 373,333


4,400,000 × 14% × 6/12 = 308,000
2,300,000 × 14% × 3.5/12 = 93,917
9,900,000 775,250
2011
On opening 9,900,000 × 14% × 11/12 = 1,270,500
balance
1,700,000 × 14% × 10/12 = 198,333
4,400,000 × 14% × 7/12 = 359,333
10,600,000 × 14% × 6/12 = 742,000
2,500,000 × 14% × 3/12 = 87,500
1,500,000 × 14% × 1/12 = 17,500
2,675,166

Page 37 of 46
93
(W-3) Detail of Payments:

Funds Used From


Date Particulars Amount
Specific General
1-3-2010 1st Payment 3,200,000 -- 3,200,000
nd
30-6-2010 2 Payment 4,400,000 -- 4,400,000
rd
16-9-2010 3 Payment 2,300,000 -- 2,300,000
th
1-12-2010 4 Payment 6,200,000 6,200,000 --
th
31-1-2011 5 Payment 5,500,000 3,800,000 1,700,000
th
1-5-2011 6 Payment 4,400,000 -- 4,400,000
1-6-2011 Loan repayment + Interest 10,600,000 -- 10,600,000
(10 + 0.6)
1-10-2011 7th Payment 2,500,000 -- 2,500,000
th
1-12-2011 8 Payment 1,500,000 -- 1,500,000

Ans: 6
ALPHA TRADING LIMITED
STATEMENT OF FINANCIAL POSITION (Extracts)
As on 31-12-2014
“Rs in Millions”

Non- Current Assets:


Capital work in progress-[ Office building(w-1)] 216.355
Current Asets:
Advance to controller [50-9.4-11.8] 28.8
Equity: -
Non- Current Liabilities:
Long term loan [100-25] 75
Retention money payable [4.7+5.9] 10.6
Current Liabilities:
Current portion of long term loan 25
Payable to contractor 100.3

Workings:

Detail of payments:
Date Particulars Amount Net payment Specific General
1-07-2014 Advance 50 50 50 -
31-10-2014 1st Bill 94(79.9/85%) 79.9 50 29.9
31-12-2014 Interest on loan 5.5 5.5 - 5.5
[100 X 11% X 6/12]

Page 38 of 46
94
Don’t commit sins and Fear Allah. Allah is all hearing, All seeing and All knowing.

Borrowing cost to be capitalized:


Specific borrowings:
Interest incurred:
100X 11% x 5/12 = 4.58
Investment income:
50 X 7% x 3/12 = (0.875)
3.705
General borrowings:
29.9 X *13% x 2/12 = 0.65
5.5 X 13% X 0/12 = -
*(24.7/190) X 100 = 13% 0.65

Total borrowings cost to be capitalized = [3.705+0.65] = 4.355

Cost of capital work in progress:


1st bill [79. 9/85%] = 94
2nd bill [100.3/85%] = 118
Borrowing cost = 4.355
216.355

Accounting entries: [Related to CWIP] (for understanding only):


1-07-2014 Advance 50
Cash 50
10-09-2014 CWIP 94
Advance (10%) 9.4
Retention money (5%) 4.7
Payable 79.9
31-10-2014 Payable 79.9
Cash 79.9
CWIP 118
31-12-2014
Advance (10%) 11.8
Retention money (5%) 5.9
Payable 100.3

Answer No. 7

All figures in Rs. In ‘000’


Cost of capital work in progress 2015 2014
Invoices raised (6,500 + 2,250):(2,000 + 4,500 + 4,750) 8,750 11,250
b/f from previous year 11,347 -
Specific borrowing (W-3) 981 -
Interest accrued on general borrowings (W-4) 292 97
21,370 11,347

Page 39 of 46
95
(W-1)

Running
Owned
Date Description Specific finance
resources
facility
31/7/14 Payment of 1st invoice (2,000)
nd
31/10/14 Payment of 2 invoice of Rs. 4,500 (1,000) (3,500)
rd
31/1/15 Payment of 3 invoice (4,750)
th
30/4/15 Payment of 4 invoice (6,500)

Last payment of Rs. 2,250 is made on 31 August, 2015 which is after the completion of project, hence ignored.
(W-2) Calculation of Capitalisation Rate

Period Loan
Loan Rate Interest
outstanding outstanding
Bank – A 25,000 × 12/12 25,000 18% 4,500
Bank – B 35,000 × 12/12 35,000 15% 5,250
Bank – C 10,000 × 9/12 7,500 20% 1,500
67,500 11,250
Rate on general borrowing (11,250/67,500) 16.67%

Note: There is no need to calculate Capitalisation rate in 2015 because in 2015 there is no payment from general
borrowings.
(W-3) Calculation of borrowing cost to be capitalised in case of Specific Borrowings

2015 2014
Interest accrued on specific borrowing (17,000 × 15% × 6(7-1)/12):(-) 1,275 -
Less: Investment income
(17,000 × 6% × 1/12): 85 -
(12,250 × 6% × 2(3-1)/12)): 123 -
(5,750 × 6% × 3/12): 86 -
(294) -
981

(W-4) Calculation of borrowing cost to be capitalised in case of General Borrowings

2015 2014
Interest accrued on general borrowing
(3,500 × 16.67% × 6(7-1)/12)):(3,500 × 16.67% × 2/12) 292 97

Page 40 of 46
96
If your dream is Jannah then follow Quran and Sunnah.

Summary of IAS 23
Borrowing Costs are interest and other costs (e.g. commitment fee/arrangement fee) incurred in connection with
borrowing of funds.

Qualifying Asset: is an asset that takes substantial time period to get ready for its intended use or sale examples of
borrowings;

Example of borrowing Related borrowing cost


Preference shares (if classified as liability) Preference dividend
Debentures, TFC's Interest expense
Long term / medium term loans /bank Overdraft Interest expense

In addition any bank charges e.g. commitment fee, charged by bank in connection with borrowings are also
borrowing cost.

Following is not borrowing cost.


 Dividend on ordinary share capital/right shares.

Examples of qualifying assets:


Depending upon the circumstances, following may be qualifying assets
I. Inventories (IAS-2)
II. Manufacturing plants and power generation facilities (IAS-l6)
III. Construction of a factory or head office building (IAS-16)
IV. Intangible assets e.g. any research and development project (IAS-38)

However inventories that are manufactured or produced over a short period of time are not qualifying assets.

Assets that are ready for their intended use or sale when acquired are qualifying assets (e.g. furniture, computer,
vehicles etc)

Recognition an entity shall recognize borrowing costs that are directly attributable to the acquisition, construction
or production of a qualifying asset as part of the cost of that asset. An entity shall recognize other borrowing costs
as an expense in the period in which they are incurred.

An example of acquisition is purchase of the building. An example of construction is manufacturing of plant.

An example of production is manufacturing of inventory.


1) Specific borrowings: borrowings which are taken for the sole purpose of construction, acquisition are
production of qualifying asset.
2) General borrowings: borrowings that are entered into for general purpose. These funds may be utilized for
buying inventories, paying off expenses and multitude of other purposes in addition to acquisition,
construction and production of qualifying asset.
 Specific borrowings: In case of specific borrowings, the amount to be capitalized is calculated as follows:

Amount of interest incurred on outstanding balance of loan xxxx


Less: investment income (if any) on temporary investment of surplus funds (xxx)
Amount of interest to be capitalized xxx

Page 41 of 46
97
General borrowings amount to be capitalized is calculated as follows
Expenditure on qualifying asset X capitalization rate X time period
Capitalization rate is to be calculated if there are more one general loan, otherwise the rate applicable on a single
general loan is capitalization rate.
borrowing cost incurred
Capitalization rate = × 100
weighted Borrowing outstanding
Period during which capitalization of borrowing cost must occur:

Commencement of capitalization
a) It incurs expenditures for the asset;
b) It incurs borrowing cost and undertake activities to prepare the asset; and
c) Activities necessary to prepare the asset have started.

The activities necessary to prepare the asset for its intended use or sale encompass more than physical
construction of the asset. They include technical and administrative work prior to commencement of physical
construction, such as the activities associated with attaining permits prior to commencement of the physical
construction of the asset. [Consider Spin Limited].

Suspension of Capitalization
An entity shall suspend capitalization of borrowing costs during extended periods in which it suspends active
development of a qualifying asset unless temporary delay is a necessary part of the process of getting the asset
ready for its intended use or sale.

Cessation of Capitalization
An entity shall cease capitalizing borrowing costs when substantially all the activities necessary to prepare the
Qualifying asset for its intended use or sale are complete.

Important considerations
 When determining whether your borrowings are either general or specific it is useful to remember that whilst
a bank overdraft facility is often used as general purpose borrowings, it is also possible for a bank overdraft
facility to be arranged specifically for a qualifying asset. The particular circumstances, should therefore always
be considered when deciding whether the borrowing is general or specific
 if an entity has funds available from its own resources, specific borrowings and general borrowings at any
given date then assume that:
• First; own funds are utilized
• Second; funds from specific borrowing ;and
• Then funds from general borrowings
• If an entity has funds available from:
 Owned resources
 Specific borrowings; and
• General Borrowings

At the date of repayment of principal and interest then assume that funds were utilized in above order.
 When construction of a qualifying asset is completed in parts and each part is capable of being used
separately while construction continues on other parts, capitalization of borrowing costs ceases when
substantially all activities necessary to prepare that part for its intended use or sale are complete.

Disclosure
a) Amount of borrowing Cost capitalized during the period
b) Capitalization rate used in case of general borrowings is to determine the amount of borrowing cost eligible
for capitalization.

Page 42 of 46
98
Further practice
Example1: SHAYAN LIMITED
Question: Shayan Limited (SL) started the construction of its new factory on 1 January 2018 with a loan of Rs.
50,000,000 borrowed at an interest rate of 8% per annum.

The loan was used on the factory as follows:

Date of Payment Rs. in million


Jan 1, 2018 25
May 1, 2018 15
Oct 1, 2018 10

The construction of the asset was completed on 31 December 2018. However, during the accounting period SL
invested the surplus funds at an interest rate of 3%.

Required:

How much the amount of borrowing cost eligible for capitalization at 31.12.2018?

Answer:
Borrowing costs to be capitalised Rs.
Borrowing costs incurred Rs. 50m x 8% 4,000,000
Less: Temporary investment income Rs. 25m x 3% x 4/12 (250,000)
Rs. 10m x 3% x 5/12 (125,000)
3,625,000

Example 02: S LIMITED


Question: On January 1, 2018 S Limited (SL) started the construction of an asset. To meet the financing general
requirements, borrowing was made from three different banks at the start of the year as follows:

Banks Amount (Rs.) Interest Rate per annum


A 70,000 10%
B 60,000 8%
C 50,000 12%

The funds were used on the assets as follows:

Date of Payment Rs.


Jan 1, 2018 30,000
May 1, 2018 20,000
Oct 1, 2018 15,000

The construction of asset was completed on 31 December 2018.

Page 43 of 46
99
Required: Calculate the general weighted average borrowing rate and eligible borrowing cost.

Answer:

Borrowing costs eligible for capitalization Rs.


30,000 x 9.89% x 12/12 2,967
20,000 x 9.89% x 8/12 1,318
15,000 x 9.89% x 3/12 371
Borrowing costs to be capitalised 4,656

Capitalisation rate = (70,000 x 10%) + (60,000 x 8%) + (50,000 x 12%) X 100 = 9.89%
70,000 + 60,000 + 50,000

Example 03: LOONEY


Question: Looney has recently finished building a new item of plant for its own use. The item is a press for use in
the manufacture of industrial diamonds. Looney commenced construction of the asset on 1st April 2013 and
completed it on 1st April 2015.
1st January 2013, Looney took out a loan to finance the construction of the asset. Interest is charged on the loan at
the rate of 5% per annum. The annual interest must be paid in four equal installments at the end of each quarter.
Looney capitalises interest on manufactured assets in accordance with the rules in IAS 23 Borrowing costs.
The costs (excluding finance costs) of manufacturing the asset were Rs. 28 million.

Required:
State the IAS 23 rules on the capitalisation of borrowing costs, calculate the cost of the asset oninitial recognition
and explain the amount of borrowing cost capitalised.
Answer:
IAS 23 should be applied in accounting for borrowing costs. Borrowing costs are recognised as an expense in the
period in which they are incurred unless they are capitalised in accordance with IAS 23 which says that borrowing
costs that are directly attributable to the acquisition, construction or production of a qualifying asset can be
capitalised as part of the cost of that asset.

 A qualifying asset is an asset that necessarily takes a substantial period of time to get ready for its intended
use or sale.
 Borrowing costs that are directly attributable to acquisition, construction or production are taken to mean
those borrowing costs that would have been avoided if the expenditure on the qualifying asset had not been
made.

When an enterprise borrows specifically for the purpose of funding an asset, the identification of the borrowing
costs presents no problem as the amount capitalised is the actual borrowing costs net of any income earned on the
temporary investment of those borrowings.

If funds are borrowed, generally, the amount of borrowing costs eligible for capitalisation is determined by
applying a capitalisation rate to the expenditures on that asset calculated as the weighted average of the
borrowing costs applicable to general borrowings.
IAS 23 also contains rules on commencement of capitalisation, suspension of capitalisation and cessation of
capitalisation.

Page 44 of 46
100
It is never too late to turn to Allah. Leave shirk and biddah and turn to Allah.

Amount capitalised Rs.000


Cost of manufacture 28,000
Interest capitalised (Rs.28m × 5% × 2 years) 2,800
30,800

Example 04: GOOGLY INDUSTRIES LIMITED


Question: On 1 January 2016 Googly Industries Limited (GIL) borrowed Rs.15 million to finance the production of
two assets, both of which were expected to take a year to build. Work started during 2016. The loan facility was
drawn down and incurred on 1 January 2016, and was utilised as follows, with the remaining funds invested
temporarily.

Asset A Asset B

----------- Rs. in million ---------


1 January 20X6 2.5 5
1 July 20X6 2.5 5

The loan rate was 9% and GIL can invest surplus funds at 7%.

Required: Calculate the borrowing costs which may be capitalised for each of the assets and consequently the
cost of each asset as at 31 December 2016.

Answer:
The borrowing cost to be capitalized & cost of assets are as under:
Cost of asset A Rs.
Expenditure 5,000,000

Borrowing costs incurred 5m (2.5+2.5)x 9% x 12/12 450,000


Less: Temporary investment income 2.5m x 7% x 6/12 (87,500)
362,500
5,362,000

Cost of asset B Rs.


Expenditure 10,000,000

Borrowing costs incurred 10m (5+5) x 9% x 12/12 900,000


Less: Temporary investment income 5m x 7% x 6/12 (175,000)
725,000
10,725,000

Page 45 of 46
101
Example 05: KHAN LIMITED
Question: Khan Limited (KL) has the following general loan arrangements as at 1 January 2020:

Rs. in million

7% Debentures 55
8% Loan notes 110
12% Line of credit 85
10% Running finance arrangement 150

On the 1 January 2020, KL commenced the construction of a new factory. The construction of the factory will cost
Rs.100 million and the company funded the construction with the existing borrowings. The factory was completed
on 31 August 2020 but was not available for use until 31 January 2021 as a result of minor modification. During the
construction period, active work was interrupted and the building construction was stopped for two months as a
result of adverse weather conditions.

Required:

Calculate the borrowing cost to be capitalised and the cost of the building to be recognised uponinitial recognition.

Answer:
The borrowing cost to be capitalized & cost of assets are as under:
Rs.
Expenditure 100,000,000
Borrowing costs to be capitalised 100,000,000 x 9.46% x 6/12 4,730,000
104,730,000

January to August are 8 months – two months suspension = 6 months

Capitalisation rate = (55 x 7%) + (110 x 8%) +(85 x 12%) + (150 x 10%) X 100 = 9.46%
55+110+85+150

Page 46 of 46
102
“Avoid sin but whenever you commit a sin, say “Astaghfirullah (I ask Allah for forgiveness).”

Statement of Cash Flow: (IAS-7)

While preparing statement of financial position and statement of comprehensive income we use accrual basis of
accounting. In accrual basis of accounting;

 Incomes are recorded when they are earned whether or not received; and
 Expenses are recorded when they are incurred whether or not paid.
However in a statement of cash flow information to users are provided on the basis of cash concept.
Therefore always consider cash flow statement just like a receipt & payment in a statement form.

 Receipt and payment A/c is a combined cash and bank A/c. e.g
Receipts and Payments
b/d 10,000 b/d 2,000 (Bank O/D)
Receipts 500,000 Payments 400,000
(Bank OD) c/d 12,000 c/d 120,000

A simplified cash flow statement from this receipt and payment account can be prepared as follows:
Receipts 500,000
Payments (400,000)
Net cash flow 100,000
Opening Balance (10,000 – 2,000) 8,000
Closing Balance (120,000 – 12,000) 108,000

Important points to remember:


 If there is no opening and closing balances of debtors then sale should be equal to receipt.
 If there is no opening and closing balances of creditors then purchase is equal to payment.
 If there is no opening and closing balances of stock then purchase is equal to cost of sales in a trading
organization.
 If there is no opening and closing balances of prepaid expenses or expenses payable then it means expense is
equal to payment.
 If there is no opening and closing balances of dividend payable then it means dividend declared is equal to
dividend paid.

Example:
Opening balance of advance tax is 70,000
Opening balance of income tax payable is 100,000
Current tax expense for the period 800,000
Closing balance of advance tax is 50,000

Page 1 of 72
103
Closing balance of income tax payable is 120,000
Required:
Calculate the tax paid during the period.

Solution
Advance Tax + Tax Payable
b/d 70,000 b/d 100,000
Cash (Bal) 760,000 Tax expense 800,000
c/d 120,000 c/d 50,000

Conclusion
Always prepare separate ledger of every expense payable or prepaid expense however;

If there is no breakup of expense into prepaid expense ledger and expense payable ledger then we can prepare a
combined ledger of prepaid expense and expense payable to calculate balancing figure of payment.

Format of Cash Flow Statement

Cash flow from operating activities X

Cash flow from investing activities X

Cash flow from financing activities. X

Net cash inflow (or outflow) during the period X

Cash and cash equivalents at the beginning of the period X

Cash and cash equivalents at the end of the period X

Important Definitions:

Financing Activities: Those activities which results into changes in equity and borrowing of business.

Investing Activities: Acquisition and disposal of *non-current assets and those investment not included in cash
and cash equivalents.

*Non-current Assets include Property, plant and equipment, intangible assets, capital work in progress and long
term investments etc.

Operating Activities Principal revenue producing activities of the business; e.g Receipts from customers, payment
to suppliers, payments for expenses etc.

Example
Identify operating, Investing and financing activities from the following:
a) Cash received from debtor
b) Cash received from sale of goods
c) Cash received from sale of fixed assets
d) Cash received from issue of shares
e) Cash repayments of amount borrowed
f) Dividend received
g) Dividend paid

Page 2 of 72
104
h) Wages paid to employees
i) Interest paid
j) Interest received
k) Purchase of fixed assets
l) Acquisition of fixed asset by issue of shares
m) Cash advances to suppliers
n) Taxes paid
o) Depreciation charge for the year
p) Cash paid to purchase goods
q) Loan given by the company to its subsidiary company

List of Non-cash Transactions


 Revaluation surplus/Loss
 Amortization
 Bad debts expenses/ Bad debt written off etc.

Cash flows between cash and cash equivalents are not shown in statement of cash flows. e.g. cash deposited into
or withdrawn from bank (because they are cancelled out)

Components of cash & cash equivalents

Cash in hand -
Cash at bank -
*Short term investment -
Bank overdraft (-)
Total of cash & cash equivalents -
Definitions:

Cash: comprises of cash in hand and demand deposits (means cash at bank)

Cash equivalents: means short term highly liquid investments that are readily convertible into cash.

*Short term investments in cash flow statement means those investments having a maturity date of 3 months or
less from the date of acquisition (i.e date of investment)

For example
 Amount investment in bank on 15-6-2015 is Rs 1,000,000
 Statement of Financial Position date is 30-6-2015
 Encashment date (i.e maturity date) is 15-12-2015

It is a short term investment for the purpose of preparing statement of financial position but for cash flow
statement it is not a cash equivalent investment. Had the maturity date is upto 15-9-2015, then this short term
investment should be classified as cash equivalent for the purpose of cash flow statement.

Page 3 of 72
105
“Allah is sufficient for believers.”

Presentation of Cash Flow from Operating Activities


 
Direct Method Indirect Method

Direct Method (Format) Assumed Figures


Statement of cash flows: direct method Cash flows from operating activities Rs.

Cash receipts from customers xxx


Cash payments to suppliers (xxx)
Cash payments to employees (xxx)
Cash paid for other operating expenses (if any) (xxx)
Cash generated from operations xxx
Taxation paid (Xxx)
Interest charges paid (Xxx)
Net cash flow from operating activities xxx

Q.1 The following data relates to Shahnawaz Sports (Private) Limited for the year ended June 30,

2007 2006
Particulars
Rs. Rs.
Inventory 230,000 185,000
Prepaid expenses 14,000 16,000
trade debtors 52,000 30,000
Cash 15,000 38,000
Accounts payable 39,000 44,000
Income tax payable 5,000 4,000
Sales 500,000
Cost of sales 310,000
Operating expenses 80,000
Interest expenses 21,000
Taxation 30,000
Depreciation - included in operating expenses 6,000
included in cost of goods sold 6,000
Profit before tax 89,000

Required:
What will be the net cash flow from operating activities for the year ended 30.06.2007 as under?
(i) Direct method
(ii) Indirect method

Indirect Method (Format):


Cash flow from operating activities:
Profit before tax Xxx
Adjustment for:
i) Non-cash Transactions
e.g Depreciation/ Amortization Xxx

Page 4 of 72
106
Bad debts/bad debts reversed xxx/(xxx)
Revaluation Loss etc Xxx
ii) Those items effect of which should have been in investing activities:
e.g Gain on disposal/Loss on disposal (xxx)
Dividend income (xxx)
Interest Income etc (xxx)
iii) Increase/decrease in some items of current assets and current liabilities:
Increase in Current Asset  In cash flow statement - ve figure
Decrease in Current Asset  In cash flow statement + ve figure
Increase in Current Liabilities  In cash flow statement + ve figure
Decrease in Current Liabilities  In cash flow statement - ve figure

Cash generated from operations xx


Interest paid (xx)
Tax Paid (xx)
Net cash from Operating Activities xx

Important points to remember:


 In a question of cash flow statement, if a statement of financial position is given then it means that all
accounting entries should have been correctly made and recorded unless there is any indication in the
question.
 Short term finance can be included in cash and cash equivalents as bank overdraft or can be classified in
financing activities. Preferably include in financing activities.
 If information is available then always prepare separate ledger of cost and accumulated depreciation.
Similarly information is available prepare separate ledger of debtors without deducting the provision and
separate ledger of provision for bad debts.
 If there is no information about items of income statement then statement of cash flow can not be prepared
by using the direct method.
 Always prepare statement of cash flow by using indirect method unless examiner requires to use the direct
method.

Sequence of questions

Extra questions 1, universal, quality

Data traders

Discussion of share capital and dividend

Nadir ltd

Page 5 of 72
107
Answers of practice questions

A.1 i) Direct Method


Shahnawaz Sports (Pvt) Limited
Statement of Cash Flow
For the year ended June 30,2007

Cash flow from Operating Activities:


Cash received from debtors 478,000
Cash paid to suppliers (354,000)
Expenses paid (72,000)
Cash generated from Operations 52,000
Interest paid (21,000)
Taxes Paid (29,000)
Net cash from Operating Activities 2,000

Debtors Accounts Payable


b/d 30,000 Cash 478,000 Cash 354,000 b/d 44,000
Sales 500,000 c/d 52,000 c/d 39,000 Purchase 349,000

Inventory Prepaid Expense


b/d 185,000 Cost of sales (310-6) 304,000
b/d 16,000 Expenses 74,000
Purchase 349,000 c/d 130,000
cash 72,000 c/d 14,000

Taxation Payable
Cash 29,000 b/d 4,000
c/d 5,000 Tax 30,000

ii) Indirect Method


Shahnawaz Sports (Pvt) Limited
Statement of Cash Flow
For the year ended June 30,2007
Profit before tax 89,000
Depreciation 12,000
Interest Expense 21,000
Operating Profit before working capital 122,000
Changes in working capital:
Prepaid Expense 2,000
Inventory (45,000)
Debtors (22,000)
Accounts Payable (5,000)
Cash generated from operations 52,000
Interest Paid (21,000)
Tax Paid (29,000)
Net cash from operating Activities 2,000

Page 6 of 72
108
“I tell my grief to Allah.”

FORMATS

Format-Direct Method Assumed Figures


Statement of cash flows: direct method Rs.
Cash flows from operating activities
Cash receipts from customers 348,800
Cash payments to suppliers (70,000)
Cash payments to employees (150,000)
Cash paid for other operating expenses (if any) (30,000)
Cash generated from operations 98,800
Taxation paid (21,000)
Interest charges paid (2,500)
Net cash flow from operating activities 75,300
Cash flows from investing activities:
Acquisition of shares (debentures, etc.) (5,000)
Purchase of property, plant and machinery (35,000)
Proceeds from sale of non-current assets 6,000
Interest received/dividends received 1,500
Net cash used in investing activities (32,500)
Cash flows from financing activities:
Proceeds from issue of shares 30,000
Proceeds from new loan 10,000
Repayment of loan (17,000)
Dividends paid to shareholders (25,000)
Net cash used in financing activities (2,000)
Net increase/decrease in cash/cash equivalents 40,800
Cash/cash equivalents at the beginning of the year 5,000
Cash/cash equivalents at the end of the year 45,800

Format The indirect method Assumed Figures


Statement of cash flows: indirect method Rs. Rs.

Cash flows from operating activities

Profit before taxation 80,000


Adjustments for:
Depreciation and amortization charges 20,000
Interest charges in the statement of profit or loss 2,300
Gains on disposal of non-current assets (6,000)

Losses on disposal of non-current assets 4,500


Profit before working capital changes 100,800
Increase in trade and other receivables (7,000)
Decrease in inventories 2,000

Page 7 of 72
109
Increase in trade payables 3,000
Cash generated from operations 98,800
Taxation paid (21,000)
Interest charges paid (2,500)
Net cash flow from operating activities 75,300

Cash flows from investing activities:


Acquisition of shares (debentures, etc.) (5,000)
Purchase of property, plant and machinery (35,000)
Proceeds from sale of non-current assets 6,000
Interest received/dividends received 1,500
Net cash used in investing activities (32,500)
Cash flows from financing activities:
Proceeds from issue of shares 30,000
Proceeds from new loan 10,000
Repayment of loan (17,000)
Dividends paid to shareholders (25,000)
Net cash used in financing activities (2,000)
Net increase/decrease in cash/cash equivalents 40,800
Cash/cash equivalents at the beginning of the year 5,000
Cash/cash equivalents at the end of the year 45,800

Page 8 of 72
110
“May Allah forgive us and guide us all to right path.”

Extra practice:
Q.1 Following is the abridged balance sheet of Platinum International Traders Limited as at June 30, 2006.

Rs. 000
Particulars
2006 2005
Property, plant and equipment 10,500 10,000
Intangible assets 1,200 1,000
Short term investments 750 500
Stocks 1,290 1,000
Trade receivables (net of provisions) 450 400
Advances and deposits 200 200
Cash and bank balances 160 100
Bank overdraft 400 200
Creditors and other liabilities 500 400
Provision for taxation 2,250 1,500
Net Assets 11,400 11,100
Net Equity and Surplus on Revaluation of Fixed assets 11,400 11,100

The following additional information is also available:


(i) Accounting profit after tax is Rs. 3,000,000.
(ii) Income tax paid during the year amounted to Rs. 1,000,000.
(iii) Depreciation for the year is Rs. 750,000.
(iv) Provision for doubtful debts at the end of the year amounted to 5% of closing balance of trade
receivables. The opening balance of the provision was NIL.
(v) 48,000 shares were issued during the year at a premium of Rs. 2.50 per share. Face value of the
company’s share is Rs. 10.
(vi) Markup paid during the year amounted to Rs. 500,000.
(vii) An old machine having book value of Rs. 500,000 was exchanged with a new machine costing Rs.
950,000. The company paid Rs. 350,000 as the difference. No other fixed asset was sold during the year.
(viii) Intangibles are amortized at 35% of book value.
(ix) The company revalues its major fixed assets periodically. A revaluation carried out during the year
showed that the value of fixed assets had declined by Rs. 3,300,000. The amount was charged against
surplus recorded in earlier years.

Required:
Prepare a cash flow statement under ‘indirect method’ in accordance with IAS-7. Also submit necessary workings.

Note:
1. If only statement of financial position is given in a question of statement of cash flow then question cannot
be solved by using direct method.
2. If the figure of debtors given in statement of financial position is net of allowance for doubtful debts then
prepare separate workings for gross debtors and allowance for doubtful debts. From allowance a/c find out
bad debts for the period and difference in gross debtors should be presented in working capital changes (if
indirect method is used).
3. In a question of cash flow statement, if a statement of financial position and/or statement of comprehensive
income is given then it means all accounting entries would have been correctly made and recorded.
4. Short term investments can be classified as cash and cash equivalents or can be presented in investing
activities. Do not include short term investments in cash and cash equivalents unless maturity date is given or

Page 9 of 72
111
it is mentioned in question that short term investments are highly liquid investments readily convertible into
cash.

Q.2 The following information has been extracted from its financial statements of Sky Limited (SL) for the
year ended 30 June 2014.
Debit Credit
Particulars
Rs. in million
Sales 172
Cost of sales 80
Operating and selling expenses 40
Bad debt expense 6
Loss on settlement of insurance claim 2
Finance charges paid 8
Taxation expense 15
Closing stock in trade 10
Trade receivables 28
Provision for doubtful debts 6
Trade payables 20
Provision for taxation 5
Property, plant and equipment - WDV 105

Additional information:
(i) At the sbeginning of the period, the assets and liabilities were valued as under:
Rs. in million
Property, plant and equipment 52

Stock in trade 4
Trade receivables 8
Trade payables 12
(ii) During the year, SL incurred a capital expenditure of Rs. 70 million.
(iii) Loss on settlement of insurance claim relates to a car which was destroyed in an accident. Its cost and
written down value at the time of accident was Rs. 5 million and Rs. 4 million respectively. There were no
other disposals during the year.

Required:
1. Prepare operating activities section of the statement of cash flows for the year ended 30 June 2014 using the
direct method in accordance with the International Financial Reporting Standards.
2. Prepare operating activities section of the statement of cash flows for the year ended 30 June 2014 using the
indirect method in accordance with the International Financial Reporting Standards.

Q.3 One of your clients has contacted you to prepare cash flow statement as per the requirements of IAS-7
and has provided you the following information.
2005 2004
Rupees in ‘000’
Cash and bank 21,750 17,000
Trade & other receivable 17,000 13,400
Stocks 14,000 12,000
Investments-(Non-current) - 4,000
Building 28,000 35,000

Page 10 of 72
112
Equipments 40,000 20,000
Intangible Asset 5,000 6,250
Provision for doubtful debts 3,000 4,500
Accumulated depreciation-Equipments 3,500 6,000
Accumulated depreciation-Building 8,500 12,000
Creditors 12,000 10,000
Dividend payable - 6,000
. Current maturity of long term loans 3,000 4,000
Long term loans 33,000 29,000
Issued, subscribed & paid up capital 40,000 28,000
Un-appropriated profit 22,750 8,150

Additional data relating to the accounts for the year ended June 30, 2005 is as follows:

 Equipment that had cost Rs. 11 million and was 40% depreciated at the time of disposal was sold for Rs. 2.5
million.
 Investments were sold at Rs 2.5 million above their cost.
 Rs. 12 million of the long term loan was settled by issuing, 1,200,000 ordinary shares of Rs.10 each.
 Cash dividend of Rs. 6.0 million was paid on September l, 2004.
 A long term loan of Rs. 16 million was obtained to Finance the purchase of equipment.
 On July 1, 2004 a portion of the building was completely destroyed by fire. Insurance claim of Rs. 15 million
was received from the insurance company. The additions to the building during the year amounted to Rs.
10.5 million and depreciation provided during the year was Rs 2.0 million.
 Interest and income taxes paid during the year were Rs. 2 million and Rs. 5 million respectively.

Required:
Prepare cash how statement for the year ended June 30, 2005.

Q. 4 A summary of revenues and expenses of AB Enterprise for the year ended 30 June 2013 is given below:

Rupees
Sales 2,345,000
Cost of goods manufactured and sold (1,624,000)
Gross profit 721,000
Selling, general and administrative expenses (509,000)
Net income before income tax 212,000
Income tax (90,000)
Net income 122,000

Net changes in working capital items for the year ended 30 June 2013 were as follows:

Net changes

Dr. Cr.
Cash 32,000
Trade receivables (net) 74,000
Inventories 105,000
Prepaid expenses (selling and general) 6,000

Page 11 of 72
113
Accrued expenses 15,000
Income tax payable 28,000
Trade payables 90,000

Depreciation for the year amounted to Rs. 68,000.

Required:
Prepare a cash flow statement for the year ended 30 June 2013.

Q. 5 Galaxy Brothers commenced their business on 1 January 2013 with cash of Rs. 50 million, a building valued at
Rs. 25 million and a motor vehicle costing Rs. 1.4 million. Following is the summarised Trial Balance as of 31
December 2013:

Rs. In millions
Particulars Debit Credit

Sales 136.00
Cost of sales (including depreciation expense of Rs. 9 million) 83.5
Operating and selling expenses (including depreciation expense of Rs. 6.25 million) 37.3

Miscellaneous income (net of loss of Rs. 0.35 on settlement of total loss


claim) 0.50
Finance charges 2.50
Taxation expense 6.00
Cash and bank balances 5.00
Bank overdraft 23.00
Accounts receivable 18.00
Provision for doubtful debts 0.90
Closing inventory 10.00
Accounts payable 14.00
Interest payable 1.20
Provision for taxation (net of payments) 1.00
Partners' capital (net of cash withdrawals) 73.95
12% Long term loan payable 25.00
Property, plant and equipment 128.25
Accumulated depreciation 15.00

290.55 290.55

Settlement of the insurance claim pertained to an accident of a new car costing Rs. 1.8 million and having a
depreciation charge of Rs. 0.25 million for the period in use.

Page 12 of 72
114
“Fragrance of perfume is not enough, try to bring Fragrance with your good deeds.”

Required:
Prepare a statement of cash flow for the year ended 31 December 2013. (13)

Q. 6 Financial statements of Data Traders are as follows:


30 June 2014 30 June 2013
Assets: Rs. In ‘000’ Rs. In ‘000’
Non-Current Assets:
Fixed assets 161,300 160,500
Investment 20,500 12,200
Current Assets:
Stock in trade 34,200 36,600
Short term investment-Cash equivalents 1,500 2,300
Trade debts (Net) 25,500 18,700
Prepayments 2,700 3,000
Cash and bank 4,100 --
249,800 233,300
Equity and Liability:
Capital 166,100 139,900
Revaluation surplus (building) 8,000 3,000

Non-Current Liabilities:
Bank loan 20,300 22,500
Current liabilities:
Trade payables 22,400 35,100
Accrued expenses 33,000 28,800
Overdraft -- 4,000
249,800 233,300
Additional Information:
(i) Provision for doubtful debt at 30 June 2014 was Rs. 1.2 million (2013: Rs. 1.7 million.)
(ii) During the year total payment of 7.6 million including interest was made to bank against loan.
(iii) Proprietor makes a monthly drawing of Rs. 3 million. Additionally, he withdrew stock of Rs. 8 million during
the year.
(iv) Interest and tax expense for the year was Rs. 3.5 million and Rs. 20.1 million respectively.
(iv) Accrued expenses include interest payable of Rs. 1.1 million (2013: Rs. 0.4 million) and tax payable of Rs.
4.5 million (2013: Rs. 3 million).
(vi) Following is the details of fixed assets:
30 June 2014 30 June 2013
Rs. In ‘000’ Rs. In ‘000’
Land and building 88,700 85,200
Plant and machinery 65,600 67,300
Intangibles 7,000 8,000
161,300 160,500
(a) Renovation of building of Rs. 3 million was capitalized during the year.
(b) Proprietor purchased machines of Rs. 10 million for business from his own pocket.

Page 13 of 72
115
(c) Depreciation expense of land & building and plant and machinery was Rs. 4.5 million and 6.5 million
respectively for the year just ended.
(d) Few machines were sold at profit of Rs. 500,000.

Required:
Prepare statement of cash flows for the year ended 30 June, 2014 using indirect method as per IAS-7. (18)

Q.7 Following are the extracts from income statement of Quality Enterprises (QE) for the year ended 31
December 2015 and its statement of financial position as al that date, together with some additional information:

Income statement for the year ended 31 December 2015


Rs. in '000
Profit from operations 6,402
Other income 1,357
Interest expense (100)
Profit before tax 7,659
Income tax expense (1,376)
Profit for the year 6,283

Statement of financial position as at 31 December 2015


2015 2014 2015 2014
Equity and liabilities Assets
— Rs. in '000 — --- Rs. in '000 ---
Non-current assets
Owner's capital 14,219 10,703 Property, plant and equipment 19,628 11,845
Un-appropriated profit 10,652 6,697 Investments 7,645 6,498
27,273 18,343
Revaluation surplus 2,676 1,911
10%bank loan 6,000 -
Current liabilities Current assets
Trade and other payables 3,337 4,953 Inventories 4,642 3,073
Income tax payable 1,300 994 Trade and other receivables 2,273 3,865
Bank overdraft - 27 Cash and bank 3,996 4
4,637 5,974 10,911 6,942
38,184 25,285 38,184 25,285

Additional information:
(i) During the year, movements in property, plant and equipment include:
 Depreciation amounting to Rs. 5,280,000.
 Machinery having a carrying amount of Rs. 2,481,000 was sold for Rs. 3,440,000.
 Factory building was revalued from a carrying amount of Rs. 5,963,000 to Rs. 8,000,000.
 An office building which had previously been revalued, was sold at its carrying amount of Rs.
2,599,000.
(ii) The owner of QE withdrew Rs. 300,000 per month. The amounts were debited to un-appropriated
profit.

Page 14 of 72
116
(iii) Trade debts written off during the year amounted to Rs. 200,000. The provision for bad debts as at
31 December 2015 was Rs. 400,000 (2014: Rs. 550,000).
(iv) The interest on bank loan is payable on 30th June every year. The bank loan was received on 1
November 2015. Interest for two months has been accrued and included in trade and other
payables.
(v) Other income includes investment income of Rs. 398,000. As at 31 December 2015, trade and other
receivables included investment income receivable amounting to Rs. 96,000 (2014: Rs. 80,000).
Required:
Prepare a statement of cash flows for Quality Enterprises for the year ended 31 December 2015, using the
indirect method. (18)
Q.8
Following are the extracts from the financial statements of Universal Limited (UL) for the year ended 30 June
2017:
Statement of Financial Position as on 30 June 2017

2017 2016 2017 2016


Assets Equity & Liabilities
Rs. in 000 Rs. in 000
Property, plant and 158,500 120,000 Share capital (Rs. 10 each) 175,000 150,000
equipment
Stock in trade 58,000 45,000 Retained earnings 54,434 21,500
Trade receivables 68,000 56,000 Revaluation Surplus 10,000 -
Cash 47,934 48,000 Debentures 18,000 26,000
Interest payable 1,000 2,500
Trade payables 42,000 39,000
Accrued liabilities 20,000 18,000
Unearned maintenance 2,000 4,000
Provision for taxation 10,000 8,000
332,434 269,000 332,434 269,000

Statement of profit or loss for the year ended 30 June 2017

Rs. in ‘000’
Sales 273,000
Cost of sales (187,500)
Gross profit 85,500
Operating expenses (46,766)
Other income 11,200
Profit before interest and tax 49,934
Interest expense (2,000)
Profit before tax 47,934
Tax expense (15,000)
Profit after tax 32,934

Page 15 of 72
117
“Don’t speak lie but speak the truth. Harm to none, help to everyone.”

Additional information:
(i) 60% of sales were made on credit.
(ii) UL maintains a provision for doubtful receivables at 6%. During the year, trade receivables of Rs. 7 million
were written off.
(iii) Depreciation expense for the year was Rs. 22.5 million. 70% of the depreciation was charged to cost of
sales.
(iv) Other income comprises of:
 Gain of Rs. 3 million on disposal of vehicles for Rs. 12 million;
 Maintenance income of Rs. 8.2 million

Required:
Prepare UL’s statement of cash flows for the year ended 30 June 2017 using direct method. (15)

Summary of discussion about dividend


It is a distribution of profits to owners (shareholder). If dividend is given as a percentage then multiply the rate
with the amount of share capital (nominal value of share capital) which is at the date of declaration of dividend to
calculate the amount of dividend. A company may pay dividend either in cash or in form of shares.
Dividend
Cash Dividend (Either Final or Interim) Bonus Shares (Either Final or Interim)

When Declared Dividend XXX


Dividend(Retained Earnings) XXX
Dividend Payable A/c XXX Share Capital A/c XXX

When Paid
Dividend Payable A/c XXX
Cash/ Bank A/c XXX

 Bonus shares are distributed from share premium and if the balance of premium is not available then
from retained earnings.
 Dividend is recognized on the date of declaration.
 If the dividend is declared after reporting date but before the authorization of the financial statements it
is disclosed in the financial statement to which it relates.
 Dividend is adjusted against retained earnings in the statement of changes in equity,
 If the date of declaration is not given, then:
1. For interim dividend assume that dividend is declared during the accounting period.
2. For final dividend assume declared after the reporting date.
 If nature of dividend (means whether final or interim) is not given then assume final dividend.
 If type of dividend (means cash or bonus) is not available then assume cash dividend.

Page 16 of 72
118
“Repent and seek forgiveness from Allah. Allah forgives all sins.”

Answers
A.1
Platinum International Traders
Statement of Cash Flow
For the year ended 30-6-2006
Rs 000 Rs 000
Cash flow from operating activities
Profit before tax (3,000 + 1,750) 4,750
Depreciation 750
Bad debts 24
Interest Expense 500
Amortization of Intangibles 646
Gain on exchange of machine (100)
Profit from operations before working capital changes 6,570
Working capital changes
Increase in stock in trade (290)
Increase in trade receivable (74)
Increase in creditors 100
Cash generated from operations 6,306
Income tax (1,000)
Interest paid (500)
Net cash flow from operating activities 4,806
Cash flow from investing activities
Property, Plant and equipment (4,100 + 350) (4,450)
Intangible assets (846)
Short term investment (750-500) (250)
Net cash used in investing activities (5,546)

Cash flows from financing activities


Cash received from issuance of share (48,000 x 12.5) 600
Net cash flow from financing activities 600
Net decrease in cash and cash equivalents (140)
Cash and cash equivalents at the beginning of the period (100)
Cash and cash equivalents at the end of the period (240)

Working
W.1 2006 2005
Note for cash and cash equivalent 160 100
Cash and bank balance (400) (200)
Bank overdraft (240) (100)

W.2 Rs 000
Profit before tax
Profit after tax 3,000
Income tax expenses (W-3) 1,750
Profit before tax 4,750

Page 17 of 72
119
W.3
Provision for taxation
Cash 1,000 b/d 1,500
c/d 2,250 Tax expense (bal) 1,750
3,250 3,250

W.4
Machinery Disposal
Fixed Asset (book value) 500 Fixed Asset (950-350) 600
Profit & Loss (Gain) 100
600 600

W.5
Fixed Assets
b/d 10,000 Depreciation 750
Disposal 600 Disposal 500
Cash (bal) 4,100 Revaluation Surplus 3,300
Cash (machine) 350 c/d 10,500
15,050 15,050

W.6
Intangible Assets
b/d 1,000 Amortization 646
(1,200÷65% x 35%)
Cash (bal) 846 c/d 1,200
1,846 1,846

W.6
Net Equity & Surplus on Revaluation of Fixed Asset
b/d 11,100
Revaluation Deficit 3,300 Profit for the year 3,000
c/d 11,400 Cash (shared issue) (480+120) 600
16,450 16,450

A.2
1.
Sky Limited
Extracts from Statement of Cash Flows (direct method)
for the year ended 30 June 2014
Rs. in million
Cash flows from operating activities
Cash receipts from customers 152
Cash paid to suppliers (65)
Cash paid for expenses (40)
Cash generated from operations 47
Interest paid (8)
Taxation paid (10)
Net cash inflow from operating activities 29

Page 18 of 72
120
“Don’t be sad nor grieve. Allah is with us.”

Stock
Debtor b/d 4 Cost of sales 67
(80-13)
b/d 8 Cash 152
Purchases 73 c/d 10
Sales 172 c/d 28

Creditor PPE-WDV
Cash 65 b/d 12 b/d 52 Disposal 4
c/d 20 Purchase 73 Cash 70 Depreciation 13
c/d 105

Tax Payable
Cash 10 b/d -
c/d 5 Expense 15

2.

Sky Limited
Extracts from Statement of Cash Flows (indirect method)
for the year ended 30 June 2014
Rs. in million
Cash flows from operating activities
Profit before tax (172-80-40-6-2-8) 36
Adjustments for:
Depreciation 13
Bad debts 6
Loss on disposal 2
Financial charges 8
Profit before working capital changes 65
Changes in working capital
Stock (10 - 4) (6)
Trade receivables (28 - 8) (20)
Trade payables (20 - 12) 8
Cash generated from operations 47
Interest paid (8)
Taxation paid (14)
Net cash inflow from operating activities 29

Page 19 of 72
121
A.3
CLIENTS
Rs. 000
Cash flow from operating activities
Profit before taxation (14,600 + 5,000) 19,600
Adjustments for:
Depreciation: Equipment 1,900
Depreciation: Building 2,000
Gain on investment 2,500
Loss on sale of equipment (W3) 4,100
Amortization of intangible asset 1,250
Interest expense 2,000
Gain on receipt of insurance claim (W4) (3,000)
Reversal of provision for doubtful debts (1,500)
Operating profit before working capital changes 23,850
Working capital changes:
Increase in trade and other receivables (3,600)
Increase in stock (2,000)
Increase in creditors 2,000
Cash generated from operations 20,250
Interest paid (2,000)
Income tax paid (5,000)
Net cash generated from operating activities 13,250

CASH FLOW FROM INVESTING ACTIVITIES (31,000)


Purchase of equipment (W1) 2,500
Sale proceeds of equipment (10,500)
Purchase of building (W2) 15,000
Insurance claim received 6,500
Receipts from sale of investments
Net cash used in investing activities (17,500)
CASH FLOW FROM FINANCING ACTIVITIES
Long term loan received 16,000
Repayment of long term loan (1,000)
Dividend paid (6,000)
Net cash generated from financing activities 9,000
Net increase in cash and cash equivalents 4,750 17,000
Cash and bank at beginning of year
Cash and bank at end of the year 21,750

Page 20 of 72
122
“If you want that Allah would love you then obey Allah.”

ADDITITIONAL DISCLOSURES REGARDING NON-CASH INVESTING ANFD FINANCIANG TANSACTION

During the year Rs 12 million of the long term loan was settles by issuing 1,200,000 ordinary shares of Rs 10 each.

Workings
W-1
Equipment
b/d 20,000
Cash 31,000 Disposal 11,000
c/d 40,000
W-2
Building
b/d 35,000
Cash 10,500 Disposal 17,500
c/d 28,000
W-3
Disposal - Equipment
Equipment 11,000 Cash 2,500
Acc depreciation 4,400
Loss 4,100
W-4
Disposal - Building
Building 17,500 Cash 15,000
Gain 3,000 Acc Depreciation 5,500
W-5
Acc Depreciation- Building
Disposal 5,500 b/d 12,000
c/d 8,500 Depreciation 2,000
W-6
Acc Depreciation- Equipment
Disposal 4,400 b/d 5,000
c/d 3,500 Depreciation 1,900

Retained Earnings
b/d 8,150
c/d 22,750 Profit after tax (bal) 14,600

Long Term Loan


Share capital 12,000 b/d 33,000
Cash 1000 Cash 16,000
c/d 36,000 Depreciation 1,900

Page 21 of 72
123
A.4
AB Enterprise
Cash flow for the year ended 30 June 2013
--Rs.in 000’s-
Net income before income tax 212
Add non-cash item: Depreciation 68
280
Changes In working capital:
Increases: Debtors 74
Prepaid expenses 6 80

Decreases:
Inventory 105
Accrued expenses 15
Trade payables 90 210 (130)
150
Income tax paid (90+28) (118)
Increase in cash 32

Tax payable
Cash 118,000 b/d (suppose) 100,000
c/d 72,000 Expense 90,000

A.5

Galaxy Brothers
Statement of Cash Flow
For the Year Ended 31-12-2013
Rs. In Millions
Cash flow from operating activities
Profit before tax (136 – 83.5 – 37.3 + 0.5 – 2.5) 13.2
Adjustment for:
Depreciation (9 + 6.25) 15.25
Finance charge 2.50
Bad debts 0.90
Loss on settlement of claim 0.35
Profit before increase / decrease 32.20
Increase / Decrease:
Accounts receivables (18.00)
Inventories (10.00)
Accounts payable 14.00
Cash generated from Operations 18.20
Finance charge paid (1.30)

Page 22 of 72
124
Income taxes paid (5.00)
Net Cash from operating activities 11.90
Cash Flow From Investing Activities:
Purchase of property, plant & equipment (103.65)
Proceeds from settlement of claim 1.20
Net cash from investing activities (102.45)
Cash Flow From Financing Activities:
Cash contributions by partners 50
Drawings made (2.45)
Proceeds from long term loan 25.00
Net cash from financing activities 72.55
Net cash Flow (18.00)
Cash and Cash equivalents at the beginning --
Cash and cash equivalents at the end (18.00)

WORKINGS:

2013 2012
Cash & Bank 5.00 --
Bank overdraft (23.00) --
(18.00) --

Property Plant and Equipment


b/d -- Disposal 1.8
Capital 25
Motor vehicle 1.4
103.65 c/d 128.25

Interest Payable

Cash 1.3 b/d --


Expense 2.5
c/d 1.2

Tax Payable

Cash 5 b/d --
Expense 6
c/d 1

Disposal Account
Asset 1.8 Account Depreciation 0.25
Cash 1.2

Page 23 of 72
125
Loss 0.33

Accumulated Depreciation
Disposal 0.25 b/d --
Expense 15.25
c/d 15

Capital Account

Cash (drawings) 2.45 b/d --


Cash 50
Building 25
MV 1.4
c/d 73.95

*(As income statement is not yet prepared so capital account is without profit)

A.6
Statement of Cash Flows
For the year ended 30 June 2014

Rs. In ‘000’ Rs. In ‘000’


Operating actives:
Profit before tax (W-1: 60,200 + 20,100) 80,300
Interest expense 3,500
Depreciation (4,500 + 6,500) 11,000
Amortization (8,000 – 7,000) 1,000
Decrease in provision (1,200 – 1,700) (500)
Profit on sale of machine (500)
94,800
Working capital adjustments:
Increase in inventory (W-2) (5,600)
Increase in debtors (20,400 – 26,700) (6,300)
Decrease in prepayments (3,000 – 2,700) 300
Decrease in payables (35,100 – 22,400) (12,700)
Increase in accrued expenses (W-3) (27,400 – 25,400) 2,000
Cash generated from operation 72,500
Interest paid (W-4) (2,800)
Tax paid (W-5) (18,600)
Net cash from operating activities 51,100
Investing Activities:
Cash received from disposal of machinery (W 6) 5,700

Page 24 of 72
126
Cash paid for renovation (3,000)
Cash paid for investment (20,500 – 12,200) (8,300)
Net cash from investing activities (5,600)
Financing Activities:
Drawings (3,000 × 12) (36,000)
Loan obtained (W-8) 2,600
Loan repaid (7,600 – 2,800) (4,800))
Net cash from investing activities (38,200)

Net cash Flow 7,300


Add: Opening cash and cash equivalents (2,300 – 4,000) (1,700)
Closing cash and cash equivalents (1,500 + 4,100) 5,600

(W-1) Capital Account All figures in ‘000’

b/d 139,900
Cash drawings 36,000 Profit after tax (bal) 60,200
Stock drawings 8,000 Machine 10,000
c/d 166,100
210,100 210,100
(W-2) Inventory
b/d 36,600 Capital 8,000
Increase (bal) 5,600
c/d 34,200
42,200 42,200

(W-3)
30 June 2014 30 June 2013
Rs. In ‘000’ Rs. In ‘000’
Accrued expenses 33,000 28,800
Interest payable 1,100 (400)
Tax payable 4,500 (3,000)
Remaining 27,400 25,400

(W-4) Interest Payable

b/d 400
Cash 2,800 Expense 3,500
c/d 1,100
3,900 3,900

(W-5) Tax Payable

Page 25 of 72
127
b/d 3,000
Cash 18,600 Expense 20,100
c/d 4,500
23,100 23,100

(W-6) Plant and Machinery

b/d 67,500 Depreciation 6,500


Capital 10,000 Disposal (bal) 5,200
c/d 65,600
77,300 77,300

(W-7) Land and Building

b/d 85,200 Depreciation 4,500


Cash 3,000
Revaluation (Balancing figure) 5,000
c/d 88,700
93,200 93,200

(W-8) Bank Loan

b/d 22,500
Cash paid (7,600 – 2,800) (W-4) 4,800 Loan obtained (bal) 2,600
c/d 20,300
25,100 25,100
Disposal

Plant and Machine 5,200 5,700


Gain 500
5,700 25,100
Intangibles

b/d 8,000 Amortization (bal) 1,000


c/d 7,000
8,000 8,000

Page 26 of 72
128
Ans.7 Quality Enterprises:
Statement of Cash Flow
For the year ended 31-12-2015
Cash Flow from Operating Activities:
Profit before tax 7,659,000
Investment income (398,000)
Interest expense 100,000
Depreciation 5,280,000
Bad debts (200 – 150) 50,000
Profit on disposal (959,000)
Profit before working capital changes 11,732,000

Working Capital Charges:


Inventories (3,073 – 4,642) (1,569,000)
Debtors 1,558,000
Trade and other payables (1,716,000)
Cash generated from operations 10,005,000
Tax paid (1,070,000)
Net cash from operating activities 8,935,000
Cash Flow from Investing Activities:
Purchase of PPE (16,106,000)
Sale of PPE (3,440 + 2,599) 6,039,000
Investment income received 382,000
Purchase of investment (7,645 – 6,498) (1,147,000)
Net cash flow from investing activities (10,832,000)
Cash Flow from Financing Activities:
Receipt from bank loan 6,000,000
Additional capital 3,516,000
Drawings (3,600,000)
Net cash flow from financing activities 5,916,000
Net cash flow 4,019,000
Opening balance (23,000)
Closing balance 3,996,000

Workings:
Components of Cash and Cash Equivalents:

2015 2014
Cash and bank 3,996,000 4,000
Bank overdraft (27,000)
3,996,000 (23,000)

Page 27 of 72
129
PPE

b/d 11,845 Depreciation 5,280


R.S 2,037 Disposal 2,481
Cash 16,106 Disposal 2,599
c/d 19,628

R.S

R.E (balance) 1,272 b/d 1,911


PPE 2,037
c/d 2,676

R. Earnings

Drawing 3,600 b/d 6,697


Profit after tax 6,283
R.Surplus 1,272
c/d 10,652

Capital

b/d 10,703
Cash 3,516
c/d 14,219

Trade Debtors

b/d (3,865 – 80 + 550) 4,335 Bad debts 200


Decrease 1,558
c/d [2,273-96+400] 2,577

Provision

b/d 550
Bad debts 150
c/d 400

Interest Receivable

b/d 80
Income 398 Cash 382
c/d 96

Page 28 of 72
130
Trade payable

Decrease 1,716 b/d 4,953


c/d (3,337 – 100) 3,237

Interest Payable

b/d --
Expense 100
c/d 100

Tax Payable

b/d 994
Cash 1,070 Exp. 1,376
c/d 1,300

Disposal

PPE 2,481 Cash 3,440


Gain 959

Disposal

PPE 2,599 Cash 2,599

A.8
Universal Limited
Statement of Cash Flow
For the year ended 30-6-2017
Cash Flow from Operating Activities:

Cash receipt from customer [273,000 × 40% + 144,034] 253,234


Receipt from maintenance services (W) 6,200
Payment to suppliers (W) (181,750)
Payment for expenses (W) (30,250)
Cash generated from operations 47,434
Interest paid (3,500)
Tax paid (13,000)
Net cash from operation activities 30,934
Cash Flow from Investing Activities:
Receipt from disposal 12,000

Page 29 of 72
131
Purchase of PPE (60,000)
Net cash from investing activities (48,000)
Cash Flow from Financing Activities:
Receipt from issue of shares 25,000
Repayment of debentures (8,000)
Net cash from financing activities 17,000
Net cash flow (66)
Cash and cash equivalents at the beginning of the period 48,000
Cash and cash equivalents at the end of the period 47,934

Workings:
Cash and Cash Equivalents:

2017 2016
Cash 47,934 48,000

PPE
b/d 120,000 Disposal 9,000
R.S 10,000 Depreciation 22,500
Cash 60,000
c/d 158,500

Stock
b/d 45,000 Cost of sales 171,750
[187,500 – 22,500 × 70%]
Purchases 184,750
c/d 58,000

Provision A/c
b/d 35,761
Expenses 766
c/d 4,340

Trade Receivables
b/d [56,000 + 3,574] 59,574 Bad debts 7,000
Sales 163,800 Cash 144,034
c/d [68,000 + 4,390] 72,340

Share Capital
b/d 150,000
Cash 25,000
c/d 175,000

R/Earnings
b/d 21,500

Page 30 of 72
132
PAT 32,934
c/d 54,434

R. Surplus
b/d --
PPE 10,000
c/d 10,000

Debentures
b/d 26,000
Cash 8,000
c/d 18,000

Interest Payable
b/d 2,500
Cash 3,500 Expenses 2,000
c/d 1,000

Trade Payable
b/d 39,000
Cash 181,750 Purchases 184,750
c/d 42,000

Accrued Liability
b/d 18,000
Cash 30,250 Expenses
[46,766 – 7,000 – 22,500 × 30% - 766] 32,250
c/d 20,000

Unearned maintenance Income


b/d 4,000
Income 8,200 Cash 6,200
c/d 2,000

Disposal
Cash 12,000
PPE 9,000
Gain (bal) 3000

Provision for Tax


b/d 8,000
Cash 13,000 Expenses 15,000
c/d 10,000

Page 31 of 72
133
Prayer purifies our hearts and brings mercy of Allah to us.”
A.8
Cash Flow From Operating Activities: (by using Indirect Method)

Rs. in ‘000’
Profit before tax 47,934
Depreciation 22,500
Bad debts [7,000 + 766] 7,766
Gain on disposal (3,000)
Interest expense 2,000
Profit before working capital charges: 77,200

Working Capital Changes:


Stock (13,000)
Debtors (19,766)
Trade payable 3,000
Accrued expenses 2,000
Unearned Income (2,000)
Cash Generated from Operations 47,434

Trade receivable

b/d 59,574 Bad debts 7,000


Net increase 19,766
c/d 72,340

Page 32 of 72
134
“May Allah accept our prayers and forgive our sins’”

ICAP study text


Statement of cash flow:
Introduction
Generating positive, sustainable cash flow is critical for an organisation's long-term success. Keeping track of cash
flows is particularly important for management to project the financial health of their organisation to potential
investors. Analysing the cash flow statement is extremely valuable because it provides a reconciliation of the
beginning and ending cash and cash equivalents on the balance sheet.

What are cash flows?

Cash flows are inflows and outflows of cash and cash equivalents. Cash comprises cash on hand and demand
deposits (means bank balances). Cash equivalents are short-term, highly liquid investments that are readily
convertible to known amounts of cash and which are subject to an insignificant risk of changes in value.

What Cash Flow does not indicate?


Cash is one of the major lubricants of business activity, but there are certain things that are not reflected in cash
flows. For example, profit earned after tax during a period because profitability is composed also of things that
are not cash based. Therefore the overall financial well-being of the company is not indicated in cash flows.
Furthermore accounts receivable and accounts payable are also not reflected in the cash flow statement.

Importance of cash flow for business


Monitoring the cash flows is one of the most pressing management tasks for any business. The most common
outflow of cash includes payments of salaries and to suppliers. The inflow includes the receipt from customers
and investors.
Businesses must have sufficient cash; otherwise they cannot survive.
A business can make a loss but still survive if it has sufficient cash or access to liquidity (cash or assets that can be
quickly turned into cash and new sources of borrowing).
On the other hand, a business that is profitable cannot survive if it cannot pay its obligations when they fall due,
because it does not have enough cash or access to other sources of liquidity.
Cash flow is therefore extremely important, and it is appropriate that entities should present a statement of cash
flows as a financial statement.
The purpose of a statement of cash flows is to show what the cash flows of the entity have been. It can also be
used to make assessments of what the cash flows of the entity might be in the future. In other words, the cash
flow statement is a compressed version of the company's check book that includes a few other items that affect
cash, like the financing section, which shows how much the company spent or collected from the repurchase or
sale of shares, the amount of issuance or retirement of debt and the amount the company paid out in dividends.

Purpose of statements of cash flows


IAS 1 states that a statement of cash flows is a part of a complete set of the financial statements of an entity. It
provides information about:

the cash flows of the entity during the reporting period, and

the changes in cash and cash equivalents during the period.

Page 33 of 72
135
BENEFITS OF CASH FLOW
IAS 7: Statements of cash flows sets out the benefits of cash flow information to users of financial statements.

A statement of cash flows provides information that helps users to evaluate changes in the net assets of an entity
and in its financial structure (including its liquidity and solvency).

It provides information that helps users to assess the ability of the entity to affect the amount and timing of its
cash flows in order to adapt to changing circumstances and unexpected opportunities.

It is useful in assessing the ability of the entity to generate cash and cash equivalents.

It helps users of accounts to compare the performance of different entities because unlike profits, comparisons of
cash flows are not affected by the different accounting policies used by different entities.

Historical cash flows are often a fairly reliable indicator of the amount, timing and certainty of future cash flows.
Cash Flows & Financial Statements
When a business makes a profit of Rs. 1,000, this does not mean that it receives Rs. 1,000 more in cash than it has
spent. Profit and cash flow are different, for several reasons:
There are items of cost in the statement of comprehensive income that do not represent a cash flow. Examples
are:
 depreciation and amortisation charges; and
 The gain or loss on the disposal of non-current assets.
There are items of cash flow that do not appear in the statement of comprehensive income. Examples are:
Cash flows relating to the acquisition or disposal of investments, such as the purchase of new non-current assets,
and cash from the sale of non-current assets. (The statement of comprehensive income includes gains or losses
on the disposal of non-current assets, but this is not the same as the cash proceeds from the sale.)
Cash flows relating to financial transactions, such as obtaining cash by issuing shares or obtaining loans, the
repayment of loans and the payment of dividends to ordinary shareholders.
 Theoretically this could be done by analysing every entry in and out of the cash account(s) over the course of
a period. However, the cash account is often the busiest account in the general ledger with potentially many
thousands of entries. Documents that summarise the transactions are needed.
 These documents already exist. They are the other financial statements (statement of financial position and
statement of profit or loss and other comprehensive income).
 A lot of the numbers in cash flow statements are derived from comparing opening and closing positions of
line items in the statement of financial position. Other causes of movement can then be identified leaving the
cash double entry as a balancing figure.

Statement of cash flows


A statement of cash flows provides information about where a business obtained its cash during the financial
period, and how it made use of its cash.
A statement of cash flows groups inflows and outflows of cash under three broad headings:

cash generated from or (used in) operating activities

cash obtained from or (used in) investing activities

cash received from or paid in financing activities.
It also shows whether there was an increase or a decrease in the amount of cash held by the entity between the
beginning and the end of the period (net cash flows)

Page 34 of 72
136
“If you want success then obey Allah, remember Allah and Pray to Him.”

For the purpose of a statement of cash flows, cash and cash equivalents are treated as being the same thing. This
means that cash flows between cash and cash equivalent balances are not shown in the statement of cash flows
(For example cash debit bank credit). These components are part of the cash management of an entity rather
than part of its operating, investing and financing activities.
Cash and cash equivalents are held in order to meet short-term cash commitments, rather than for investment
purposes or other purposes.
Examples of cash equivalents are:

a bank deposit where some notice of withdrawal is required (fixed short term deposits)

short-term investments with a maturity of three months or less from the date of acquisition (e.g. government
bills).
Bank borrowings are generally considered to be financing activities. In that case they would be held outside cash
and cash equivalents and movements on the bank borrowings would be shown under financing activities as a cash
inflow if borrowing increase or as a cash outflow if borrowings fell.
Sometimes, bank overdrafts which are repayable on demand form an integral part of an entity's cash
management. In these circumstances, bank overdrafts are included as a component of cash and cash equivalents.

Extra practice questions

Q. 1 Following information pertains to Nadir Limited:

Extract from statement of profit or loss for the year ended 31 December 2017
Rs. in ‘000
Profit before taxation 8,955
Taxation (2,945)
Profit after taxation 6,010

Extract from statement of financial position as on 31 December 2017


2017 2016 2017 2016
Equity and liabilities Assets
---- Rs. in ‘000 ---- ---- Rs. in ‘000 ----
Share capital 12,400 10,000 Property plant &
Share premium 1,400 - equipment – net 21,400 15,800
Retained earnings 13,450 12,440 Current assets:
Surplus on revaluation 4,000 - Stock-in-trade 5,600 5,750
Non-current liabilities: Trade receivables – net 6,840 4,446
Long-term loans 4,100 5,000 Other receivables 2,385 800
Current liabilities: Cash & bank 2,355 3,204
Trade payables 1,900 1,400
Accruals & other payables 680 660
Tax liability 650 500
38,580 30,000 38,580 30,000

Other information:
(i) Shares issued during the year were as follows: 10% bonus shares in March 2017.
Right shares in July 2017.

Page 35 of 72
137
(ii) During the year, a plant costing Rs. 9,500,000 and having a book value of Rs. 5,200,000 was disposed of for
Rs. 4,800,000 of which Rs. 1,800,000 are still outstanding.
(iii) Depreciation for the year amounted to Rs. 7,350,000.
(iv) Financial charges for the year amounted to Rs. 1,100,000. Accrued financial charges as on 31 December
2017 amounted to Rs. 112,000 (2016: Rs. 48,000).
(v) Provision for doubtful trade receivables is maintained at 5%.

Required:
Prepare statement of cash flows for the year ended 31 December 2017, in accordance with
IAS 7 ‘Statement of Cash Flows’ using indirect method. (15)

Page 36 of 72
138
A.1
Nadir Limited
Statement of Cash Flow
For the year ended 31-12-2017
Cash flow from operating activities:
Profit before tax 8,955
Loss on disposal 400
Depreciation 7,350
Interest expense 1,100
Bad debts 126
Profit before working capital changes 17,931
Changes in working capital:
Trade payable 500
Accrues expenses (44)
Stock 150
Debtors (2,520)
Other receivables 215
Cash generated from operating activities 16,232

Interest paid (1,036)


Tax paid (2,795)
Net cash flow from operating activities 12,401

Cash flow from investing activities:

Receipt from disposal of plant (4,800 – 1,800) 3,000


Purchase of PPE (14,150)
Net cash flow from investing activities (11,150)

Cash flow from financing activities:

Receipt from 2,800


Dividend paid (4,000)
Repayment of loan (900)
Net cash flow from financing activities (2,100)

Net cash flow (12,401 – 11,150 – 2,100) (849)


Opening cash and cash equivalent 3,204
Closing cash and cash equivalents 2,355

Workings:
Components of cash and cash equivalents:
2017 2016
Cash and Bank 2,355 3,204

Page 37 of 72
139
Share Capital
b/d 10,000
Cash 1,400
Bonus share 1,000
(10,000 x 10%)
c/d 12,400

Share Premium
b/d -
Cash 1,400
c/d 1,400

Retained Earnings
Dividend 1,000 b/d 12,440
(Bonus Share)
Cash 4,000 PAT 6,010
c/d 13,450

Revaluation Surplus
b/d -
PPE 4,000
c/d 4,000

Loan
b/d 5,000
Cash 900
c/d 4,100

Trade Payable
b/d 1,400
500
c/d 1,900

Other Payable
b/d 612
(660 – 48)
Cash 44
c/d 568
(680 – 112)

Allowance
b/d 234
(4,446/95
x5
Bad debts 126

Page 38 of 72
140
c/d 360
(6,840/95 x 5)

Tax Payable
b/d 500
Cash 2,795 Tax expense 2,945
c/d 650

Debtors
b/d 4,680
(4,446/95 x 100)
2,520
c/d 7,200
(6,840/95 x 100)

PPE
b/d 15,800 Disposal 5,200
R. surplus 4,000 Depreciation 7,350
Cash 14,150
c/d 21,400

Disposal
PPE 5,200 Cash 3,000
Receivable 1,800
Loss 400

Stock
b/d 5,750
150
c/d 5,600

Other Receivables
b/d 800
Disposal Outstanding 1,800 215
c/d 2,385

Interest Payable
b/d 48
Cash 1,036 Expense 1,100
c/d 112

Page 39 of 72
141
“Prayer is the key of success. Prayer will work for us If we obey Allah.”

Q.2 Junior Accountant of Drum Limited has prepared the following statement of cash flows for the year ended 31
December 2018:

Statement of cash flows


Rs. in '000
Cash flows from operating activities
Increase in retained earnings 1,360
Increase in dividend payable 200
Increase in net trade receivables (100)
Increase in interest accrued 50
1,510
Cash flows from investing activities
Increase in land and building (2,600)
Increase in equipment (1,550)
Decrease in inventory 400
Decrease in tax payable (60)
(3,810)
Cash flows from financing activities
Increase in share capital and premium 2,350
Decrease in long term loan (1,000)
Increase in trade and other payables 600
1,950
Decrease in cash balance during the year (350)
Opening cash balance 450
Closing cash balance 100

Junior Accountant informed you that he has taken the difference of opening and closing balances of each balance
sheet item and classified each difference as either operating, investing or financing cash flows. He further
informed that the statement is tied up with the cash balances appearing in the balance sheet. He has ignored the
following information:

(i) Depreciation on building and equipment amounted to Rs. 480,000 and Rs. 810,000 respectively.
(ii) During the year, an equipment costing Rs. 560,000 and having a book value of Rs. 310,000 was sold for Rs.
440,000.
(iii) Provision for doubtful debts was increased by Rs. 140,000.
(iv) Dividend amounting to Rs. 700,000 was paid during the year.
(v) Interest and tax expenses for the year amounted to Rs. 378,000 and Rs. 650,000 respectively.
(vi) Trade and other payables as at 31 December 2018 included Rs. 950,000 for purchase of land and building.

Required:
Prepare statement of cash flows for the year ended 31 December 2018, in accordance with IAS 7 ‘Statement of
Cash Flows’ using indirect method. (14)

Page 40 of 72
142
“Prayer cures a sick soul, heals broken hearts and stops you doing wrong.”

Ans. 2 Drum Limited


Statement of cash flows for the year ended 31 December 2018
Rs. in ‘000
Cash flows from operating activities
Profit before tax 1,360+700+200+650 2,910
Adjustment for:
Depreciation 480+810 1,290
Gain on disposal (130)
Increase in provision of doubtful debts 140
Interest expense 378
4,588
Working capital change
Decrease in inventory 400
Increase in trade receivables –100–140 (240)
Decrease in trade payable 600–950 (350)
(190)
Cash generated from operations 4,398
Interest paid –378+50 (328)
Tax paid –650–60 (710)
3,360

Cash flows from investing activities


Purchase of land and building –2,600–480+950 (2,130)
Purchase of equipment –1,550–810–310 (2,670)
Disposal of equipment 440
(4,360)

Cash flows from financing activities


Issuance of shares 2,350
Loan repaid (1,000)
Dividend paid (700)
650
Decrease in cash during the year (350)
Opening cash 450
Closing cash 100

Page 41 of 72
143
Equipment WDV
b/d - Disposal 310
Cash 2,670 Dep 810
c/d 1,550

Land & Building WDV


b/d - Dep 480
Payable 950
2,130 c/d 2,600
Dividend Payable
Cash 700 b/d -
c/d 200 Dividend 900

Payable
350 b/d -
c/d 600 Building 950

Retained earnings
Dividend 900 b/d -
c/d 1,360 PAT 2260

Debtor – Net
b/d -
c/d 100

Debtor – Gross
b/d -
240 c/d 240

Allowance
b/d -
c/d 140 140

Interest payable
328 b/d -
c/d 50 Exp 378

Tax payable
710 b/d 60
c/d - Exp 650

Inventories
b/d 400 400
c/d -

Page 42 of 72
144
S.C & S.P
b/d -
c/d 2,350 Cash 2,350

Loan
1,000 b/d 1,000
c/d -

PBT 2,910
Tax 650
PAT 2,260

Further Practice:

1 TRANGO LIMITED
The following information has been extracted from the financial statements of Trango Limited for the year ended
31 December 2015.

Statement of comprehensive income for the year ended 31 December 2015

Rs.
Sales 905,000
Cost of sales (311,000)
Gross profit 594,000
Loss on disposal of non-current asset (9,000)
Wages and salaries (266,000)
Other expenses (including depreciation Rs.46,000) (193,000)
126,000
Interest charges
(24,000)
Profit before tax 102,000
Tax on profit (38,000)
Profit after tax 64,000

The asset disposed of had a carrying amount of Rs. 31,000 at the time of the sale.

Extracts from the statements of financial position:

At At
1 January 2015 31 December 2015

Rs. Rs.
Trade receivables 157,000 173,000
Inventory 42,000 38,000
Trade payables 43,600 35,700
Accrued wages and salaries 4,000 4,600
Accrued interest charges 11,200 10,000
Tax payable 45,000 41,000

Page 43 of 72
145
Required
Present the cash flows from operating activities as they would be presented in a statement of cash flows:
a) Using the direct method
b) Using the indirect method.

2 MR. MOOSANI
The comparative statements of financial position of Mr. Moosani show the following information:

December 31
2015 2014
Rs. Rs.
Cash 5,200 41,400
Accounts receivable 31,700 21,500
Inventory 25,000 19,400
Investments - 16,900
Furniture 80,000 64,000
Equipment 86,000 43,000
Total 227,900 206,200
Allowance for doubtful accounts 6,500 9,700
Accumulated depreciation on equipment 24,000 18,000
Accumulated depreciation on furniture 8,000 15,000
Trade creditors 10,800 6,500
Accrued expenses 4,300 10,800
Bills payable 6,500 8,600
Long-term loans 31,800 53,800
Capital 136,000 83,800
Total 227,900 206,200

Additional data related to 2015 is as follows:


 Equipment that had cost Rs. 23,000 and was 40% depreciated at the time of disposal was sold for Rs. 6,500.
 Payments against long-term loans amounted to Rs. 22,000 of which Rs. 12,000 was paid by Mr. Moosani out
of his personal account.
 On January 1, 2015, the furniture was completely destroyed by a fire. Proceeds received from the insurance
company amounted to Rs. 60,000.
 Investments were sold at Rs. 7,500 above their cost.
 Mr. Moosani withdraws Rs. 15,000 each month for his personal use.

Required:
Prepare a statement of cash flows for the year ended 31 December 2015. (12).

Page 44 of 72
146
“Focus on prayer and leave worry because prayers changes the situation.”

3 SAKHAWAT HUSSAIN
The statements of financial position of Sakhawat Hussain as at December 31, 2015 and 2014 are as follows:

2015 2014
Rs. Rs.
Current assets 4,750,000 2,850,000
Investments 2,600,000 2,500,000
Non-current assets 9,750,000 9,600,000
Accumulated depreciation (2,950,000) (2,450,000)
14,150,000 12,500,000

Non-current liability (loan) 2,000,000 2,000,000

Current liabilities 1,850,000 1,450,000


Interest liability 200,000 150,000
Capital 9,000,000 8,000,000
Profit and loss account 1,100,000 900,000
14,150,000 12,500,000

Other information for the year 2015 is as follows:


 Investments costing Rs. 250,000 were sold for Rs. 320,000.
 Fully depreciated furniture costing Rs. 200,000 was written-off.
 Non-current assets costing Rs. 960,000 with a net book value of Rs. 160,000 were sold for Rs. 250,000.
 Interest amounting to Rs. 180,000 was paid during the year.
 Sakhawat Hussain withdrew Rs. 1,200,000 from the profits of 2014 and 2015.
 20% of the opening and closing balances of current assets are represented by cash.

Required:
Prepare a statement of cash flows for the year ended December 31, 2015. (11)

4. MR JUNAID JANJUA

Mr Junaid Janjua has provided you the following statements of financial position and statement of comprehensive
income.

Statement of financial position as on December 31, 2015

2015 2014
Rupees Rupees
Cash 145,000 32,000
Accounts receivable 280,000 104,000
Long-term investments 220,000 170,000
Inventory 424,000 200,000
Prepaid insurance 24,000 36,000
Office supplies 14,000 7,000

Page 45 of 72
147
Land 1,810,000 2,500,000
Building 2,800,000 2,300,000
Accumulated depreciation (890,000) (720,000)
Equipment 1,200,000 1,150,000
Accumulated depreciation (380,000) (350,000)
Total assets 5,647,000 5,429,000

Accounts payable 158,000 263,000


Wages payable 40,000 24,000
Short-term loans 580,000 580,000
Long-term loans 985,000 1,160,000
Capital 3,884,000 3,402,000
Total liabilities and equity 5,647,000 5,429,000

Statement of comprehensive income for the year ended December 31, 2015

2015
Rupees
Sales revenue 9,280,000

Cost of goods sold (6,199,000)

Gross profit 3,081,000

Operating expenses Selling expenses 634,000

Administrative expenses 1,348,000

Depreciation expenses 230,000

Income from operations 869,000

Other revenues/expenses Gain on sale of land 64,000

Gain on sale of long term investment 32,000

Loss on sale of equipment (15,000)

Net income 950,000

Drawings (568,000)

Retained earnings 382,000

Notes:
 Part of the long term loan amounting to Rs. 100,000 was paid by Mr. Junaid from his personal account.
 Long term investments costing Rs. 100,000 were sold during the year.
 Depreciation charged during the year on equipment amounted to Rs. 60,000. Equipment having a book value
of Rs. 75,000 was sold during the year.

Page 46 of 72
148
“May Allah make among us who do not leave prayers.”

Required:
Prepare a statement of cash flows for the year ended December 31, 2015. (14)

5. THE SINDH ROBOTICS COMPANY (direct method question)

The statements of financial position and statement of comprehensive incomes of The Sindh Robotics Company for
two consecutive financial years are shown below.

Statements of financial position

31 December 2013 31 December 2014

Cost Dep’n Net Cost Dep’n Net


Rs.000 Rs.000 Rs.000 Rs.000 Rs.000 Rs.000
Non-current assets
43,000 43,000 63,000 63,000
Land
Buildings 50,000 10,000 40,000 90,000 11,000 79,000
Plant 10,000 4,000 6,000 11,000 5,000 6,000
103,000 14,000 89,000 164,000 16,000 148,000
Investments
50,000 80,000

Current assets
Inventories 55,000 65,000

Trade receivables 40,000 50,000


Bank 3,000 -
98,000 115,000
237,000 343,000

Capital
40,000 50,000
Issued shares of Rs. 1 each

Share premium 12,000 14,000


Revaluation surplus - land - 20,000
Accumulated profit 25,000 77,000 25,000 109,000

Non-current liabilities
100,000 150,000
10% loan borrowings
Current liabilities
40,000 60,000
Trade payables
Dividend payable 20,000 20,000

Bank overdraft - 60,000 4,000 84,000

237,000 343,000

Page 47 of 72
149
Statements of comprehensive incomes
2013 2014
Rs.000 Rs.000
Revenue 200,000 200,000
Cost of sales (100,000) (120,000)
Gross profit 100,000 80,000
Distribution and administration expenses (50,000) (47,000)
50,000 33,000
Interest (10,000) (13,000)
Net profit for year 40,000 20,000

Only one dividend is declared each year which is paid in the following year. No sales of non-current assets have
occurred during the relevant period. Ignore taxation.

Required:
Prepare a statement of cash flows for the year ended 31 December 2014 using the direct method.

6. NARDONE LIMITED
The following information has been extracted from the draft financial information of Nardone Limited.

Statement of comprehensive income for the year ended 31 December 2015

Rs (000) Rs. (000)


Sales revenue 490
Administration costs (86)
Distribution costs (78) (164)
Operating profit 326
Interest expense (23)
Profit before tax 303
Taxation (87)
Profit after tax 216
Dividends paid (52)
Retained profit for the year 164

Statements of financial position

31 December 2015 31 December 2014


Rs.000 Rs.000 Rs.000 Rs.000
Non-current assets (see below) 1,145 957
Current assets:
Inventory 19 16
Receivables 38 29
Bank 19 32
76 77

Page 48 of 72
150
Total assets 1,221 1,034

Share capital 323 232


Revaluation reserve 170 0
Retained earnings 553 389
1,046 621
Non-current liabilities:
Long-term loans 70 320
Current liabilities:
Trade payables 12 17
Tax payable 93 76
105 93
Total equity and liabilities 1,221 1,034

Note on non-current assets

Land and Fixtures


Machinery Total
buildings &fittings
Cost or valuation Rs.000 Rs.000 Rs.000 Rs.000
At 31 December 2014 830 470 197 1,497
Additions - 43 55 98
Disposals - (18) - (18)
Adjustment on revaluation 70 - - 70
At 31 December 2015 900 495 252 1,647
Acc. Depreciation as on
31.12.2014 (90) (270) (180) (540)
At 31 December 2014
Charge for the year (10) (56) (8) (74)
Disposals - 12 - 12
Adjustment on revaluation 100 - - 100
At 31 December 2015 0 (314) (188) (502)
Carrying amount as on
31.12.2014 740 200 17 957
At 31 December 2014
At 31 December 2015 900 181 64 1,145

You have been informed that included within distribution costs is Rs.4,000 relating to the loss on a disposal of a
non-current asset.

Required: Prepare a statement of cash flows for Nardone Limited for the year ended 31 December 2015.
(20)

Page 49 of 72
151
7. HOT SAUCE LIMITED

Hot Sauce Limited summarised final accounts are as follows

Statements of financial position


31 December 2014 31 December 2015
Rs.000 Rs.000 Rs.000 Rs.000
Non-current assets:
Plant and machinery at cost 2,700 3,831
Accumulated depreciation (748) (1,125)
Carrying amount 1,952 2,706
Current assets:
Inventory 203 843
Receivables 147 184
Bank 51 -
401 1,027
Total assets 2,353 3,733
Ordinary share capital (Rs1 740 940
shares)
Share premium account 0 100
Retained earnings 671 1,034
1,411 2,074
Non-current liabilities:
Loans 320 150
Current liabilities:
Bank overdraft 0 766
Trade payables and Accruals 152 141
Current taxation 470 602
622 1,509
Total equity and liabilities 2,353 3,733

Statement of comprehensive income for year ended 31 December 2015

R.s 000
Profit before tax 1,195
Taxation (602)
Profit after tax 593

Dividend payments during the year were Rs.230, 000.

The following information is also available:


 The only new loan raised during the year was a five-year bank loan amounting to Rs.65, 000.
 Interest charged during the year was Rs.156, 000. Interest accrued was Rs.24, 000 last year and Rs.54, 000
this year.
 Depreciation charged during the year amounted to Rs.401, 000.
 During the year plant which originally cost Rs.69, 000 was disposed of for Rs.41, 000.

Page 50 of 72
152
 During the year the company issued 200,000 new shares.

Required
Prepare a statement of cash flows. (20)

Note: if an accounting head includes more than one items for which treatment is different then prepare separate
workings.

8. QUETTA TRACK LIMITED


The financial statements of Quetta Track Limited, a limited liability company, at 30 June were as follows.

2015 2014
Rs.000 Rs.000 Rs.000 Rs.000
ASSETS
Non-current assets Property
cost 22,000 12,000

Depreciation (4,000) 18,000 (1,000) 11,000

Plant and equipment Cost 5,000 5,000

Depreciation (2,250) 2,750 (2,000) 3,000

20,750 14,000
Current assets
Inventories 16,000 11,000
Trade receivables 9,950 2,700

Cash and cash equivalents — 1,300

25,950 15,000
Total assets 46,700 29,000

Rs.000 Rs.000 Rs.000 Rs.000

EQUITY AND LIABILITIES


Capital and reserves Equity
capital 3,000 3,000
Accumulated profits 16,200 3,800
19,200 6,800
Non-current liabilities
Loan 6,000 10,000
Current liabilities
Operating overdraft* 11,000 -
Trade payables 8,000 11,000
Income tax payable 1,800 1,000
Accrued interest 700 200
21,500 12,200

Total equity and liabilities 46,700 29,000

Page 51 of 72
153
*another name of bank overdraft

Statement of comprehensive income (extracts)

2015 2014
Rs.000 Rs.000
Operating profit 15,400 5,900
Financing cost (Interest) (1,000) (1,400)
Profit before tax 14,400 4,500
Income tax expense (2,000) (1,500)
Net profit for the year 12,400 3,000

Equipment of carrying amount Rs.250, 000 was sold at the beginning of 2015 for Rs.350, 000. This equipment had
originally cost Rs.1, 000, 000.
In recent years, no dividends have been paid.

Required:
Prepare a statement of cash flows, under the indirect method, for the year ended 30 June 2015.

9. TARBELA TRADERS
Tarbela Traders is the trading name of a sole trader.

The statements of financial position of Tarbela Traders at the end of two consecutive financial years were:

Statements of financial position at

31 December 2015 31 December 2014


Rs.000 Rs.000
Non-current assets (at WDV)
Premises 37,000 38,000
Equipment 45,800 17,600
Motor vehicles 18,930 4,080
101,730 59,680
Investments 25,000 17,000
126,730 76,680
Current assets
Inventories 19,670 27,500
Trade receivables and prepayments 11,960 14,410
Short-term investments 4,800 3,600
Cash and bank balances 700 1,800
37,130 47,310

Total assets 163,860 123,990

Capital and reserves


Opening capital 75,040 67,940
Capital introduced/(withdrawn) (6,500) 4,000

Page 52 of 72
154
Profit/(loss) for year 25,200 15,300
Drawings (15,130) (12,200)
Closing capital 78,610 75,040
Non-current liabilities
Interest-bearing borrowings 25,000 28,000
Current liabilities
Trade payables and accrued expenses 32,050 20,950
Bank overdraft 28,200 —
60,250 20,950

Total equity and liabilities 163,860 123,990

Profit for the year ended 31 December 2015 (Rs.25,200,000) is after accounting for

Rs.000
Depreciation:
Premises 1,000
Equipment 3,000
Motor vehicles 3,000
Profit on disposal of equipment 430
Loss on disposal of motor vehicle 740
Interest expense 3,000

The written down value of the assets at date of disposal was:


Rs.000
Equipment 5,200
Motor vehicles 2,010
Interest accrued at 31 December 2015 is Rs. 400,000.

Required:
Prepare a statement of cash flows for the year ended 31 December 2015. Assume that short-term investments
are cash equivalents. (14)

10. AMIN INDUSTRIES

The statements of financial position of Amin Industries as at 31 August 2014 and 2015 are as follows:

2015 2014 2015 2014


Rs. Rs. Rs. Rs.

Noncurrent assets
Capital 33,433,000 27,942,000 15,172,000 12,346,000
At book value

Current liabilities Current assets


Short term finance 2,545,000 1,616,000 Investments 4,911,000
Creditors 3,457,000 2,850,000 Inventory 12,178,000 14,950,000
Trade debts - net
6,002,000 4,466,000 of provision for 6,732,000 4,887,000
bad debts
Bank 442,000 225,000

Page 53 of 72
155
24,263,000 20,062,000
39,435,000 32,408,000 39,435,000 32,408,000

The following information is also available:

Rupees
Profit during the year ended 31 August 2015 3,161,000
Mr. Amin’s withdrawals during the year 3,120,000
Accumulated depreciation on non-current assets - 31 August 2014, 5,605,000
Accumulated depreciation on non-current assets - 31 August 2015 7,470,000
Provision for bad debts - 31 August 2014 385,000
Provision for bad debts - 31 August 2015 484,000

During the year non-current assets costing Rs. 1,500,000 with a carrying amount of Rs. 867,000 were sold for Rs.
1,284,000.

Required:
Prepare a statement of cash flows for the year ended 31 August 2015. Show necessary workings.

Note: If information is available then always prepare separate ledger of cost and accumulated depreciation.
Similarly prepare separate ledger of debtors without deducting the provision and separate ledger of provision for
bad debts. (13)

11. ABIDA
Abida made a net profit of Rs. 256,800 for the year ended June 30, 2015 after charging depreciation of Rs. 17,500
and loss on disposal of furniture of Rs. 6,800. The sale proceeds of the furniture were Rs. 12,000.

During the year, the net book value of non-current assets decreased by Rs. 7,400; receivables increased by Rs.
11,700; inventories decreased by Rs. 21,600 and creditors increased by Rs. 8,900. A long-term loan of Rs. 75,000
was repaid during the year and Abida withdrew Rs. 120,000 for his own use.

Required:
Prepare the statement of cash flows for the year ended June 30, 2015. (07)

Answers:

1 TRANGO LIMITED

Statement of cash flows: direct method Rs.


Cash flows from operating activities
Cash receipts from customers 889,000
Cash payments to suppliers (314,900)
Cash payments to employees (265,400)
Cash paid for other operating expenses (193 -46) (147,000)
Cash generated from operations 161,700
Taxation paid (42,000)
Interest charges paid (25,200)

Page 54 of 72
156
Net cash flow from operating activities 94,500

Statement of cash flows format: indirect method Rs.


Cash flows from operating activities
Profit before taxation 102,000
Adjustments for:
Depreciation and amortisation charges 46,000
Loss on disposal of non-current asset 9,000
Interest charge 24,000

181,000
Increase in receivables (173,000 - 157,000) (16,000)
Decrease in inventories (42,000 - 38,000) 4,000
Decrease in trade payables
(43,600 + 4,000) - (35,700 + 4,600) (7,300)
Cash generated from operations 161,700
Taxation paid (42,000)
Interest charges paid (25,200)
Net cash flow from operating activities 94,500

Workings: in 000

Debtor account

Balance b/d 157 Cash (bal) 889


Sale 905 Balance c/d 173
1,062 1,062

Inventory account

Balance b/d 42 Cost of sales 311


Purchase (bal) 307 Balance c/d 38
349 349

Trade payables account

Cash (bal) 314.9 Balance b/d 43.6


Balance c/d 35.7 Purchase 307
350.6 350.6

Accrued wages account

Cash (bal) 265.4 Balance b/d 4


Balance c/d 4.6 Expense 266

Page 55 of 72
157
Interest payable account

Cash (bal) 25.2 Balance b/d 11.2


Balance c/d 10.0 Expense 24.0
35.2 35.2

Tax payable account

Cash (bal) 42.0 Balance b/d 45.0


Balance c/d 41.0 Expense 38.0
83.0 83.0

2.

Mr. Moosani
Statement of Cash Flow
For the year ended December, 31, 2015
Cash Flow From Operating Activities:

Rs.
Profit before tax (W-1) 220,200
Bad debts (3,200)
Depreciation (W-2) 23,200
Profit on sale of investment (7,500)
Loss on disposal – Equipment (W-3) 7,300
Gain on disposal – Furniture (W-4) (11,000)
Profit before working capital changes 229,000

Increase / Decrease:
Accounts receivable (10,200)
Inventories (5,600)
Creditors 4,300
Accrued Expenses (6,500)
Bills Payable (2,100)
Cash generated from Operations 208,900

Cash Flow from Investing Activities:


Proceeds from sale of equipment 6,500
Purchase of equipment (66,000)
Proceeds from insurance claim 60,000

Page 56 of 72
158
Purchase of furniture (W-3) (80,000)
Proceeds from sale of investments (16,900 + 7,500) 24,400
Net Cash from Investing Activities (55,100)
Cash Flow from Financing Activities:
Repayment of loan (10,000)
Drawings (15,000 × 12) (180,000)
Net Cash from Financing Activities (190,000)
Net Cash Flow (36,200)
Cash & Cash equivalents at the beginning of the period 41,400
Cash & Cash equivalents at the end of the period 5,200

WORKINGS:
Capital Account

Rs. Rs.
Drawings (15,000 × 12) 180,000 b/d 83,800
Loan 12,000
c/d 136,000 Profit (bal) 220,200

Accumulated Depreciation Account

Rs. Rs.
Disposal 9,200 b/d 18,000
Depreciation (bal) 15,200
c/d 24,000

Furniture Account

Rs. Rs.
b/d 64,000 Disposal 64,000
Cash (bal) 80,000
c/d 80,000

Accumulated Depreciation Account


Rs. Rs.
Disposal 15,000 b/d 15,000
Depreciation(bal) 8,000
c/d 8,000
Disposal Account – Equipment
Rs. Rs.
Equipment 23,000 Accumulated Depreciation 9,200
Cash 6,500

Page 57 of 72
159
Loss(bal) 7,300
Disposal Account – Furniture
Rs. Rs.
Furniture 64,000 Accumulated Depreciation 15,000
Cash 60,000
Gain 11,000

Equipment Account
Rs. Rs.
b/d 43,000 Disposal 23,000
Cash 66,000
c/d 86,000
Provision for bad debts
Rs. Rs.
Bad debts (bal) 3,200 b/d 9,700
c/d 6,500

3.

Sakhawat Hussain
Statement of Cash Flow
For the year ended December, 31, 2015

Cash Flow from Operating Activities:


Profit before tax 1,400,000
Depreciation 1,500,000
Gain on disposal – Investment (70,000)
Gain on disposal – Fixed Assets (90,000)
Interest Expense 230,000
Profit before working capital changes 2,970,000
Increase / Decrease:
Increase in current assets (1,520,000)
Increase in current liabilities 400,000
Cash generated from operations 1,850,000
Interest paid (180,000)
Net cash from operating activities 1,670,000

Cash Flow from Investing Activities:

Page 58 of 72
160
Sale proceeds of fixed assets 250,000
Purchase of fixed assets (1,310,000)
Sale proceeds of investment 320,000
Purchase of investments (350,000)
Net Cash from Investing Activities (1,090,000)

Cash Flow from Financing Activities:


Proceeds from capital 1,000,000
Drawings (1,200,000)
(200,000)
Net Cash Flow 380,000
20% Opening balance of cash & cash equivalents 570,000
20% Closing balance of cash & cash equivalents 950,000

WORKINGS:
Accumulated Profits

Rs. Rs.
Drawings 1,200,000 b/d 900,000
Profit after tax 1,400,000
c/d 1,100,000

Fixed Assets

Rs. Rs.
b/d 9,600,000 Disposal 200,000
Disposal 960,000
Cash (bal) 1,310,000
c/d 9,750,000

Accumulated Depreciation

Rs. Rs.
Disposal 200,000 b/d 2,450,000
Disposal 800,000 Depreciation (bal) 1,500,000
c/d 2,950,000

Disposal Account

Rs. Rs.
Fixed Asset 200,000 Accumulated Depreciation 200,000
-- --

Disposal Account

Page 59 of 72
161
Rs. Rs.
Fixed Asset 960,000 Accumulated Depreciation 800,000
Cash 250,000
Gain 90,000

Capital Account

Rs. Rs.
b/d 10,000,000
Bank (bal) 1,000,000
c/d 1,100,000

Investment Account

Rs. Rs.
b/d 2,500,000 Disposal 250,000
Cash (bal) 350,000
c/d 2,600,000

Increase in Current Assets:

Current Assets of 2009 excluding cash 3,800,000 (4,750,000 × 80%)


Current Assets of 2008 excluding cash 2,280,000 (2,850,000 × 80%)
Difference (increase) 1,520,000

4.

Junaid Janjua
Statement of Cash Flow
For the year ended December, 31, 2015
Cash Flow from Operating Activities:

Rs.
Profit before tax 950,000
Depreciation (170,000 + 60,000) 230,000
Gain on sale of land (64,000)
Gain on sale of long term investment (32,000)
Loss on sale of equipment 15,000
Profit before working capital changes 1,099,000

Page 60 of 72
162
“Sincere prayer and faith in Allah brings light in the heart.”

Working Capital Changes:


Accounts receivable (176,000)
Inventory (224,000)
Prepaid Insurance 12,000
Office Supplies (7,000)
Accounts payable (105,000)
Wages payable 16,000 (484,000)
Cash Generated from Operations 615,000

Cash Flow from Investing Activities:


Purchase of long term investment (150,000)
Sale of land (754,000)
Sale of Equipment 60,000
Sale of investments 132,000
Purchase of Building (500,000)
Purchase of Equipment (155,000)
Net Cash from Investing Activities 141,000

Cash Flow from Financing Activities:


Repayment of Loan (75,000)
Drawings made (568,000) (643,000)
Net Cash Flow 113,000
Cash & Cash equipment at the beginning of the period 32,000
Cash & Cash equivalents at the end of the period 145,000

WORKINGS:
Long Term Loan

Rs. Rs.
Capital 100 b/d 1,160
Cash (bal) 75
c/d 985

Investment

Rs. Rs.
b/d 170 Disposal 100
Cash(bal) 150

Page 61 of 72
163
c/d 220

Land

Rs. Rs.
b/d 2,500 Disposal 690
c/d 1,810
Disposal - Land
Rs. Rs.
Land 690 Cash (bal) 754
Gain 64
Building
Rs. Rs.
b/d 2,300
Cash (bal) 500
c/d 2,800
Accumulated Depreciation
Rs. Rs.
b/d 720
Depreciation(bal) 170
c/d 890
Capital Account
Rs. Rs.
Drawings 568 b/d 3,402
Profit 950
Loan 100
c/d 3,884

Equipment Account
Rs. Rs.
b/d 1,150 Disposal 105
Cash(bal) 155
c/d 1,200
Accumulated Depreciation
Rs. Rs.
Disposal 30 b/d 350
Depreciation (bal) 60
c/d 380

Page 62 of 72
164
Disposal – Equipment
Rs. Rs.
Equipment 105 Cash 60
Accumulated Depreciation 30
Loss (bal) 15
5.

THE SINDH ROBOTICS COMPANY


The Sindh Robotics Company: Statement of cash flows for the year ended 31 December 2014
Rs.000 Rs.000

Cash flows from operating activities


Cash receipts from customers 190,000
Cash paid to suppliers (108,000)
Cash paid for admin and distribution (47,000)
Cash generated from operations 35,000
Interest paid (13,000)
Dividends paid* (20,000)
Net cash from operating activities 2,000
Cash flows from investing activities
Purchase of property and plant (40,000 + 1,000) (41,000)
Purchase of investments (30,000)
Net cash used in investing activities (71,000)
Cash flows from financing activities
Proceeds from issued shares (10,000 + 2,000) 12,000
Proceeds from long-term borrowings 50,000
Net cash from financing activities 62,000
Net decrease in cash and cash equivalents (7,000)
Cash and cash equivalents at 1 January 2014 3,000
Cash and cash equivalents at 31 December 2014 (4,000)

* Could be shown as a financing cash flow.

Workings

Cash and cash equivalents 2015 2014


Bank - 3,000
Bank overdraft (4,000) -
(4,000) 3,000

Retained earnings
Rs.000 Rs.000
Dividend 20,000 Balance b/d 25,000
Balance c/d 25,000 Profit after tax 20,000
45,000 45,000

Page 63 of 72
165
Receivables control
Rs.000 Rs.000
Balance b/d 40,000 Cash receipts (pal fig) 190,000
Sales 200,000 Balance c/d 50,000
240,000 240,000

Payables Account
Rs.000 Rs.000
Cash paid (pal fig) 108,000 Balance b/d 40,000
Purchases 128,000
Balance c/d 60,000
168,000 168,000

Inventory
Rs.000 Rs.000
Opening inventory 55,000 Cost of sales (120 - 2) 118,000
Purchases 128,000 Closing inventory 65,000
183,000 183,000

6.
NARDONE LIMITED
Nardone Limited: Statement of cash flows for the year ended 31 December 2015

Rs.000 Rs.000
Cash flows from operating activities
Profit before taxation 303
Adjustments for:
Depreciation 74
Interest charges in the statement of comprehensive 23
Income
Losses on disposal of non-current assets 4
404
Increase in receivables (38 - 29) (9)
Increase in inventories (19 - 16) (3)
Decrease in trade payables (17 - 12) (5)
Cash generated from operations 387
Taxation paid (W1) (70)
Interest charges paid (23)
Net cash flow from operating activities 294
Cash flows from investing activities
Purchase of non-current assets (98)
Proceeds from sale of non-current assets (W2) 2
Net cash used in (or received from) investing Activities (96)
Cash flows from financing activities
Proceeds from issue of shares (323 - 232) 91

Page 64 of 72
166
Repayment of loans (320 - 70) (250)
Dividends paid to shareholders (52)
Net cash used in (or received from) financing activities (211)
Net increase/(decrease) in cash and cash equivalents (13)
Cash and cash equivalents at beginning of the year 32
Cash and cash equivalents at the end of the year 19
Workings
Retained earnings

dividend (bal) 52 Balance b/d 389

Balance c/d 553 Profit after tax 216


605 605

Tax payable account

Cash (bal) 70 Balance b/d 76


Balance c/d 93 Expense 87
163 163

Disposal account

Asset 18 Acc depreciation 12


cash 2
Loss 4
18 18

7.

HOT SAUCE LIMITED

Statement of cash flows for the year ended 31 December 2015

Rs.000 Rs.000
Cash flows from operating activities
Profit before taxation 1,195
Adjustments for:
Depreciation 401
Loss on sale of plant 4
Interest charges in the statement of comprehensive
Income 156
1,756
Increase in receivables (184 - 147) (37)
Increase in inventories (843 - 203) (640)
Decrease in trade payables (41)
Cash generated from operations 1,038
Taxation paid (470)
Interest charges paid (126)
Net cash flow from operating activities 442

Page 65 of 72
167
Cash flows from investing activities
Purchase of non-current assets (1,200)
Proceeds from sale of non-current assets 41
Net cash used in (or received from) investing activities (1,159)
Cash flows from financing activities
Proceeds from issue of shares (200 +100) 300
Bank loan raised 65
Repayment of loans (235)
Dividends paid to shareholders (230)
Net cash used in (or received from) financing activities (100)
Net increase/(decrease) in cash and cash equivalents (817)
Cash and cash equivalents at beginning of the year 51
Cash and cash equivalents at the end of the year (766)

Workings
Cash and cash equivalents 2015 2014
Bank - 51
Bank overdraft (766) -
(766) 51

Loan account

cash (bal) 235 Balance b/d 320


Balance c/d 150 Cash 65
385 385

Tax payable account

Cash (bal) 470 Balance b/d 470


Balance c/d 602 Expense 602
1,072 1,072

Disposal account

Asset 69 Acc depreciation 24


cash 41
Loss 4
69 69

trade payable account

difference 41 Balance b/d (152 - 24) 128


Balance c/d (141 - 54) 87
128 128
Interest payable account

Page 66 of 72
168
Cash (bal) 126 Balance b/d 24
Balance c/d 54 Expense 156
170 170
Plant and machinery account

Balance b/d 2,700 Disposal 69

Cash (bal) 1,200


Balance c/d 3,831
3,900 3,900
Accumulated depreciation account

disposal 24 Balance b/d 748


Balance c/d 1,125 depreciation 401
1,149 1,149

8.

QUETTA TRACK LIMITED


Statement of cash flows for the year ended 30 June 2015

Rs.000 Rs.000
Cash flows from operating activities
Net profit before tax 14,400
Adjustments for
Depreciation Rs.(3,000 + 1,000) 4000
Profit on sale of non-current assets (W3) (100)
Interest expense 1,000
Operating profit before working capital adjustments 19,300
Increase in inventories (5,000)
Increase in trade receivables (7,250)
Decrease in trade payables (3,000)
Cash generated from operations 4,050
Interest paid (W5) (500)
Income taxes paid (W4) (1,200)
Net cash from operating activities 2,350
Cash flows from investing activities
Purchase of property (10,000)
Purchase of plant and equipment (W1) (1,000)
Proceeds from sale of plant and equipment (W3) 350
Net cash used in investing activities (10,650)
Cash flows from financing activities
Part repayment of loan (4,000)
Net cash used in financing activities (4,000)
Net decrease in cash and cash equivalents (12,300)
Cash and cash equivalents at beginning of year 1,300

Page 67 of 72
169
Cash and cash equivalents at end of period (11,000)

Workings

Cash and cash equivalents 2015 2014


Cash - 1,300
Bank overdraft (11,000) -
(11,000) 1,300
Retained earnings

Rs.000 Rs.000
Bal b/d 3,800
Bal c/d 16,200 Profit after tax 12,400
16,200 16,200

Plant and machinery – Cost

Rs.000 Rs.000
Bal b/d 5,000 Disposal 1,000
Additions (P) 1,000 Bal c/d 5,000
6,000 6,000

Plant and machinery - Accumulated depreciation


Rs.000 Rs.000
Disposal 750 Bal b/d 2,000
Bal c/d 2,250 Depreciation charge for year (B) 1,000
3,000 3,000

Plant and machinery - Disposals


Rs.000 Rs.000
Cost 1,000 Accumulated depreciation 750
Profit on sale 100 Proceeds 350
1,100 1,100

Tax payable
Rs.000 Rs.000
Cash paid (P) 1,200 Bal b/d 1,000
Bal c/d 1,800 T ax charge to P&L 2,000
3,000 3,000
Interest payable
Rs.000 Rs.000
Cash paid (P) 500 Bal b/d 200
Bal c/d 700 Charge to P&L 1,000
1,200 1,200

Page 68 of 72
170
9.
TARBELA TRADERS
Statement of cash flows for the year ended 31 December 2015
Rs.000 Rs.000
Cash flows from operating activities
Net profit 25,200
Adjustments for:
Depreciation 7,000
Net loss on disposals (740 - 430) 310
Interest expense 3,000
Operating profit before working capital changes 35,510
Decrease in trade receivables 2,450
Decrease in inventories 7,830
Increase in trade payables 10,700
Cash generated from operations 56,490
Interest paid (2,600)
Net cash from operating activities 53,890
Cash flows from investing activities
Purchase of long-term investments (25,000 - 17,000) (8,000)
Purchase of equipment and cars (36,400 + 19,860) (56,260)
Proceeds from sale of equipment and cars (W3) 6,900
Net cash used in investing activities (57,360)
Cash flows from financing activities
Capital and other drawings (6,500 + 15,130) (21,630)
Borrowings repayment (3,000)
Net cash used in financing activities (24,630)
Net decrease in cash and cash equivalents (28,100)
Cash and cash equivalents at beginning of period 5,400
Cash and cash equivalents at end of period (22,700)
Workings

Cash and cash equivalents 2015 2014


Cash and bank 700 1,800
Bank overdraft (28,200) -
Short term investment 4,800 3,600
(22,700) 5,400
Premises (WDV)

Rs.000 Rs.000
Bal b/d 38,000 Depreciation 1,000
Bal c/d 37,000
38,000 38,000

Page 69 of 72
171
Equipment (WDV)

Rs.000 Rs.000
Bal b/d 17,600 Disposal 5,200
Depreciation 3,000
Additions (bal) 36,400 Bal c/d 45,800
54,000 54,000

Motor vehicles (WDV)

Rs.000 Rs.000
Bal b/d 4,080 Disposal 2,010
Depreciation 3,000
Additions (bal) 19,860 Bal c/d 18,930
23,940 23,940
Disposal

Rs.000 Rs.000
Equipment 5,200
Profit on disposal (equipment) 430 Proceeds (bal) 5,630
5,630 5,630

Disposal

Rs.000 Rs.000
Motor vehicle 2,010 Loss on disposal (vehicles) 740
Proceeds (P) 1,270
2,010 2,010
10.
Amin Industries
Statement of cash flows for the year ended 31 August 2015
Rs. Rs.
Cash flows from operating activities
Profit for the year 3,161,000
Adjustments for:
Depreciation charge 2,498,000
Profit on sale of non-current assets (1,284,000 - 867,000) (417,000)
bad debts (484,000 - 385,000) 99,000
Operating profit before working capital adjustments 5,341,000

Decrease in inventory 2,772,000

(Increase) in trade debts (1,944,000)


Increase in payables 607,000

Page 70 of 72
172
Net cash from operating activities 6,776,000
Cash flows from investing activities
Purchase of non-current assets (6,191,000)
Sale proceeds of non-current assets 1,284,000
Purchase of investment (4.911.000)
Net cash from investing activities (9,818,000)
Cash flows from financing activities
Capital input 5,450,000
Withdrawals by Mr. Amin (3,120,000)
Short term finance 929,000
Net cash from financing activities 3,259,000
Increase in cash and cash equivalents 217,000
Opening bank balance 225,000
Closing bank balance 442,000

Workings

Non-current assets – cost

Opening (12,346+5605) 17,951,000 disposal 1,500,000


Additions - balancing figure 6,191,000 Closing (15,172+7,470) 22,642,000

24,142,000 24,142,000

Accumulated depreciation
On assets sold (1,500-867) 633,000 Opening 5,605,000
Closing balance 7,470,000 Charge for the year 2,498,000

8,103,000 8,103,000

Trade debts

Opening (4,887+385) 5,272,000


Increase in balance 1,944,000 Closing (6,732+484) 7,216,000

7,216,000 7,216,000

Capital account - Mr. Amin

Withdrawals 3,120,000 Opening 27,942,000


Closing balance 33,433,000 Profit for the year 3,161,000
Capital introduced 5,450,000
36,553,000 36,553,000

Page 71 of 72
173
11.
ABIDA
Cash flow for year ended June 30, 2015

Cash flows from operating activities


Profit for the year 256,800
Depreciation 17,500
Loss on sale of furniture 6,800
Increase/decrease in working capital
Decrease in inventories 21,600
Increase in payables 8,900
Increase in receivables (11,700)
299,900
Cash flows from investing activities
Proceeds from sale of non-current assets 12,000
Purchase of non-current assets (W) (28,900)
(16,900)
Cash flows from financing activities
Payment of long term loan (75,000)
Drawings (120,000)
(195,000)
Net increase in bank balance
88,000
Workings:

Non-current assets

b/d 7,400 Depreciation 17,500


Purchase of assets - balancing 28,900 Disposal 18,800
figure
c/d -----

36,300 36,300
Sale proceeds 12,000
Loss on disposal 6,800
WDV 18,800

Page 72 of 72
174
“Prayer is powerful, never doubt the power of prayers.”

Ratios
Obtaining information from the financial statements: financial ratios
Financial statements are used to make decisions. They are used by shareholders, investors, lenders, as well as by
management. The financial statements contain a large number of figures, but the figures themselves do not
necessarily have much meaning to a user of the financial statements. However, the figures can be analyzed and
interpreted by calculating financial ratios.

Financial ratios can help the user of the financial statements to assess:
 the financial position of the entity, and
 its financial performance

Users of the financial statements and their information needs


The IASB Conceptual Framework identifies several groups of people who may use financial statements:
 investors and potential investors
 lenders
 employees
 suppliers
 customers
 government and government agencies
 the general public
 management

All these groups are interested in financial performance, financial position and cash flows, but some users are
mainly interested in performance and profitability, while others may be more interested in liquidity and gearing or
other matters.

For example:
 A private investor needs to know whether to continue to hold shares or to sell them. He or she will tend to be
most interested in profitability ratios (such as gross and net profit margin and return on capital employed) and
investor ratios (such as earnings per share, dividend cover and price earnings ratio).(Investor ratios are not in
the syllabus)
 A potential acquirer/investor needs information about an entity’s profitability and probably also information
about whether or not the entity is managed efficiently. The acquirer’s management is likely to focus on profit
margins, return on capital employed, asset turnover and working capital ratios.
 A bank that has been approached to lend money to an entity needs to know whether it will receive interest
payments when these are due and whether the money that it lends will eventually be repaid. A bank manager
will normally be most interested in cash flows and liquidity ratios (current ratio, acid test ratio) gearing and
interest cover. A potential lender will also be interested in predicting future performance as without sales there
will be no cash.
 Employees are interested in fair wages and bonus linked with profitability.
 Management does planning and forecasting as a decision maker.
An examination question might ask you to interpret an entity’s financial statements for the benefit of specific people
or groups of people. Therefore, your analysis should focus on the needs of the users.
 What do they need to know?
 What are they interested in?

Page 1 of 58
175
 What decision do they need to make?

Purpose and Importance of Financial Ratio Analysis


Financial ratio analysis helps a business in a number of ways. The importance and advantages of financial ratios
are given below:
 Ratios help in analysing the performance trends over a long period of time.
 They also help a business to compare the financial results to those of competitors.
 Ratios assist the management in decision making.
 They also point out problem and weak areas along with the strength areas.
 Ratios help to develop relationships between different financial statement items.
 Ratios have the advantage of controlling for differences in size. For example, two businesses may be
quite different in size but can be compared in terms of profitability,liquidity, etc., by the use of ratios.

In a question of ratio analysis; it is assumed that all accounting entries are correctly recorded in financial
statements unless otherwise mentioned.

1. Liquidity Ratios:
Liquidity means cash or access to cash readily available to meet obligations.
Sr# Particular Formula

1 Working Capital Current Asset – Current Liabilities

(Answer in Rs.)

Interpretation +Ve / -Ve shows we have Surplus / deficit funds in business to meet current liability.

2 Current Ratio Current Asset____

Current Liabilities

(Answer is in times)

The ratio indicates the ability of the company to pay its current obligation out of the current
Interpretation asset, usually 2:1 is acceptable. This ratio shows the ability of the company to meet its
obligations in ordinary course of business (i.e within next 12 months).

3  Acid test ratio; or Current assets - Inventory


 Liquid Ratio; or
Current liabilities
 Quick Ratio.
(Answer is in times)

Interpretation This ratio shows the ability of the company to settle its obligations immediately. A 1:1 ratio
is usually considered to be acceptable.

*Liquidity means having cash or access to cash readily available to meet obligations.

Page 2 of 58
176
2. Turnover / Efficiency / Working capital efficiency Ratios:

4 Inventory Turnover Cost of Goods Sold

*Average Inventory

* Opening + closing

(Answer is in times)

Interpretation The number of times stock has been converted into sale. The higher the ratio the better it is.
Ratio should be higher due to more sales not due to lower stock.

5 Inventory Turnover Period Average Inventory x 365

Cost of Goods Sold

(Answer in days)

Interpretation In how many days stock is converted into sales. The minimum the days, the better it is.

6 Debtor turnover Net Credit Sales

Avg Trade Debtors


(Answer is in times)

Interpretation Normally higher the turnover ratio the better it is because it signifies speedy and effective
collection.

7 Debtor Turnover Period Average trade Debtor x 365

(Collection Period) Net Credit Sales


(Answer in days)

Interpretation The number of days, company takes between date of sale and receipt of cash. Usually time
period should not exceed 3 month. However, evaluation is made in terms of credit policy of
company.

8 Creditor turnover

Net Credit Purchases

Avg Trade creditors


(Answer is in times)

Interpretation The final decision about this ratio should be on the basis of answer of next ratio. However, if
answer in times is more it means we are paying quickly otherwise late.

Page 3 of 58
177
9 Creditor Turnover Period Avg Creditor x 365

Credit Purchase
(Answer in days)

Interpretation In how many days, payment is made to creditors after purchase. Days should be compared with
credit period agreed.

Note: If there is no information of stocks then assume no stocks; or constant stocks which therefore
means that purchase = cost of sales

10 Business/Operating/Cash/working capital Inventory days + debtor days – Creditor days


Cycle

(Examples on page 6)

Interpretation The time period between the payment of cash to creditors and receipt of cash from debtors.

Note: If answer is to be taken in weeks, then instead of 365; take 52; if answer is to be taken in months, then take
12.

3.Profitability Ratios

11 Total Asset Turnover / net Asset Turnover Net Sales_ __

(Utilization of Total Assets) Avg. Capital Employed


(Answer is in times)

Interpretation How much time the sales are in relation to capital employed. The maximum the ratio, the better
it is.

12 Gross profit Ratio Gross Profit x 100 =…...%

Net Sales

(Answer in %)

Interpretation The higher the ratio, better it is.

13 Operating Profit Ratio Operating Profit x 100

(Operating profit is Profit before interest & Tax) Net Sales

Page 4 of 58
178
(Answer in %)

Interpretation Maximum the ratio, the better it is.

14 Expense Ratio

Cost of sales x 100 Expenses x 100

Sales Net Sales

Admin expense x 100 (Answer in %)

Sales

Selling expense x 100

Sales

Interpretation The lower the ratio, the better it is.

15 Net Profit after tax Ratio

Profit After tax x 100

Net Sales

(Answer in %)

Interpretation The higher the ratio, the better it is.

16 Return on Assets

Profit before interest & tax x 100

Avg total Assets


(Answer in %)

Interpretation Total assets include both current assets and non current assets. However sometimes return
on non current assets is also calculated.The higher the ratio, the better it is.

Page 5 of 58
179
17* Return on Capital Employed (ROCE) a) Profit before interest & tax x100
Avg Capital Employed

(Answer is in %)
Long term loans (also called long term debt)

=TFC + Debentures + Redeemable Preferences shares +


Bond + loan stock b) Capital Employed = Equity +
Long Term Loans

c) Capital employed = Total


Assets – C.L
Interpretation The ratio shows the return on total funds invested in the business. The higher the ratio, the
better it is.

18* Return on shareholders capital (Equity) or Investment PAT – *Preference Dividend x100
[ROSC]
Average Equity
Equity= (Share capital + Reserves)
(Answer is in %)

*If preference shares are non-


redeemable

Interpretation The higher the ratio, the better it is.

*Note: if the question is silent regarding the preference shares then simply ignore them.

If the question is silent regarding whether the preference shares are redeemable or irredeemable then assume
these are irredeemable.

4. Debt ratio or Long term solvency ratios


Ratios calculated by financial institutions before advancing loans to customers:

19 Gearing Ratio/Debt to Equity Ratio: Gearing also called as leverage, measures the total long
term debt of a company as a percentage of either:

i) Equity of the company; or


ii) Capital employed of the company
a) Long term loans x 100
Equity
(Answer in %)
OR

b) Long term loans______ x 100


Equity + long term loans
(Answer in %)
Interpretation If company is either above 100%

Page 6 of 58
180
[in case of (a) above] or 50%

[in case of (b) above], the company will be considered as highly geared or highly leveraged
company, otherwise low geared. If question is silent then (b) is preferable

20 Interest Cover:

Profit before interest & Tax

Interest expense

(Answer in times)

Interpretation How many times profit is available against interest expense. The higher the ratio the more
confident will be loan providers to receive their interest. An interest cover ratio of less than
3.0 times is considered very low.

Example on Page 9
In a manufacturing business there are three types of inventory:
 Raw material
 Work in process
 Finished goods

Therefore inventory turnover is divided into:


1. Raw material inventory holding period (it is the time period between purchase and consumption(usage) of
raw material)

Average raw material Inventory x 365

Raw material consumed (Answer in days)

2. Work in process inventory holding period (it is the time period between consumption(usage) of raw material
and completion of finished goods or in other words, production time of finished goods)

Average work in process Inventory x 365

Cost of Goods manufactured (Answer in days)

3. Finished goods inventory holding period (it is the time period between completion of finished goods and their
sale)

Average finished goods Inventory x 365

Cost of Goods Sold (Answer in days)

Example: Business Cycle

i) Goods purchased and payment made on 1-1-2015

Goods sold on credit on 31-1-2015


Cash received from customers on 15-2-2015
Inventory turnover period 31 days
Debtor turnover period 15 days

Page 7 of 58
181
Creditor turnover period -
Business cycle 46 days

ii) Goods purchased on 1-1-2015

Payment for goods on 20-1-2015


Goods sold on credit on 31-1-2015
Cash received from customers on 15-2-2015
Inventory turnover period 31 days
Debtor turnover period 15 days
Creditor turnover period (20) days
Business cycle 26 days
Ordinary vs. preference shares
Companies issue two main types of shares:
 Ordinary Shares
 Preference Shares
 Irredeemable Preference shares
 Redeemable Preference shares

Comparison of ordinary shares and irredeemable preference shares

Ordinary shares Irredeemable Preference shares


Dividend rate Variable – higher in a good year, Fixed per annum
lower in a bad year
Dividend distribution Paid only if there are spare funds Receives the dividend before ordinary
after the payment of preference shareholders (therefore lower risk)
dividend
Liquidation The last to be repaid in a Repaid before (in preference to) the
liquidation ordinary shareholders
Voting rights Receive the right to vote on major No right to vote on company decisions.
decisions. Each ordinary share
would attract one vote.

Dividend presentation In equity In equity (before ordinary dividend)

Amount of capital In equity In equity


presentation

Comparison of redeemable preference shares and irredeemable preference shares

Irredeemable preference shares Redeemable Preference shares


Dividend rate Fixed per annum Fixed per annum

Dividend distribution Paid only if there are spare funds after Receives the dividend before
the payment of a redeemable preference Irredeemable preference shareholders
dividend in preference to ordinary and ordinary shareholders (therefore
shareholders lower risk)

Page 8 of 58
182
Liquidation The last to be repaid in a liquidation but Repaid before (in preference to)
before ordinary shareholders. Irredeemable preference shareholders
and the ordinary shareholders
Voting rights No right to vote on company decisions no right to vote on company decisions

Dividend presentation In equity (before ordinary dividend) In financial charges in income


statement
Amount of capital In equity In non-current liabilities
presentation

Page 9 of 58
183
“Prayers take you near to Allah.”

Comparison of ordinary shares and redeemable preference shares

Ordinary shares Redeemable Preference shares


Dividend rate Variable – higher in a good year, lower Fixed per annum
in a bad year
Dividend distribution Paid only if there are spare funds after Receives the dividend before
the payment of preference dividend Irredeemable preference shareholders
and ordinary shareholders (therefore
lower risk)
Liquidation The last to be repaid in a liquidation Repaid before (in preference to)
Irredeemable preference shareholders
and the ordinary shareholders
Voting rights Receive the right to vote on major no right to vote on company decisions
decisions. Each ordinary share would
attract one vote.
Dividend presentation In equity In financial charges in income
statement
Amount of capital In equity In non-current liabilities
presentation

Example
The following information pertains to Shale Distributors Limited (SDL):
Rs. in million
Sales 300
Purchases 140
Cost of goods sold 150
Trade receivables 50
Trade payables 21
Inventories 30
All the purchases and sales are on credit.

Required:
Calculate the cash operating cycle of SDL and explain briefly its significance.
(Assume a 360-day year)

Solution

Cash operating Cycle = Inventory turnover ratio period + Trade receivable turnover period – Trade payable

= 72(W-1) + 60(W-2) – 54(W-3)

= 78 days

Page 10 of 58
184
Workings Notes

W-1

Inventory turnover period = Inventories_____ x 360

Cost of Goods Sold

= 30 x 360

150

= 72 days

W-2

Trade receivables turnover period = Trade receivables x 360

Sales

= 50 x 360

100

= 60 Days

W-3

Trade payable turnover period = Trade Payable x 360

Sales

= 21_ x 360

140

= 54 days

Significance

It shows the time between payment of materials & recovery from debtors. The organization requires financing
for this period.

Page 11 of 58
185
“No one is worth to worship except Allah so pray to Allah and none.”

Example:

You are given the following information about Company R:


At 31 December 31-12-2015
Rs.000
Total assets 5,800
Share capital 1,200
Retained earnings 2,400
3,600
Long-term liabilities (Bank loans) 1,500
Current liabilities 700
5,800

For the year to 31 December 2015

Rs.000
Profit before interest and taxation 700
Interest (230)
Profit before tax 470
Taxation (140)
Profit after taxation 330
Required
Use this data to calculate:
a) The gearing ratio at 31 December 2015.
b) The interest cover in the year ended 31-12-2015

Answer
a)
i) Gearing = 1,500 x 100 =41.7%
3,600

Alternatively:

ii) Gearing = 1,500 x 100 = 29.4%


(3,600 + 1,500)

(The company is fairly low-geared.)

b)
Interest cover = 700 = 3.04 times
230

Page 12 of 58
186
Self-Test Questions
Q.1 Indicate the effect of the following transactions on working capital by indicating:
 No effect with justification
 Increase with justification
 Decrease with justification
1) Purchase of raw material of Rs. 50,000
2) Sale of finished goods of Rs. 65,400
3) Depreciation for the year Rs. 70,000
4) Operating expenses paid of Rs. 90,000
5) Purchased equipment of Rs. 120,000 on account
6) Paid cash Rs. 150,000 on account of payable.
7) Received Rs. 20,000 on account of receivables
8) Declared & paid a cash dividend of Rs. 40,000
9) Issued Rs. 100,000 share capital for cash.
10) Borrowed Rs. 200,000 on short term loan.
11) Dividend declared
12) Dividend declared in last year paid during the year.

Q.2 Acoms is a small business with limited liability. Its summarized financial results are given below:

Acoms
Income statement for the year ended 31 May 2007
Rs.000
Revenue 375
Cost of sales (280)

Gross profit 95
Distribution & administrative expenses (45)
Profit from operations 50
Finance costs (5)
Profit before tax 45
Income tax expense (15)
Profit for the period 30

Acoms
Balance sheet as at 31 May 2007
Rs.000 Rs.000
Assets
Non-current assets 410
Current assets
Inventory 96
Trade receivables 34
Cash and bank 3 133
Total assets 543
Equity and liabilities
Capital and reserves
Rs.1 Ordinary shares 300
Retained earnings 90

Page 13 of 58
187
390
Current liabilities
Trade payables 88
Taxation 15 103
Non-current liabilities
10% Loan notes 50
Total equity and liabilities 543

Additional Information:
The following are ratios for Acoms for the year to 31 May 2006 and the industry average ratios for 2007:

Acoms Industry Average


Ratio 2006 2007
Gross profit percentage (%) 34.7 30.0
Operating profit percentage (%) 17.7 20.0
Current ratio 1.5 1.5
Acid test (Quick) ratio 1.1 1.0
Receivables collection period (days) 16.0 20.0

Required:
a) Calculate the following ratios for Acoms for the year ended 31 May 2007. State clearly the formula used for
each ratio.
i. Gross profit percentage.
ii. Operating profit percentage
iii. Current ratio
iv. Acid test (Quick) ratio
v. Receivables Collection period

b) Use the information given and the ratios you calculated in part (a) to comment on the performance of Acoms.

Q.3 Given below is the information of a limited company


Balance Sheet
Assess Rs.
Fixed Assets at W.D.V. 1,150,000
Investments 300,000
Current Assets
Stock 310,000
Sundry Debtors 350,000
Advances 100,000
Cash and bank Balance 40,000
800,000
Total Assets
2,250,000
Capital and Liabilities
Share Capital 700,000
Unappropriated Profit (Retained Earnings) 250,000

Page 14 of 58
188
950,000

Long Term Loans 725,000

Current Liabilities
Accounts Payable 125,000
Sundry Creditors 250,000
Accrued and other liabilities 200,000
575,000

2,250,000

Profit and Loss Account

Sales 1,675,000
Cost of Sales (1,000,000)
Gross Profit 675,000

Administrative Expenses (250,000)


Selling Expenses (225,000)

Profit before tax 200,000


Taxation (50,000)
Net Profit after tax 150,000

Required:
a) Calculate:
(i) Acid Test Ratio
(ii) Debtors Turnover Ratio
(iii) Inventory Turnover Ratio
(iv) Assets Turnover Ratio

b) Convert the income statement figures into percentages on the basis of sales (means vertical analysis).
Q.4 Following are the data related to Yee Co.

Rs.
Gross profit 35,000,000
Closing stock decreased compared to
Opening stock 75,000
Gross profit rate 33%
Stock turnover 3 times
Debtors Turnover 3 times
Creditors Turnover 5 times
Required:
Calculate the following:
 Sales
 Cost of sales
 Trade debtors
 Trade creditors
 Stock in hand

Page 15 of 58
189
“Don’t believe in luck but believe in power of prayer.”

Some extra discussion for concepts


Example:
Sting Company achieved the following results in Year 1.
1 January Year 1 31 December Year 1
Rs. Rs.
Share capital 200,000 200,000
Share premium 100,000 100,000
Retained earnings 500,000 600,000
Bank loans 200,000 500,000
Rs.
Profit before tax 210,000
Income tax expense 65,000
–––––––
Profit after tax 145,000
–––––––
Interest charges on bank loans were Rs.30,000. Dividend payments to shareholders were Rs.45,000. Sales during
the year were Rs.5,800,000.

Required
Calculate the return on capital employed for Year 1.

Answer
Capital employed at the beginning of the year = Rs.1,000,000.
Capital employed at the end of the year = Rs.1,400,000.
Average capital employed = [Rs.1,000,000 + Rs.1,400,000]/2 = Rs.1,200,000.
Profit before interest and taxation = Rs.210,000 + Rs.30,000 = Rs.240,000.

ROCE = 240,000 x 100%= 20%


1,200,000

This ROCE figure can be compared with the ROCE achieved by the company in previous years, and with the ROCE
achieved by other companies, particularly competitors.

Return on shareholders Equity [ROSC]


Using the figures in the previous example:
Shareholders’ capital at the beginning of the year = Rs.200,000 + Rs.100,000 +Rs.500,000 = Rs.800,000.
Shareholders’ capital at the end of the year= Rs.200,000 + Rs.100,000 +Rs.600,000 = Rs.900,000.
Average shareholders’ capital employed = [Rs.800,000 + Rs.900,000]/2 = Rs.850,000.

ROSC = 145,000 x 100% = 17.06%


850,000

Analyzing return: profitability and asset utilization


The size of the return on capital employed, or the size of the return on shareholders’ capital, depends on two
factors:
 The profitability of the goods or services that the entity has sold: this is known as Operating Profit Ratio.

Page 16 of 58
190
 The volume of sales that the entity has achieved with the capital and assets it has employed: this is known as
asset utilization or asset turnover ratio.

Example:
Using the figures in the previous example, the asset turnover ratio = Rs.5,800,000/Rs.1,200,000 = 4.83 times.

Note that:

ROCE = Profit x 100 x Sales________ x 100


Sales Avg Capital employed

(Where profit is defined as profit before interest and taxation).

Answer
Using the figures in the above mentioned example:
ROCE = Profit/sales × Sales/Avg. capital employed

240,000 = 240,000 × 5,800,000


1,200,000 5,800,000 1,200,000

20% = 4.14% × 4.83 times

Percentage annual growth in sales


It can be useful to measure the annual growth (or decline) in sales, measured as a percentage of sales in the previous
year.

For example, if sales in the year just ended were Rs.5,800,000 and sales in the previous year were Rs.5,500,000, the
annual growth in sales has been
(Rs.300,000/Rs.5, 500,000) × 100% = 5.45%

Purpose of working capital efficiency ratios

Working capital efficiency ratios measure the efficiency with which the entity has managed its receivables,
inventory and trade payables. The ratios are usually measured in terms of an average number of days.
The working capital ratios are a useful measure of whether the entity has too much or too little invested in working
capital.

Excessive investment in working capital is indicated by a long cash cycle (a long working capital cycle). When too
much is invested in working capital, the return on capital employed and ROSC will be lower than they should be.

Under-investment in working capital is an indication of possible liquidity difficulties. When working capital is low
in comparison with the industry average, this might indicate that current assets are being financed to an excessive
extent by current liabilities, particularly trade payables and a bank overdraft.
(The cash cycle, also called the operating cycle and the working capital cycle).

A positive working capital cycle balances incoming and outgoing payments to minimise net working capital and
maximise free cash flow. For example, a company that pays its suppliers in 30 days but takes 60 days to collect its
receivables has a working capital cycle of 30 days. This 30 day cycle usually needs to be funded through a bank
operating line, and the interest on this financing is a carrying cost that reduces the company's profitability.

Page 17 of 58
191
Growing businesses require cash, and being able to free up cash by shortening the working capitalcycle is the most
inexpensive way to grow. Sophisticated buyers review closely a target's working capital cycle because it provides
them with an idea of the management's effectiveness at managing their balance sheet and generating free cash flow.

Discussion of Debt: Equity Ratios


A high level of gearing may indicate the following:
 The entity has a high level of debt, which means that it might be difficult for the entity to borrow more when
it needs to raise new capital.
 High gearing can indicate a risk that the entity will be unable to meet its payment obligations to lenders, when
these obligations are due for payment.

The gearing ratio can be used to monitor changes in the amount of debt of a company over time. It can also be used
to make comparisons with the gearing levels of other similar companies, to judge whether the company has too
much debt, or perhaps too little, in its capital structure.

Page 18 of 58
192
“Prayer is a weapon for believers to overcome the hard situations.”

LIMITATIONS OF FINANCIAL STATEMENTS AND RATIO ANALYSIS


Financial statement are time and cost producing. Ratio analysis can be used to compare information taken from
the financial statements to gain an analytical understanding of the results, financial position and cash flows of a
business. This analysis is a useful tool, especially for an outsider such as a supplier, lender or an investor. However,
there are a number of limitations of ratio analysis which are given below:
Historical
All of the information used in ratio analysis is derived from actual historical results. This does not mean that the
same results will carry forward into the future. However, you can use ratio analysis and compare it to historical
results for consistency.
Historical versus current cost
The information on the income statement is stated in current costs (or close to it), whereas many elements of the
balance sheet are stated at historical cost (which could vary substantially from current costs). This disparity can
result in unusual ratio results.
Inflationary effect
If the rate of inflation has changed in any of the periods under review, this can mean that the numbers are not
comparable across periods. For example, if the inflation rate was 10% in one year, sales would appear to have
10% over the preceding year, when in fact sales did not change at all.

Accounting policies and estimates


Different companies in a similar industry may have different policies for recording the same accounting transaction.
For example, one company might use reducing balance method while another company uses straight-line
depreciation, FIFO method or weighted average method, cost model or revaluation model.

This means that comparing the ratio results of different companies may be like comparing apples and oranges.

Business conditions
You need to place ratio analysis in the context of the general business environment. For example, 60 days of
sales outstanding for receivables might be considered poor in a period of rapidly growing business envionment,
but might be excellent during an economic contraction when customers are in severe financial condition and
unable to pay their bills.
Interpretation
It can be quite difficult to ascertain the reason for the results of a ratio. For example, an acid test ratio of 2:1 might
appear to be excellent, until you realize that the company just sold a large amount of its stock to bolster its cash
position. A more detailed analysis might reveal that the acid test ratio will only temporarily be at that level, and will
probably decline in the near future

Company strategy
It can be difficult to interpret a ratio analysis comparison between two companies that are pursuing different
strategies. For example, one company may be following a low-cost strategy, and so is willing to accept a lower
gross margin in exchange for more market share. Conversely, a company in the same industry is focusing on a high
customer service strategy where its prices are higher and gross margins are higher, but it will never attain the
revenue levels of the first company.

Page 19 of 58
193
In short, ratio analysis has a variety of limitations that can restrict its usefulness. However, as long as you are aware
of these problems and use alternative and supplemental methods to collect and interpret information, ratio analysis
is still useful.

Interpretation of financial statements


Horizontal analysis/Vertical analysis
Overview
Financial statement analysis is the process of analyzing a company's past, current and projected performance for
decision-making purposes

Financial statement analysis allows analysts to identify trends by comparing ratios across multiple periods
and statement types to allow analysts to measure liquidity, profitability, company-wide efficiency, and
cash flow.
Financial statement analysis is of the following types:

Horizontal analysis

Vertical analysis

Ratio analysis(already explained in above sections)

Horizontal analysis

Horizontal analysis is used to compare historical data, such as ratios, or line items of financial statements,
over a number of accounting periods.
Financial analysts and investors need to identify trends and growth patterns in the company’s performance
over a number of years, a year-end balance sheet or income statement is not enough to evaluate whether the
company is operating efficiently and profitably.
Procedure:
The following is the formula for horizontal analysis:

Amount in current year – Amount in previous year x 100


Amount in previous year

The following figure is an example of how to prepare a horizontal analysis for two years.

Page 20 of 58
194
C Ltd
Profit & Loss Account
For the year ended December 31, 2015

% age change
2015 2014
from 2014 to 2015

Rs. in millions

Sales 86,320 75,200 14.79

Cost of Sales (44,618) (40,900) 9.09

Gross Profit 41,702 34,300 21.58

Other income 1,488 1,000 48.80


Distribution costs (19,597) (15,380) 27.42

Administrative expenses (2,339) (2,053) 13.93

Other operating expenses (1,322) (1,052) 25.67

Operating profit 19,932 16,815 18.54

Finance cost (343) (300) 14.33

Profit before taxation 19,589 16,515 18.61

Vertical analysis
In vertical analysis each category of line items in financial statements of the same period is shown as a percentage.
Procedure:
 Line items of assets are shown as a percentage of total assets.
 Line items of equity and liabilities are shown as a percentage of total equity and liabilities.
 Line items of income statement are shown as a percentage of sales.

This analysis of income statements gives the company information about expenses which appears to be too high
when compared to sales and allows the management to identify the reasons and take action to resolve the
problem(s).

Page 21 of 58
195
C Ltd
Statement of Financial Position
For the year ended December 31, 2015

2015 2014

Rs. in millions

Assets

Non-Current Assets

Property, plant & equipment 15,000 42.33% 12,000 32.71%

Intangibles 500 1.41% 600 1.64%

Long term investments 120 0.34% 100 0.27%

Long term loans 200 0.56% 150 0.41%

Long term deposits and


Prepayments 70 0.20% 180 0.49%

15,890 44.84% 13,030 35.51%

Current Assets

Stores and spares 650 1.83% 585 1.59%

Stock in trade 6,000 16.93% 5,500 14.99%

Trade debts 2500 7.05% 1200 3.27%

Loans and advances 800 2.26% 300 0.82%

Short term deposits and 750 900


2.12%
Prepayments 2.45%

Other receivables 350 0.99% 175 0.48%

Cash and bank balances 8,500 23.98% 15,000 40.88%

19,550 55.16% 23,660 64.49%

Total assets 35,440 100% 36,690 100%

Page 22 of 58
196
Equity and liabilities

Share capital and reserves

Share capital 1,000 2.82% 1,000 2.73%

Reserves 2,950 8.32% 7,095 19.34%

Liabilities

Non-current liabilities

Staff retirement benefits 290 0.82% 295 0.80%

Current liabilities

Trade and other payables 30,000 84.65% 27,500 74.95%

Provisions 1200 3.39% 800 2.18%

Total current liabilities 31,200 88.04% 28,300 77.13%

Total liabilities 31,490 88.85% 28,595 77.94%

Total equity and liabilities 35,440 100% 36,690 100%

C Ltd
Profit & Loss Account
For the year ended December 31, 2015
2015 2014
Rs. in millions
Sales 86,320 100% 75,200 100%
Cost of Sales (44,618) 51.69% (40,900) 54.39%
Gross Profit 41,702 48.31% 34,300 45.61%
Distribution costs (19,597) 22.70% (15,380) 20.45%
Administrative expenses (2,339) 2.71% (2,053) 2.73%
Other operating expenses (1,322) 1.53% (1,052) 1.40%
Other income 1,488 1.72% 1,000 1.33%
19,932 23.09% 16,815 22.36%
Finance cost (343) 0.40% (300) 0.40%
Profit before taxation 19,589 22.69% 16,515 21.96%

Page 23 of 58
197
“Prayer can change your situation so remember Allah and offer prayers.”

Impact of specific transactions on ratios


It is important to understand how certain future transactions or events might affect already calculated ratios, for
example, an entity may be required to certain current ratio of at least 1.75 as part of its running finance agreement
with the bank.
The following mathematical rules are useful in most circumstances:
 Increase in numerator will increase the measure of ratio
 Decrease in numerator will decrease the measure of ratio
 Increase in denominator will decrease the measure of ratio
 Decrease in denominator will increase the measure of ratio
 Increase in numerator and denominator by equal amount will increase the lower side of fraction bar, more
proportionately.
 Decrease in numerator and denominator by equal amount will decrease the lower side of fraction bar, more
proportionately.
EPIVAC last attempt

Window dressing
One should also be careful of unethical practices like window dressing while interpreting the financial statements.

Window dressing is the adaptation of the rules and practices to present financial statements in a way that business
situation appears better than it actually is. This manipulates the financial information and misleads the users of
financial statements.
Examples
Some of the ways in financial statements may be manipulated include:
 Delay in paying suppliers, so that the period-end cash balance appears higher.
 Using lower estimate for allowance for doubtful debts.
 Capitalize smaller expenditures that would normally be charged to expense, to increase reported profits.
 Offer customers an early shipment discount, thereby accelerating revenues from a future period into the
current period.
 Lower depreciation expense by using higher useful lives or residual values, etc.

Page 24 of 58
198
Solutions
A.1
Effect on Working Capital
1. No effect:
Purchase of raw material would either decrease cash or increase liability. On the other hand, it will increase the
stock in trade, so there-will be no effect on working capital.
2. Increase:
Sale of finished goods would either increase cash or increase debtors with sale price, whereas stock in trade will
decrease with the cost price, so working capital will increase only with the amount of profit in such sale.or
working capital will decrease with the amount of loss.
3. No effect:
As one effect would be in the income statement and the other would be in fixed assets.
4. Decrease:
By making payment of operating expenses, cash would decrease and on the other hand expenses in income
statement would increase. Hence, a decrease in working capital. If expenses already accrued have been paid,
then no effect.
5. Decrease:
A long-term asset would be debited whereas a current liability would be created for it. So decrease in working
capital.
6. No effect:
After the payment of a current liability out of a current asset (cash) the amount of working capital will remain
the same.
7. NO effect:
As it is conversion of a current asset (debtor) into another current asset (Cash).
8. Decrease:
If dividend is paid and declared during the same period, the amount of cash would decrease whereas current
liabilities would remain unchanged. Hence a decrease in the amount of working capital.
9. Increase:
Cash (a current asset) will increase by the issue of shares. Hence, increase in the amount of working capital.
10. No change:
A current asset (cash) would increase as well as current liability (short term loan) would also increase. Hence,
no in working capital.

Page 25 of 58
199
A.2

Acoms

(i) Gross profit percentage Gross profit x 100 95__ x 100. = 25.3%
Revenue
375
(ii) Operating profit percentage Net profit from operations x 100 50 x 100 = 13.3%
Revenue 375

(iii) Current ratio Current assets 133 =1.3:1


Current liabilities 103

(iv) Acid test (Quick) ratio Current assets — inventory 13 3 -9 6 =0 .3 6 :1


Current liabilities 103

(v) Receivables collection period Trade receivables x3 6 5 34 X 36 5 =33days


Sales 375

(b)
Comments on the performance of Acorns

Gross-Profit
Gross profit percentage has reduced from the previous ‘year by 27%. This might indicate increased competition in
the market and that selling prices have been discounted. Alternatively the cost of purchases may have increased
significantly. The situation is particularly worrying because this ratio is now below the industry average.

Operating Profit
The net profit percentage has also deteriorated on the previous year and is below the industry average. This
suggests that the control of costs needs to be improved if the company is to remain competitive.

Current Ratio
The current ratio has deteriorated slightly on the previous year and also ‘below industry average. The business has
sufficient current assets to cover its current liabilities. However, the composition of the current assets is: heavily
weighted with inventory. The company may have problems converting inventory to cash if it is required quickly (as
inventories not converting into cash quickly).

Acid Test Ratio


The acid test ratio gives a better indication of liquidity than the current ratio. This ratio is 0.36:1 and has fallen
significantly below the industry average. This ratio suggests the company may be experiencing some liquidity
problems. The current, inventory levels might also indicate the business is having some trading problems, (as
inventory is not converting into cash quickly).

Receivables collection period


The receivables collection period has more than doubled since tire previous year and is 13 days longer than the
industry average. The business may be giving customers, more credit in order to sell more inventory. Alternatively
the receivables collection procedures may need to be tightened up, which would help to improve the business
liquidity situation.

Page 26 of 58
200
“May Allah guide us to offer prayers in the light of Quran and Sunnah.”

A.3

Solution
Limited Company
(a)
(i) Acid Test Ratio = Current Asset – Inventories
Current Liabilities
= 800,000 – 310,000
575,000
= 0.85 times or 0.85:1

(ii) Debtors Turnover Ratio = Total Credit Sales


Average Debtors
= 1,675,000
350,000
= 4.79 times

(iii) Inventory Turnover Ratio Cost of Sales____


Average Inventory
= 1,000,000
310,000
= 3.23 times

(iv) Asset Turnover Ratio = Net Sales______


Capital Employed
= 1,675,000___
950,000 + 725,000
= 1 time

(b) Income Statement as a percentage of sale

Percentage (%)

Sale 100.00

Cost of Sales 59.70

Gross Profit 40.30

Administrative Expenses 14.93

Selling Expenses 13.43

28.36

Page 27 of 58
201
Profit before tax 11.94

Taxation 2.99

Profit after Tax 8.95

A.4

Solution
Yee Co

Sales (W-1) 106,060,606


Cost of Sales (W-2) 71,060,606
Trade Debtors (W-3) 35,353,535
Stock in Hand (W-5.2) 23,649,369
Trade in Creditors (W-5) 14,197,121

Workings
W-1
Gross Profit Ratio = Gross Profit x 100
Sales
33% = 35,000,000 x 100
Sales
Sales = 35,000,000 x 100
33
= 106,060,606
W-2
Cost of Sales = Sales – Gross profit
= 106,060,606 – 35,000,000
= 71,060,606
W-3
Debtors Turnover = Credit Sales
Trade Debtors
3 = 106,060,606
Trade Debtors
Trade Debtors = 106,060,606
3
= 35,353,535 approx
Note: as there is no information of opening balance of debtors, therefore we assume that average debtors are
closing as well.

W-4
Stock Turnover = Cost of Sales
Average Stock
3 = 71,060,606__
Average Stock

Page 28 of 58
202
Average Stock = 71,060,606
3
= 23,686,869 approx

W-5
Creditors Turnover =Credit Purchases
Trade Creditors
5 = 70,985,606(W-5.1)
Trade Creditors
Trade Creditors = 70,985,606
5
= 14,197,121 approx
Note: as there is no information of opening balance of creditors, therefore we assume that average creditors are
closing as well.

W-5.1 Stock A/c

b/d 23,724,369 71,060,606

Purchases 7,085,606 c/d (5.2) 23,649,369

W-5.2
We know from W-4 that
Opening Stock +Closing Stock = 23,686,869
2
Let the opening stock be X then closing stock will be X -75,000
X +X-75,000 = 23,686,869
2
2X -75,000 23,686,869 x 2
47,373,738
2X 47,373,738 + 75,000
47,448,738
X (opening stock) = 23,724,369
Closing Stock = 23,724,369 – 75,000
23,649,369

Or
If x =closing stock
Then
X+75,000+X = 23,686,869

2
X = 23,649,369

Page 29 of 58
203
“Prayer becomes powerful when it is done with patience and sincere faith in Allah.”

Extra Practice Questions:


Question No. 1
Following amount have been determined from the records of Hassan Limited.

2014 2015 2016


Description
---------- Rs. In million -----------
Sales 100.00 120.00 135.00
Cost of sales 75.00 90.00 101.25
Profit before interest and tax 6.00 5.50 5.60
Account receivable 16.50 25.00 35.00
Account payable 13.00 14.70 15.00
Inventory 18.75 26.00 30.40
Cash at bank / (overdraft) 5.00 (0.50) (2.00)

Required:
Calculate liquidity ratios and working capital cycle for 2015 and 2016 and comment on the results of your
calculation, assuming that all sales and purchases are made on credit. (10)
Question No. 2
Sachal Limited (SL) is planning to acquire 100% shareholding in Waris Limited (WL). Before submission of financial
proposal, SL is carrying out an analysis of WL’s financial and operating performance. The CFO of SL has gathered
the following information which is based on the financial statements for the year ended December 31, 2010:

Industry Ratios
Description WL’s Ratios
High Low Average
Operating Performance Ratios
Gross profit 29% 30% 20% 25%
Operating profit 11% 15% 10% 13%
Return on shareholders equity 9% 13% 7% 10%
Working Capital Ratios
Current ratio 1.54:1 2:1 1:1 1.5:1
Inventory turnover 83 days 114 days 81 days 91 days
Receivables collection 93 days 95 days 60 days 74 days
Gearing ratios
Debt equity ratio 55:45(122%) 60:40(150%) 40:60(67%) 50:50(100%)
Interest cover 1.3 times 3 times 1.2 times 2 times

Required:
Draft a report to the board of directors on behalf of the CFO, analyzing the financial performance of Waris Limited
by evaluating each category of ratios in comparison with the industry. (10)

Page 30 of 58
204
Question No. 3
Emeraid Limited and Garnet Limited are two companies engaged in manufacturing electrical appliances. Both
operate from rented premises. Their financial statements at 31 March 1997 were as follows:

Profit and loss accounts for the year ended 31 March 1997
Emeraid Garnet
Rs. 000 Rs. 000 Rs. 000 Rs. 000
Sales 1,075 756
Less: Costs of sales
Materials (360) (280)
Manufacturing wages (130) (140)
Depreciation of plant (125) (74)
Other manufacturing cost (100) (715) (80) (574)
Gross profit 360 182
Sundry expenses (125) (95)
Operating profit 235 87
Interest on debentures (10) (30)
Net profit before tax 225 57
Taxation (50) (15)
Net profit after tax 175 42
Proposed dividend (100) (40)
Retained profit 75 2

Balance sheet as at 31 March 1997


Emeraid Garnet
Rs. 000 Rs. 000 Rs. 000 Rs. 000
Tangible fixed assets
Plant and machinery:
Cost 1,250 1,480
Aggregate depreciation (450) 800 (1,160) 320
Office equipment:
Cost 65 180
Aggregate depreciation (20) 45 (82) 98
Motor vehicles:
Cost 60 50
Aggregate depreciation (20) 40 (20) 30
885 448

Current Assets:
Stock 210 170
Debtors 400 300
Other Receivables 50 40
Cash at bank 20 -
680 510

Page 31 of 58
205
Less: Current liabilities
Creditors (100) (140)
Sundry (including taxation) (80) (60)
Overdraft at bank (100) (280)
Proposed dividend (100) (40)
(380) (520)
Net current asset 300 (10)
1,185 438

10% Debentures (100) (300)


1,085 138
Share capital 600 100
Profit and loss account 485 38
1,085 138

Required:
(a) Calculate the following ratios for both companies:
(i) Current ratio
(ii) Quick ratio/acid test
(iii) Debtors collection period in days
(iv) Return on capital employed (ROCE)
(v) Return on owners equity (before taxation)
(vi) Gearing ratio
(vii) Interest Cover
(viii) Gross profit percentage on sales
(ix) Operating profit percentage on sales.

Page 32 of 58
206
Question No. 4
You are a trainee CA Accounting Technician who has been asked to analyse the performance of two companies Aye
Ltd and Bee Ltd. The two companies operate in the same line of business. You are presented with the following
information for each company for the financial year 1997/98.
Balance Sheets as at 31 May 1998

Aye Ltd Bee Ltd


Rs. 000 Rs. 000 Rs. 000 Rs. 000 Rs. 000 Rs. 000
Fixed Assets
Land 100 150
Buildings 90 115
Plant 30 65
220 330
Current Assets
Stocks 85 55
Debtors 110 50
Bank - 10
195 115
Less Current Liabilities
Creditors 120 70
Taxation 5 20
Bank 55 -
180 90
Net current assets 15 25
Debenture loan (70) (85)
165 270
Capital and Reserves
Share capital 100 100
Profit and loss account 65 170
165 270

Additional Information for the financial year 1997/98

Aye Limited Bee Limited


Rs. ‘000’ Rs. ‘000’
Sales 1,100 1,500
Cost of sales 385 1,050
Opening Stock 25 115
Profit before interest and tax 55 150
Required:
Calculate the following ratios for each company.

Page 33 of 58
207
(i) Gross Profit Percentage.
(ii) Operating Profit Percentage
(iii) Current Ratio
(iv) Acid Test Ratio (i.e. Liquidity Ratio)
(v) Stock Turnover Period.
(vi) Creditor Payments period (in months)

Question No. 5
The following are the financial statements of d Limited, a wholesaling company, for the year ended 31 December:
Profit and Loss Accounts
For the year ended 31st December, 2004 and 2005

20X4 20X4 20X5 20X5


Rs. 000 Rs. 000 Rs. 000 Rs. 000
Sales
Credit sales 2,200 2,640
Cash sales 200 160
2,400 2,800
Less: Cost of sales (1,872) (2,212)
Gross profit 528 588
Distribution costs (278) (300)
Administration expenses (112) (114)
Operating profit 138 174
Interest expense - (32)
Profit before tax 138 142

Balance sheet as on 31st December, 2004 and 2005

20X4 20X4 20X5 20X5


Rs. 000 Rs. 000 Rs. 000 Rs. 000
Tangible fixed assets 220 286
Current assets:
Stocks 544 660
Debtors 384 644
Cash at bank 8 110
936 1,414
Creditors: amounts falling due within one year:
Trade creditors (256) (338)
Net current assets 680 1,076
Total assets less current liabilities 900 1,362
Creditors: amounts falling due after more than one
year:

Page 34 of 58
208
Debentures loans - (320)
Share holders funds/equity 900 1,042

Required:
Using the information above, calculate the following ratio for both years:
(a) Profitability (GP%, NP%, Return on capital employed, Distribution cost to sale, Administration cost to sale).
(b) Liquidity (Current ratio, Quick ratio, debtor collection period in week, Creditor turnover period in week,
Inventory turnover in week). (10)
Note:
 Average will only be calculated if it can be for all relevant years.
 If purchases cannot be calculated for all relevant years, then use cost of sales instead.

Q.6 (a) The following information has been gathered by an analyst, in respect of Dairy Foods Limited (DFL) which
specializes in various dairy products.

Industry
Ratio 2016 2015 2014
Average
Profit margin % 11% 10% 8% 10.45%
Quick ratio 1.38 1.40 1.42 1.52
Current ratio 1.84 1.67 1.59 1.73
Days purchases in payables 80 91 89 82

In the latest annual report to the shareholders, Directors of DFL have claimed that liquidity position of the
Company has improved significantly.

Required:
Critically analyze and discuss whether you agree with the claim. (03)

(b) Extracts from latest financial statements of two companies are as follows:

Extracts from statements of financial position


A B A B
Equity and liabilities Assets
Rs. in million Rs. in million
Equity and reserves 51,690 72,114 Fixed assets 34,460 48,076
Long term loan - 36,057 Stock in trade 21,700 20,000
Trade creditors 35,790 45,135 Trade debtors 24,470 44,030
Other payables 12,000 8,500 Cash and bank 18,850 49,700
99,480 161,806 99,480 161,806

Page 35 of 58
209
Extracts from statements of comprehensive income
A B
------ Rs. in million ------
Revenue 161,600 220,150
Cost of sales (135,160) (180,520)
Gross profit 26,440 39,630
Operating expenses (9,840) (13,870)
Interest expense (720) (2,313)
Profit before tax 15,880 23,447
Income tax (333) (409)
Profit after tax 15,547 23,038

Required:
Analyze the profitability, liquidity and working capital ratios of both the companies. (12)

Page 36 of 58
210
Surely, good deeds erase bad deeds.

Answers:
Answer No. 1

2015 2016
Current Ratio
=
Current assets 51(25  26) 65.4(35  30.4)
Current liabilities 15.2(14.7  0.5) 17(15  12)
3.36 : 1 3.85 : 1
Quick Ratio = 25 35
Current assets  Inventory 15.2 17
Current liabilities 1.64 : 1 2.06 : 1
Account Average debtors [(25  16.5) / 2] 20.75 [(35  25) / 2] 30
Receivable = × 365 × 365 × 365
Credit Sales 120 135
collection
period 63.11 days 81.11 days

Inventory Average inventory [(26  18.75) / 2] 22.30 [(30.4  26 ) / 2] 28.2


holding period = × 365 × 365 × 365
Cost of Sales 90 101.25
90.74 days 101.66 days
Account Average Creditor [(14.7  13) / 2 ] 13.85 [(15  14.7 ) / 2 ] 14.85
payable = × 365 ×365 ×365
Credit purchases [(90  26  18.75)] 97.25 [(101.25  30.4  26 )]105.65
payment
period 51.98 days 51.30 days

Working Capital Cycle Days Days


Average days to collect receivable 63.11 81.11
Average inventory holding period 90.74 101.66
Less: Average time to pay accounts payable (51.98) (51.30)
101.87 131.47

Comments
 The company’s liquidity position, as evidenced from the current ratio and the quick ratio, appears to be
growing stronger. However, the working capital cycle of the company is getting longer in 2016 as compared
to 2015.
 The company may face liquidity problem in future, as debtors days are increasing and a large cash is blocked
in inventory.
 Higher investment in working capital would result in decrease in ROCE and Return on shareholders equity.
 The increase in debtor days may suggest inefficient collection of amounts due from debtors.

Page 37 of 58
211
Answer No. 2
To: Board of Director
From: Chief Financial Officer
Date: December 8, 2010
Subject: Financial and Operating performance of Waris Limited
As requested, I have analyzed the financial performance of Waris Limited (WL) with the industry with a view to
evaluate the feasibility of launching a takeover bid. My analysis of each category of ratios is as follows:
Profitability Ratios
The gross profit ratio is near to the highest while the operating profit is near to the lowest as compared to similar
companies. It indicates that key issue which is affecting WL’s profitability is its lack of control over operating
expenses. The positive aspect of situation is that we may be able to improve the profitability just by controlling the
operating expenses without being required to make significant changes in the current operations of WL.
Return on shareholders’ equity is around average prevailing in the industry. This ratio is obviously related to
operating profit and as discussed above it can be improved by exercising greater control over operating expenses,
after takeover.

Working Capital Ratios


WL’s working capital ratios specially the current ratio indicates that the Company’s working capital is being
appropriately managed although there may be some room for improvement.
The inventory turnover is among the lowest in the industry which shows that sound inventory management policies
are in place.
However, the level of receivables is among the highest in the industry. The possible causes of the situation may be
as follows:
 Poor efforts in making collection
 Lack of proper credit control polices or slackness in their implementation.
 Chances of bad debts which may not have been provided.

We need to seek appropriate explanation and investigate the matters if possible.


Gearing Ratios
The debt equity ratio is on the higher side.The interest cover is only 1.3. It is among the lowest in the industry and
is indicative of a high degree of risk as the profits are hardly able to cover the interest charges. Even a slight decline
in the profitability of the company may have highly adverse impact on the company’s profit after tax. (M-2)

Page 38 of 58
212
“You need to believe in Allah and obey him so that your prayer be accepted.”

Answer No. 3
Ratios:
Emerald Limited
Rs. ‘000’
1. Current Ratio: Emerald Garnet
Current Asset / Current Liabilities 680/380 510/520
1.79 times 0.98 times
2. Quick Ratio:
Current Asset - Stock / Current Liabilities 680 – 210/380 570 – 170/520
1.24 times 0.65 times
3. Debtor’s Collection Period:
Debtors / Credit Sales × 365 400/1,075×365 300/756×365
136 days 145 days
4. Return on Capital Employed:
Operating Profit / Capital 235/1,185*×365 87/438*×100
Employed × 100 19.8% 19.86%
*(1,085+100) *(138+300)

5. Return on Equity:
Profit before tax / equity × 100 225/1,085×100 57/138×100
20.7% 41.30%
6. Gearing Ratio:
Long term debt / Capital employed × 100 100/1,185×100 300/438×100
8.4% 68.5%
Or
Long term debt / Equity × 100 100/1,085×100 300/138×100
9.21% 217%
7. Interest Cover:
Profit before Interest & tax / Interest 235/10 87/30
23.5 times 2.9 times
8. Gross Profit Ratio:
Gross Profit / Sales × 100 360/1,075×100 182/756×100
33.5% 24.1%
9. Operating Profit Ratio:
Operating Profit / Sales × 100 235/1,075×100 87/756×100
21.86% 11.51%

Page 39 of 58
213
Answer No. 4
Training C.A

Rs. ‘000’
1. Gross Profit: Ayee Ltd. Bee Ltd.
Gross Profit / Sales × 100 (1,100–385)/1,100×100 (1,500–1,050)/1,500×100
65% 30%
2. Operating Profit:
Operating Profit / Sales × 100 55/1,100 × 100 150/1,500 x100
5% 10%
3. Current Ratio:
Current assets/Current Liabilities 195/180 115/90
1.08 times 1.28 times
4. Quick Ratio:
Current assets - Stock/ 110/180 60/90
Current Liabilities 0.61 times 0.67 times
5. Stock turnover:
Average Stock / Cost of Sales x365 55*/185×365 85*/1,050×365
52 days 30 days
*(25+85)/2 *(115+55)/2
6. Creditors Payment Period:
Creditors / Purchases × 12 120/445* × 12 70/990* × 12
3 months 1 months
*Stock *Stock

b/d 25 COS 385 b/d 115 COS 1050


445 990
c/d 85 c/d 55

Answer No. 5
(a) Profitability Ratio
2004 2005

Gross profit 528 588


Gross profit as % of sales = × 100 × 100 = 22% × 100 = 21.00%
Sales 2,400 2,800

Net profit 138 142


Net profit as % of sales = × 100 × 100 = 5.75% × 100 = 5.07%
Sales 2,400 2,800

Pr ofit before int erest & tax 138 174


Return on capital employed = ×100 × 100 = 15.33% × 100 = 12.78%
Average Capital Employed 900 1,362

Distributi on Costs 278 300


Distribution costs/sales = × 100 × 100 = 11.58% × 100 = 10.71%
Sales 2,400 2,800

Ad min istration exp enses 112 114


Administration expenses/sales = × 100 × 100 = 4.67% × 100 = 4.07%
Sales 2,400 2,800

Page 40 of 58
214
(b) Liquidity Ratio
2004 2005

Current Assets 936 1,414


Current Ratio = = 3.66:1 = 4.18:1
Current Liabilitie s 256 338

(Current Assets  Inventory ) 392 754


Quick Ratio = = 1.53:1 = 2.23:1
Current Liabilitie s 256 338

Efficiency Ratio
Trade receivable s 384 644
Avg. weeks to collect = × 52 × 52 = 9.07 weeks × 52 = 12.7 weeks
Credit sales 2,200 2,640

Inventory 544 660


Inventory holding weeks = × 52 × 52 = 15.11 weeks ×52 = 15.52 weeks
Cost of sales 1,872 2,212

Trade payables 256 388


Avg. weeks to pay = × 52 × 52 = 7.11 weeks ×52 = 7.946 weeks
Credit purchases / COS * 1,872 2,212

*(If there is no information about purchase then use cost of sales.)

A.6 (a) While analyzing liquidity positions of DFL, it is noted that current ratio has steadily increased over the
years and is better than industry average. However, the quick ratio has steadily declined and is even
lower then industry average. This is a clear evidence that the increase in liquidity is caused by an
increase in inventory.

Based on the above, I do not agree with the claim of DFL’s directors.

(b) Profitability ratios A B


Gross profit ratio (GP ÷ sales) 16.36% 18.00%
Profit to sales (Profit after tax ÷ sales) 9.62% 10.46%
Return on capital employed (Profit before interest and tax ÷ capital
employed) 32.11% 23.81%
Return on asset employed (Profit before interest and tax ÷ assets) 16.69% 15.92%

Company B's gross profit and net profit ratio is slightly higher as compared to Company A. The
difference is not significant and may be on account of higher level of sales resulting in lesser fixed
costs per unit.

Company A’s return on capital employed ratio and return on asset employed ratio are better than
Company B, because Company B has accumulated large balances of cash despite of availing long
term loan. Had Company B had used its cash balances to pay off the long term loan, it would have
both of these ratio better than Company A.

Liquidity Ratios A B
Current ratio (current assets ÷ current liabilities) 1.36 2.12
Quick ratio (current asset-inventory ÷ liabilities) 0.91 1.75

Company B has better current and quick ratio. However, it appears that these ratios are better than
Company A due to substantially high amount of trade debts in term of percentage

Page 41 of 58
215
“Prayer is the best medicine for sick soul and Allah is one who cures the sick soul.”

of sales as sales days. It also represents a risk that these trade debts may prove irrecoverable.
Moreover, they may be indicative of inefficient in debt collection as well.

Working capital turnover ratios A B


Stock turnover days (Stock ÷ Cost of goods sold × 365) [A] 58.60 40.44
Debtor turnover days (Debtor ÷ Revenue × 365) [B] 55.27 73.00
Creditor turnover days (Creditor ÷ Cost of goods sold × 365) [C] 96.65 91.26
Cash operating cycle [A+B–C] (days) 17.22 22.18

Stock turnover of Company B is better than that of Company A. Company B is turning over its stock
9 times whereas company A is doing it 6 times a year.

Company A is more effectively collecting it’s debtors than Company B. This could also be due to the
fact that Company B is following a lenient credit policy to attract more revenue. This fact is also
supported from higher stock turnover ratio of Company B.

Company A have availed better credit facility from its creditors but it may have forgone some
settlement discounts which might have resulted in lower gross profit ratio than that of Company B.

Overall cash operating cycle of Company A is better than Company B. Furthermore Company B has
accumulated large balances of cash despite the fact that it has also availed long term loan. Excess
cash balance should have been used to pay off the long term loan to reduce the finance cost.

Page 42 of 58
216
Extra Practice Questions:
Q.1 Boom Limited (BL) is a manufacturer of sports goods. Following financial statements for the year ended 31
December 2017 have been submitted to the Chief Executive Officer (CEO).

Statement of profit or loss

Rs. in ‘000
Revenues 21,000
Cost of sales (17,500)
Gross profit 3,500
Operating expenses (1,900)
Finance cost (450)
Profit before tax 1,150
Taxation (345)
Profit after tax 805

Statement of financial position

Rs. in ‘000
Property, plant and equipment 7,500
Current assets 1,500
9,000

Share capital 4,000


Reserves 1,000
Non-current liabilities 3,000
Current liabilities 1,000
9,000

Although performance of BL has improved from the last year, CEO wants to compare the results with other
companies operating in sports manufacturing industry. In this respect, following industry data has been gathered:

Gross profit margin 23.5%


Net profit margin 7.7%
Current ratio 2.75
Gearing ratio 50:50
Return on non-current asset 32.9%
Return on capital employed 27.4%
Return on equity 31.3%

Required:
(a) Compute BL’s ratios for comparison with the industry. (04)
(b) For each ratio, give one possible reason for variation from the industry. (07)

Page 43 of 58
217
“When prayer become part of your life then success becomes lifestyle.”

Ans.1 Boom Limited

Comparison of BL's ratios with industry average and possible reasons for variation

Ratios (a) BL's ratios Industry's ratios


Gross profit margin 16.67% (3,500/21,000 x100) 23.50%
Net profit margin 3.83%(805/21,000 x100) 7.70%
Current ratio 1.5(1,500/1,000) 2.75
Gearing ratio 37.5:62.5(3,000/(4,000+1,000+3,000) x100) 50:50
Return on non- 21.33%(1,150+450)/7,500 x100) 32.90%
current assets
Return on capital 20.00%(1,150+450)/(4,000+1,000+3,000) x100) 27.40%
employed
Return on Equity 16.10%(850/(4,000+1,000) x100) 31.30%

(b) Reasons for variation from industry


Gross profit margin:
Lower than industry
 Purchase of raw material at higher prices as compared to its competitors
 Inability to obtain economies of scale in Production as compared to its competitors
 Higher production costs due to
 Inefficiencies
 Deliberately keeping selling prices lower

Net profit margin


Lower than industry (mainly due to lower gross profit)
BL’s gross profit margin is 6.83% lower than industry (16.67% Vs 23.5%) whereas net profit margin is only 3.87%
(7.7%-3.83%) lower which indicates that BL’s operating expenses as a percentage of sales are approximately
2.96% (6.83%-3.87%) lower than the Industry

Current ratio
Lower than industry
Since gearing ratio is lower than the industry so BL might have:

 obtained running finances as compared to long-term financing by the industry


 availed extended credit terms from Suppliers (means creditors are more)
 Low inventory levels are maintained by BL
 Shorter credit terms are given to debtors

Gearing ratio
Lower than industry
 Difficulty in raising long-term finance from banks due to low profits
 Running finance or extended credit terms from suppliers are available for BL

Return on non- current assets


Lower than industry
 Lower profit margins
 Relatively newer non-current assets have higher carrying value

Page 44 of 58
218
Return on capital employed
Lower than industry
 Lower profit margins
 High shareholder’s equity

Return on Equity
Lower than industry
 Lower profit margins
 High shareholder’s equity

Page 45 of 58
219
“Perform your prayers (salat) timely and seek help from Allah through Prayer.”

Q.2 Following are the summarised financial statements of Keyboard Limited (KL):

Statement of financial position


2018 2017 2016
----------- Rs. in '000 -----------
Fixed assets 12,500 10,800 11,800

Current assets:
Inventory 4,000 4,500 3,000
Debtors 4,200 3,200 1,800
Cash - 800 2,100
8,200 8,500 6,900
20,700 19,300 18,700

Equity and reserves 10,400 9,000 8,600


Long term loan 4,400 5,000 5,600

Current liabilities:
Creditors 3,500 4,400 4,200
Bank overdraft 1,500 - -
Accrued expense 900 900 300
5,900 5,300 4,500
20,700 19,300 18,700

Statement of profit or loss


2018 2017 2016
----------- Rs. in '000 -----------
Sales 27,000 24,400 21,000
Cost of goods sold (21,300) (19,400) (17,200)
Gross profit 5,700 5,000 3,800
Operating expenses (3,400) (3,000) (2,400)
Finance cost (300) (350) (400)
Net profit 2,000 1,650 1,000

Required:
(a) Compute working capital cycle in days and liquidity ratios for 2018 and 2017. (11)
(b) Suggest three possible measures that can be taken by KL to improve working capital cycle days.
(03)

Page 46 of 58
220
Ans.2 (a) Working capital cycle 2018 2017
------------- Number of days -------------
Average days to collect debtors W-1 50.0 37.4
Average inventory holding period W-2 72.8 70.5
Less: Average time to pay creditors W-3 (69.3) (75.1)
53.5 32.8

Liquidity ratios: 2018 2017


Current ratio = Current assets 8,200 8,500
Current liabilities 5,900 5,300

1.39 : 1 1.60 : 1

Quick ratio = Current assets–inventor 8,200 – 4,000 8,500 – 4,500


Current liabilities 5,900 5,300

0.71 : 1 0.75 : 1

W1 Debtors = Average debtors x 365 [(4,200+3,200)÷2] x 365 [(3,200+1,800)÷2] x 365


collection Credit sales 27,000 24,400
period (in
days) 50 Days 37.4 Days

W2 Inventory = Average inventory x 365 [(4,000+4,500)÷2] x 365 [(4,500+3,000)÷2] x 365


holding Cost of sales 21,300 19,400
period (in
days) 72.8 Days 70.5 Days

W3 Creditors = Average creditors x 365 [{3,500+4,400)÷2] x 365 [(4,400+4,200)÷2 x 365


payment Credit purchases [21,300*+4,000 - 4,500] (19,400+4,500-3,000)
period (in
days) 69.3 Days 75.1 Days

W-4) stock

Page 47 of 58
221
“Be humble in you prayer and Pray to none but Allah.”

W-5)
Stock
b/d 3,000 COS 19,400
20,900

c/d 4,500

(b) Measures to improve working cycle days:


 Give incentives to customers to pay on time.
 Do not transact with customers who have a history of defaulting / late payments.
 Automate the monitoring of accounts receivables.
 Try to sell the inventory quickly.
 Negotiate with suppliers to increase credit period.

Q. 3 SK Limited (SKL) deals in a single product. Following are the summarized financial statements of SKL for the year
ended 31 December 2017:
Statement of financial position Statement of profit or loss

2017 2016
Rs. in million 2017 2016
Fixed assets 410 240 Units sold in million 39 30
Current assets 90 200
500 440 Rs. in million
Sales 371 300
Cost of goods sold (273) (210)
Capital 280 260
Long-term loan 170 100 Gross profit 98 90
Selling and administrative (55) (60)
Current liabilities 50 80
Finance cost (13) (8)
500 440
Net profit 30 22
Additional information:
i. With effect from 1 January 2017, selling price was decreased by 5% to boost sales volume.
ii. During the year 2017, suppliers demanded price increase of 4%. SKL resisted the price increase. However, both
parties agreed to reduce the credit period.
iii. SKL had been running its business in a rented building whose annual rent was Rs. 15 million. During the year,
SKL purchased this building for Rs. 200 million. Funds were arranged partially through a long-term loan. Useful
life of the building is estimated at 40 years.
iv. 75% of the selling and administration cost incurred in 2016 was fixed cost.

Required:
(a) Compute the following ratios for 2016 and 2017:

Gross profit margin Net profit margin


Return on capital
Return on assets employed

Page 48 of 58
222
Debt equity ratio Current ratio (08)

(b) Keeping in view the above information, comment on profitability and liquidity position of SKL for 2017.
(04)

Page 49 of 58
223
“You are never too late nor old to move forward and make your life best.”

A. 3 SK Traders
Computation of ratios

2017 2016
(i) Gross profit margin 26.42% 30.00%
(98 ÷ 371) x 100 (90 ÷ 300) x 100
(ii) Net profit margin 8.09% 7.33%
(30 ÷ 371) x 100 (22 ÷ 300) x 100
(iii) Return on assets 8.60% 6.82%
(30 + 13) ÷ 500 x 100 (22 + 8) ÷ 440 x 100
(iv) Return on capital employed 9.56% 8.33%
(30 + 13)/280 + 170) x 100 (22 + 8) ÷ (260 + 100) x 100
(v) Debt equity ratio 0.38% 0.28%
170 ÷ (280 + 170) x 100% 100 ÷ (260 + 100) x 100%
(vi) Current ratio 1.80 2.50
90 ÷ 50 200 ÷ 80

(b)
(i) Profitability:
In 2017, gross profit margin of SKL has reduce from 30% to 26.42%. however, gross and net profits
amounts have been increased by Rs. 8 million mainly due to:
 Increase in sales volume as a result of 5% decrease in selling price. This resulted in increase in
gross profit by 8.89%[(98-90)÷90×100]. ( Horizontal analysis)
 Acquisition of building has resulted in savings in expenses as rent saved (Rs. 15 million) is higher
than the depreciation (Rs. 5 million) and increased in finance cost (Rs. 5 million).
 Since 75% of selling and administrative cost was fixed, expenses did not increase due to increase
in sales volume (economies of scale).

(ii) Liquidity:
 The decrease in current ratio from 2.5 to 1.8 is net effect of the following:
 Cash payment for purchase of building which significantly decreased current assets.
 Prompt payment to suppliers which decreased the current liabilities.

Page 50 of 58
224
Further practice
1. WASIM
Wasim is an importer and retailer of vegetable oils. Extracts from the financial statements for this year and last are
set out below:
Income statements for the years ended 30 September

Year 7 Year 6
Rs. 000 Rs. 000
Revenue 2,160 1,806
Cost of sales (1,755) (1,444)
Gross Profit 405 362
Distribution costs (130) (108)
Administrative expenses (260) (198)
Profit before tax 15 56
Income tax expense (6) (3)
Profit for the period 9 53

Statement of financial position as on September

Year 7 Year 6
Rs. 000 Rs. 000
Assets
Non-current assets
Property, plant and equipment 78 72
Current assets
Inventories 106 61
Trade receivables 316 198
Cash - 6
422 265
Total assets 500 337
Equity and liabilities
Equity
Ordinary shares 110 85
Preference shares 23 11
Share premium 15 -
Revaluation reserve 20 20
Retained earnings 78 74
246 190
Current liabilities
Bank overdraft 49 -
Trade payables 198 142

Page 51 of 58
225
Current tax payable 7 5
254 147
Total equity and liabilities 500 337

Required:
Calculate profitability ratios, liquidity ratios and working capital ratios for both years to make comparison:
Profitability ratios means:
(a) Gross profit percentage.
(b) Net profit percentage
(c) Return on capital employed
(d) Asset turnover
Liquidity ratios means:
(e) Current ratio
(f) Quick ratio
Working capital ratios means:
(g) Average receivables collection period (average time to collect)
(h) Average payable period (average time to pay)
(i) Inventory turnover period

2. AMIR AND MO
The income statements and statements of financial position of two manufacturing companies in the same sector are
set out below:
Amir Mo
Rs. 000 Rs. 000
Revenue 150,000 700,000
Cost of sales (60,000) (210,000)
Gross profit 90,000 490,000
Interest payable (500) (12,000)
Distribution costs (13,000) (72,000)
Administrative expenses (15,000) (35,000)
Profit before tax 61,500 371,000
Income tax expense (16,605) (100,170)
Profit for the period 44,895 270,830
Assets
Non-current assets
Property - 500,000
Plant and equipment 190,000 280,000
190,000 780,000

Page 52 of 58
226
Current assets
Inventories 12,000 26,250
Trade receivables 37,500 105,000
Cash at bank 500 22,000
50,000 153,250
Total assets 240,000 933,250
Equity and liabilities
Equity
Share capital 156,000 174,750
Retained earnings 51,395 390,830
207,395 565,580
Non-current liabilities
Long-term debt 10,000 250,000
Current liabilities
Trade payables 22,605 117,670
Total equity and liabilities 240,000 933,250

Required:
Calculate profitability ratios, liquidity ratios and working capital ratios for each company to make comparison:
Profitability ratios means:
(a) Gross profit percentage.
(b) Net profit percentage
(c) Return on capital employed
(d) Asset turnover
Liquidity ratios means:
(e) Current ratio
(f) Quick ratio
Working capital ratios means:
(g) Average receivables collection period (average time to collect)
(h) Average payable period (average time to pay)
(i) Inventory turnover period

Page 53 of 58
227
“Unity gives victory.”

Q.3 ALPHA LIMITED AND OMEGA LIMITED


Alpha Limited and Omega Limited are in the same trade, but operate in different areas. Their accounts for the year
ended 31 December, 2016 are as follows:

Profit and loss account Alpha Limited Omega Limited


Rs. ‘000’ Rs. ‘000’ Rs. ‘000’ Rs. ‘000’
Sales 1,440 1,720
Less: Cost of sales 1,120 1,342
Gross profit 320 378
Less: overheads 220 300
Profit before tax 100 78
Taxation 40 30
Dividends 20 24
60 54
Retained earnings 40 24
Statement of financial position
Share capital of Rs. 1 each 600 200
Reserves 240 104
840 304
8% Debentures - 120
840 424
Represented by:
Non-current assets at cost 660 520
Less: Accumulated Depreciation 200 160
460 360
Current assets:
Inventory 280 172
Receivables 310 300
Cash 30 32
620 504
Current Liabilities:
Taxation 40 30
Creditors 180 344
Bank overdraft - 42
Dividends 20 24
240 440
Net Current assets 380 64

Page 54 of 58
228
840 424

Required:
(a) Compute the following ratios for each of the companies:
(i) Current ratio
(ii) Acid test
(iii) Creditors ratio (payment period)
(iv) Collection Period or Receivables Ratio
(b) Carry out comparative analysis of the companies based on the computed ratios in (a) above.

A.1 WASIM
Ratios

Profitability ratios:
Year 7 Year 6
Gross profit % =
Gross profit 405 362
× 100 × 100 = 19% × 100 = 20%
Sales 2,160 1,806

Net profit % =
Net profit 9 53
× 100 ×100 = 0.4% ×100 = 2.9%
Sales 2,160 1,806
Return on capital employed =
Pr ofit before int erest and tax 15 56
× 100 = 6% × 100 = 29%
Share capital and reserves  Long term debt capital 246 190
Asset turnover =
Sales 2,160 1,806
×100 = 8.8 times = 9.5 times
Share capital and reserv es  Long  term debt capital 246 190
Liquidity ratios:
Current ratio =
Current assets 422 265
= 1.7 times = 1.8 times
Current liabilitie
s 254 147
Quick ratio =
Current assets excludinginventor 422 106 265 61
=1.2 times = 1.4 times
Current liabilitie
s 254 147

Working capital ratios:


Average time to collect =

Page 55 of 58
229
Trade receivables 316 365 198 365
× 365 =53 days = 40 days
Sales 2,160 1,806

Average time to pay =


Trade payables 198 365 142 365
× 365 = 41 days = 36 days
Cost of purchases 1,755 1,444
(Inventory turnover =
Inventory 106 365 61 365
× 365 = 22 days = 15 days
Cost of Sales 1,755 1,444

Note 1: averages are only calculated if it can be for all relevant years.
Note 2: cost of sales are considered in payment period because purchases for all relevant periods cannot be
calculated.

A.2 AMIR AND MO


Profitability ratios:
Amir Mo
Gross profit % =
Gross profit 90,000 490,000
× 100 ×100 = 60% ×100 = 70%
Sales 150,000 700,000
Net profit % =
Net profit 44,895 270,830
× 100 ×100 = 30% ×100 = 39%
Sales 150,000 700,000
Return on capital employed =
Pr ofit before int erest and tax
Share capital and reserves  Long term debt capital
61,500  500
Amir × 100 = 28.5%
207,395  10,000
371,000  12,000
Mo × 100 = 47%
565,580  250,000
Asset turnover =
Sales
×100
Share capital and reserv es  Long  term debt capital

150,000
Amir = 0.7 times
207,395  10,000
700,000
Mo = 0.85 times
565,580  250,000
Liquidity ratios:
Amir Mo
Page 56 of 58
230
Current ratio =
Current assets 50,000 153,250
= 2.2 times = 1.3 times
Current liabilitie
s 22,605 117,670
Quick ratio =
50,000  12,000 153,250  126250
Current assets excludinginventor =1.7 =
22,605 117,670
Current liabilitie
s
times 1.1 times
Working capital ratios:
Average time to collect =
37,500 105,000
Trade receivables ×365=91 ×365=55
× 365 150,000 700,000
Sales
days days
Average time to pay =
22,605 117,670
Trade payables ×365=137 ×365=204
× 365 60,000 210,000
Cost of purchases
days days
Inventory turnover =
26,250
Inventory 12,000 ×365=46
× 365 ×365=73 days 210,000
Cost of Sales 60,000
days

Note 1: averages are only calculated if it can be for all relevant years.
Note 2: cost of sales are considered in payment period because purchases for all relevant periods cannot be
calculated.
A.3 Alpha Limited and Omega Limited
(a)

(i) Current ratio Alpha Limited Omega Limited


Current assets 620,000 504,000
= 2.58:1 = 1.15:1
Current Liabilitie
s 240,000 440,000
(ii) Acid test
Current assets  stock 340,000 * 332,000 *
= 1.42:1 = 0.75:1
Current Liabilitie
s 240,000 440,000

*(620-280) *(504-172)
(iii) Creditors ratio
Average Creditors 180,000 344,000
× 365 ×365=59 days ×365=94 days
Purchases or cos t of sale 1,120,000 1,342,000
(iv) Collection period/Receivables ratio
Average debtors 310,000 300,000
× 365 ×365=79 days ×365=64 days
Sales 1,440,000 1,720,000

Page 57 of 58
231
(b) Comments on comparative analysis of the two companies based on the ratio computed above:
(i) In terms of working capital and liquidity, Alpha Limited is in a better position to honour its obligations
as they fall due because its current ratio and acid test ratio are higher than those of Omega Limited.
(ii) Omega Limited’s payment period is better than that of Alpha Limited’s because Omega Limited uses
supplier’s funds to finance its operation. However final analysis should be on the basis of credit period
available.
(iii) Omega Limited’s collection period is also better than that of Alpha Limited. It extends shorter credit
period to its customers than Alpha Limited. However final analysis should be on the basis of credit
period available.
(iv) Omega Limited’s credit policy is better than that of Alpha Limited. This because there is 30 days (94-
64) difference between its payments period and collection periods compared with Alpha Limited that
has a longer collection period than its payment period.

Page 58 of 58
232
Accounting Policies changes in Accounting Estimates and Errors
[IAS 8]

Accounting Polices: These are principles, rules and regulations adopted by management of an entity in preparing
and presenting financial statements. These rules and regulations are specified in IFRS.

Examples of IFRS where we have discussed accounting treatments are IAS-2, IAS-16, IAS-23 etc.

 As per IAS-8, accounting policies should be selected by applying the requirements of IFRS. E.g FIFO or
Weighted Average for inventory measurement.
 As per IAS-8, once a policy is selected it should be applied consistently to achieve comparability of figures over
a period of time.

Changes in Accounting Policies: An entity shall change its accounting policy only if:

a) The change is required by an IFRS (compulsory change); or


b) Management feels that change results in financial statements providing more relevant and reliable
information (voluntary change).

Prospective change: means effect of change is applied in current and future periods. No effect of change is
applied in previous periods.

Retrospective change: means effect of change is applied in current, future as well as in all previous periods.

Transitional provisions: sometimes if there is a change in an accounting policy in an accounting standard,


standard itself explains how to change the policy (means accounting treatment of change is given in the relevant
standard). That given accounting treatment in an accounting standard is called as transitional provisions.

Impracticable: means not possible to apply after making every reasonable effort to do so, e.g records related to
all relevant previous periods are not available.

How to adjust the Financial statements in case of Change in Accounting Policy:

a) If change is required by IFRS then use transitional provisions (if available) otherwise apply the change
retrospectively, unless it is impracticable to do so. If it is impracticable to apply retrospectively, then apply the
change from the date it is practicable to change.
b) If it is a Voluntary Change then apply the change retrospectively unless it is impracticable to do so. If it is
impracticable to apply retrospectively, then apply the change from the date it is practicable to change.

Page 1 of 43
233
Example
Change in accounting policy
During 2003, a revised IFRS on borrowing costs (IAS 23) was published. The company had previously been
expensing borrowing costs as a period cost, but the revised IFRS required that all borrowing costs be capitalized to
the related asset. The borrowing costs were all incurred on construction of a plant. The revised IFRS provided
transitional provisions that allowed the company to capitalize the costs from years beginning on or after 2004, or
before this date, if preferred. This entity chose to capitalize the borrowing costs from the earliest date possible.
The plant is not yet available for use. The effect of this change is as follows:

2001 2002 2003


Interest expense
Old policy 15 000 17 000 9 000
New policy 0 0 0

The following drafts were produced before adjusting for the change in accounting policy:

Draft statement of comprehensive income 2003 2002


For the year ended 31 December 2003 (extracts)
Profit for the year 455 000 380 000
Retained earnings at the beginning of the year 500 000 120 000
Retained earnings at the end of the year 955 000 500 000

Draft statement of financial position 2003 2002 2001


As at 31 December 2003 (extracts)
ASSETS

Plant 500 000 450 000 300 000


EQUITY AND LIABILITIES
Retained earnings 955 000 500 000 120 000

Required: Prepare the followings


a) Calculate the effect of change in policy
b) Statement of Comprehensive income (extracts) for the year ended 31-12-2003.
c) Statement of changes in equity (extracts) for the year ended 31-12-2003.
d) Necessary journal entries for the year ended 31-12-2003.
e) Statement of Financial Position (extracts) as on 31-12-2003
f) Notes to the financial statements for the year ended 31-12-2003.

Page 2 of 43
234
Solution
a) Effect of change in accounting policy
Decrease in Interest Expense Increase in Profit for the year
2001 15,000 15,000
2002 17,000 17,000
2003 9,000 9,000

b) Company Name
Statement of Comprehensive Income (Extracts)
For the year ended 31-12-2003
Restated
2003 2002
Profit for the year 464,000 397,000
2003 (455,000+9,000)
2002 (380,000+17,000)

c) Company Name
Statements of Changes in Equity (Extracts)
For the Year ended 31-12-2003
Retained Earnings
Balance as on 1-1-2002 120,000
Effect of change in policy (a) above 15,000
Balance as on 1-1-2002-Restated 135,000
Profit for the year 2002-Restated 397,000
Balance as on 31-12-2002-Restated 532,000
Profit after tax 2003 464,000
Balance as on 31-12-2003 996,000

Restated has not been mentioned with figures of 2003, because these figures are reported for the first time
to users of financial statements.

d) Accounting Entries of change in policy during the year ended 31-12-2003


2001 Plant 15,000
Interest Expense 15,000

2002 Plant 17,000


Interest Expense 17,000

2003 Plant 9,000


Interest Expense 9,000

However, effects of incomes and expenses related to previous periods should be adjusted in opening balance of
retained earnings therefore entries related to 2001 and 2002 should be recorded as follows:

2001 Plant 15,000


Retained Earnings 15,000

2002 Plant 17,000


Retained Earnings 17,000

Page 3 of 43
235
Instead of above two separate entries we should record a net effect of above two entries at the start of current
year as follows:
1-1-2003 Plant (15,000+17,000) 32,000
Retained Earnings 32,000
(15,000+17,000)

In exam questions if entries are required then effect of previous period’s incomes and expenses should be
adjusted in opening balance of retained earnings while current period’s effect of incomes and expenses should
be adjusted against current period incomes and expenses. Therefore following journal entries should be
prepared in exam situation
1-1-2003 Plant (15,000+17,000) 32,000
Retained Earnings 32,000
(15,000+17,000)

31.12.2003 Plant 9,000


Interest Expense 9,000
e) Company name
Statement of financial position (extracts)
As at 31 December 2003
Restated Restated
2003 2002 2001

ASSETS

Plant (500 000+15 000+17 000+9 000) 541 000 482 000 315 000
(450 000 + 15 000+17 000)
(300 000 + 15 000)

LIABILITIES AND EQUITY


Retained earnings (statement of changes in equity) 996 000 532 000 135 000

Disclosure of change in accounting policy:

a) Nature of change in accounting policy.


b) Reason of change in policy.
c) Effect on each *line items of statement of financial position and statement of comprehensive income.

*Line items means main headings of statement of financial position and statement of comprehensive income.

f) Company name
Notes to the financial statements (extracts)
For the year ended 31 December 2003
Accounting policies
Borrowing costs
Borrowing costs are capitalized to qualifying assets. This represents a change in accounting policy.
Change in accounting policy
The company changed its accounting policy from expensing borrowing costs as they are incurred to capitalizing
borrowing costs to plant, a qualifying asset.

Page 4 of 43
236
The change was made to comply with the revised IAS 23 Borrowing costs issued during the year. The effect of the
change in accounting policy is as follows:
2003 2002

Effect on the statement of comprehensive income


Decrease in Finance costs 9 000 17 000
Increase in profit for the year 9 000 17 000

2003 2002 2001


Effect on the statement of financial position

Increase in Plant 41 000 32 000 15 000


(15+17+9) (15+17)
41 000 32 000 15 000
Increase in Retained earnings
(10500+11900+6300) (10,500+11,900)

Prior period Errors is an error made in financial statements of one or more prior periods but discovered in current
period after the publication of financial statements of those periods. For example in 2015 we have discovered that
sale was overstated in 2001.

Prior period errors are corrected retrospectively.

Errors that occurred in current period are simply corrected in current period (with no disclosure requirements).

Disclosures of prior period error

a) Nature of prior period error


b) Effect on each line items of statement of financial position and statement of comprehensive income

Q.1 Mohani Manufacturing Limited is engaged in manufacturing of spare parts for motor car assemblers. The
audited financial statements for the year ended December 31, 2007 disclosed that the profit and retained
earnings were Rs. 21 million and Rs. 89 million respectively. The draft financial statements for the year
show a profit of Rs. 15 million. However, following adjustments are required to be made:
(i) The management of the company has decided to change the method for valuation of raw materials from FIFO to
weighted average. The value of inventory under each method is as follows:

FIFO Weighted Average


Rupees in million
December 31, 2006 37.0 35.5
December 31, 2007 42.3 44.5
December 31, 2008 58.4 54.4
(ii) In 2007, the company purchased a plant for Rs. 100 million. Depreciation on plant was recorded at Rs. 25
million instead of Rs. 10 million. This error was discovered after the publication of financial statements for the
year ended December 31, 2007. The error is considered to be material.
Required:
Produce an extract showing the movement in retained earnings, as would appear in the statement of changes in
equity for the year ended December 31, 2008. [QB# 11.3]
Note: If question is silent regarding tax then ignore the effects of taxation.

Points to remember

 Closing stock has direct relationship with profit

Page 5 of 43
237
 Opening stock has inverse relationship with profit
 Closing stock of previous period becomes opening stock of next period
 If there is no information of dividend then ignore dividend
 Even if question is silent but information is available then always prepare at least one year comparative
figures in statement of changes in equity as well.

Change in Accounting Estimates

Examples of accounting estimates are allowance for doubtful debts, useful life, depreciation methods or residual
value of depreciable assets and warranty obligations as per IAS-37 etc.

Accounting estimate are always changed prospectively.


Prospective change: means effect of change is applied in current and future periods. No effect of change is applied
in previous periods.

Whenever there is change in accounting estimates of useful life or residual value or there is a change in method of
depreciation then the WDV of the asset at the time of change is depreciated over the future periods. Depreciation
of previous periods is not adjusted.

Disclosures:

a) Nature of change in accounting estimate.


b) Financial effect of change in estimate in current and future periods on profit.

Q.2 CM is the managing director of Cl Limited, a company specializing in solving murders. Cl Limited owns a
large amount of very costly forensic equipment.
 The forensic equipment was purchased on 1 April 2001, at a cost of Rs 600 000.
 At that time it was determined that the equipment would be depreciated on a straight line basis over a
period of 6 years.
 On 1 April 2003, the total life of the forensic equipment was re-estimated to be 10 years. Cl Limited decided
to adjust their records accordingly (using the re-allocation method) (means prospectively).
 The tax rate has remained 30% since the company’s inception.

Required:
Show the depreciation journal entries necessary from the information provided above, assuming:
i. Depreciation had not yet been processed for the year ended 31 March 20X4;
ii. Depreciation based on the old estimate had already been processed for the year ended 31 March 20X4.

Q.3 Wonder Limited (WL) is engaged in the manufacturing and sale of textile machinery. Following are the
draft extracts of the statement of financial position and the income statement for the year ended 30 June 2012:

Statement of Financial Position


2012 2011
Rs. in million
Property, plant and equipment 189 130

Retained earnings 198 108

Page 6 of 43
238
Statement of profit or loss
2012 2011
Rs. in million
Profit for the year 90 120

Following additional information has not been taken into account in the preparation of the above financial
statements:
(i) Cost of repairs amounting to Rs. 20 million was erroneously debited to the machinery account on 1 October
2010. The estimated useful life of the machine is 10 years.
(ii) On 1 July 2011, WL reviewed the estimated useful life of its plant and revised it from 5 years to 8 years. The
plant was purchased on 1 July 2010 at a cost of Rs. 70 million.
Depreciation is provided under the straight line method.

Required:
Prepare relevant extracts (including comparative figures) for the year ended 30 June 2012 related to the following:
(a) Statement of financial position
(b) Statement of profit or loss
(c) Statement of changes in equity
(d) Correction of error note*
*correction of error note means disclosure.

Q.4 The following information has been extracted from the financial statements of Fine Fiber Limited (FFL) for
the year ended 30 June 2013:
2013 2012
Rs. in million
Profit for the year 82 93
Share capital (Rs. 10 each) 30 30
Retained earnings at the end of the year 186 105
Subsequent to preparation of the draft financial statements, an error has been detected in the financial
statements for the year ended 30 June 2012 whereby the accounting depreciation on an assembly plant was
mistakenly accounted for at Rs. 21.8 million instead of Rs. 12.8 million.

Other relevant information is as under:


i. The assembly plant was installed on 1 July 2010 at a cost of Rs. 80 million and is depreciated at 20% per
annum using the diminishing balance method.
ii. Final cash dividend for the year ended 30 June 2012 was approved at the Annual General Meeting held on 25
September 2012 at Rs. 4 per share (2011: Rs. 5 per share).

Required:
Prepare the following extracts (including comparative figures) from FFL’s financial statements for the year ended
30 June 2013 in accordance with the International Financial Reporting Standards:
a) Retained earnings column as would appear in the statement of changes in equity; and
b) Correction of error note.

Q.5 Autumn Ltd manufactures textile machines for the gaming industry. On 1 January 2001. Autumn Ltd
bought the rights to manufacture textile machines at a cost of Rs 600 000. The cost was recognized as an
intangible asset and was amortized over its expected finite life of 20 years to a nil residual value using the
straight-line method.

Page 7 of 43
239
During 2004 it was discovered, however, that Autumn Ltd had purchased the legal rights for a period of only 15
years. The effect of this information is considered to be material. The tax authority allows Autumn Ltd to deduct
the cost of the rights over a period of 10 years on the straight-line method.

The following is an extract of the draft statement of changes in equity for the year ended 31 December 2004
before making any necessary corrections.
AUTUMN LIMITED
STATEMENT OF CHANGES IN EQUITY
FOR THE YEAR ENDED 31 DECEMBER 2004 Retained Earnings

Balance: 1 January 2003 800000


Total comprehensive income: 2003 270 000
Balance: 1 January 2004 1070 000
Total comprehensive income: 2004 370 000
Balance: 31 December 2004 1 440 000

Required:
a) Show the correcting journal entries.
b) Disclose the following in the financial statements of Autumn Ltd for the year ended 31 December 2004 in
accordance with International Financial Reporting Standards:
i. Correction of error note:
ii. Statement of changes in equity; and
iii. Statement of financial position.

Before next question:


Depreciation by number of units produced
Depreciation is calculated by expressing the useful life of an asset in terms of its expected total output and
allocating the annual charge to depreciation based on actual output.
Formula:
Rate/Unit = Cost – Residual value
Total expected production
Example
A machine cost Rs 500,000.
It is expected to produce 5,000,000 units over its useful life.
47,850 units were produced in first year of production.
The depreciation charge for the first year of operation is
Depreciation = 500,000 x 47,850 = 4,785
5,000,000
Impairment loss: if carrying amount is more than recoverable amount, difference is called as impairment loss.
Recoverable amount is higher of:
 Fair value less cost to sell
 Value in use.
Q.6 Zain Pharma Limited (ZPL) owns patents of branded products A and B. ZPL uses cost model to account for
its intangible assets. It is policy of the company to amortize the cost on the basis of total estimated
revenue earned over the life of the patents. The following information is available:

Product 'A' Product 'B'


Date of acquisition of the patents 1 -Jan-2011 l-Jul-1998
Patents’ life 15 years 18 years
Rs. in million
Acquisition cost of the patents 37.50 36.00

Page 8 of 43
240
Sales upto 31-Dec-2012 800.00 11,200.00
Sales for the year ended 31-Dec-2013 600.00 1,600.00
Future estimated sales 6,100.00 3,200.00
Recoverable amount of the patents as at 31-Dec-2013 35.00 5.00

However, in 2013 the management identified that upto the year ended 31 December 2012, the cost of patents had
erroneously been amortized on the basis of estimated useful life.

Required:
Prepare accounting entries relating to the patents, for the year ended 31 December 2013 in accordance with the
International Financial Reporting Standards.

Summary of discussion about dividend


It is a distribution of profits to owners (shareholder). If dividend Is given as a percentage then multiply the rate
with the amount of share capital (nominal value of share capital) which is at the date of declaration of dividend to
calculate the amount of dividend. A company may pay dividend either in cash or in form of shares.
Dividend
Cash Dividend (Either Final or Interim) Bonus Shares (Either Final or Interim)
When Declared Dividend XXX
Dividend(Retained Earnings) XXX Share Capital A/c XXX
Dividend Payable A/c XXX

When Paid
Dividend Payable A/c XXX
Cash/ Bank A/c XXX

 Bonus shares are distributed from share premium and if the balance of premium is not available then from
retained earnings.
 Dividend is recognized on the date of declaration.
 If the dividend is declared after reporting date but before the authorization of the financial statements it is
disclosed in the financial statement to which it relates.
 Dividend is adjusted against retained earnings in the statement of changes in equity,
 If the date of declaration is not given, then:
1. For interim dividend assume that dividend is declared during the accounting period.
2. For final dividend assume declared after the reporting date.
 If nature of dividend (means whether final or interim) is not given then assume final dividend.
 If type of dividend (means cash or bonus) is not available then assume cash dividend.

Page 9 of 43
241
Q.7 SCCL, a company listed on Karachi and Lahore stock exchange is in process of finalization of its accounts
for the year ended 31-12-2012. The following information is available

i. Shareholders equity as at 31-12-2011 and 2010 consist of


Rs in million
2011 2010
Share capital (Rs 10 each) 10,340 7,833
Unappropriated Profits 6,945 4,508

ii. Profit for the year for the years ended 31-12-2010, 2011 and 2012 (unaudited) was Rs 4,240
million, Rs 4,944 million and Rs 5,090 million respectively.
iii. During the year ended 31-12-2012 it was discovered that machinery purchased on 1-7-2011 for
Rs 35 million was erroneously debited to repair account. SCCL depreciates machinery at the rate
of 20% per annum on reducing balance method No adjustment has been made in respect of this
material error.
iv. Cash dividends and bonus declared/paid during the last three years were as follows:
Cash Dividend Bonus Shares
*Interim Final *Interim Final
For the year ended 31-12-2010 10% - - 20%
For the year ended 31-12-2011 - 15% 10% 10%
For the year ended 31-12-2012 - 10% 5% 5%

*Interim dividend/bonus was declared at time of announcement of half-yearly financial


results.

v. Right shares were issued on 30-11-2012 in the ratio of 4 right shares for every 5 shares held by
the shareholders of the company. The right issue was made at Rs 18/share.

Required: Prepare statement of changes in equity for the year ended 31-12-2012.
Note: If more than one dividend is declared on the same date then for both dividends base share capital is same.

Q.8 Oranges Ltd is a company operating in the manufacturing industry. The following draft extracts of the
statement of comprehensive income and statement of financial position have been presented to you
together with additional information that has not yet been taken into account in the preparation thereof:

ORANGES LTD
DRAFT EXTRACTS FROM STATEMENT OF COMPREHENSIVE INCOME
FOR THE YEAR ENDED 31 DECEMBER 2003
2003 2002

Profit for the period 300 000 250 000


ORANGES LTD
DRAFT EXTRACTS STATEMENT OF FINANCIAL POSITION
AT 31 DECEMBER 2003

2003 2002 2001


Property, plant and equipment 1,217,000 1,391,500 1.566,000
- Machinery carrying amount 152,000 176,500 201,000
- Equipment carrying amount 1,065,000 1,215,000 1,365,000

Page 10 of 43
242
Retained Earnings ? ? 1,000,000

Additional information:
Machinery:
During 2003, the company changed the estimated residual value of its machinery from Rs 5,000 to Rs 10,000 and
changed the total expected useful life of machinery from 10 years to 15 years. The machinery had all been
purchased on 1 January 20X0 at a cost of Rs 250,000. Depreciation is provided on the straight line method.

Equipment:
a) During 2003, it was discovered that the cost of an item of inventory sold on 1 July 2001 (cost: Rs 300,000),
had been incorrectly debited to equipment. The taxable profit and tax base were correctly calculated in all
years affected.
b) The cost of equipment was otherwise Rs 1,200,000, all having been purchased on l January 2001.
c) The company depreciates equipment at 10% per annum to nil residual values (apportioned for part of a
year where appropriate) using the straight-line basis.

Other general information:


a) The opening retained earnings as at 1 January 2002 was Rs 1,000,000. There were no dividend
declarations or transfers to or from retained earnings during 2002 and 2003.
b) All amounts are considered to be material.

Required:
a) Calculate the effect of the change in estimate.
b) Show the journal entries that would need to be processed.(Assuming all entries to be made in 2003)
c) i) Prepare statement of comprehensive income for the year ended 31-12-2003.
ii) Column of retained earnings in statement of changes in equity for the year ended 31-12-2003, and
iii) Statement of financial position as on 31-12-2003.
d) Prepare note of change in accounting estimate and error correction.

Page 11 of 43
243
Self-Practice Questions
Q.1 Question of Change in Accounting Estimate and its relevant disclosures:
On 1 January 2003 M Limited purchased a plant for Rs 500,000. The company used the reducing balance
method for calculating depreciation at a rate of 20% per annum. The tax authority grants a 40:20:20:20
allowance over four years.
During 20X6 the directors decided it was necessary to change to the straight line method of calculating
depreciation for plant. It was agreed that the remaining estimated life of the plant was 3 years and the
estimated residual value Rs 16,000 (previously this was nil).
This is considered to be a change in accounting estimate as per IAS-16.
M accounts for changes in estimates using the re-allocation method (means prospectively).
The draft statement of comprehensive income and statement of changes in equity had been prepared for
the year ended 31 December by the bookkeeper who had not been advised of the director’s decision.

M LIMITED
DRAFT STATEMENT OF COMPREHENSIVE INCOME
FOR THE YEAR ENDED 31 DECEMBER 2006
2006 2005
Revenue 800,000 650,000

Profit before depreciation 380,200 300.000


Depreciation – plant (51,200) (64,000)
Profit for the year 329,000 236,000

M LIMITED
DRAFT STATEMENT OF CHANGES IN EQUITY
FOR THE YEAR ENDED 31 DECEMBER 2006
Retained earnings

Balance: l January 2005 (40,000)


Profit for the period 236,000
Dividends (10,000)
Balance: 31 December 2005 186,000
Profit for the period 329,000
Dividends (15,000)
Balance: 31 December 2006 500,000

Required:
a) Prepare the statement of comprehensive income (extracts) and relevant extracts from the statement of
changes in equity of M Limited for the year ended 31 December 2006, in compliance with International
Financial Reporting Standards.
b) Notes to the financial statements related to change in accounting estimates.
Comparatives are required.

Page 12 of 43
244
Q.2
Chand Paints Limited (CPL) is engaged in the manufacturing of chemicals and paints. In April 2016 it was discovered
that certain errors had been made in the financial statements for the year ended 30 June 2015. The errors were
corrected in 2016. The details are as follows:

2015 After
2016 (Draft) Correction of 2015 Audited
Errors
----------- Rs. In million ----------
Statement of comprehensive income
Sales tax, commission and discounts (7,939) (8,246) (7,916)
Cost of sales (45,508) (44,606) (44,633)
Selling and distribution expenses (2,940) (2,635) (2,441)
Administration expenses (2,356) (2,254) (2,149)
Other operating charges (495) (467) (515)
Other operating income 920 427 509
Profit for the year 4,089 3,723 4,359
Statement of financial position
Trade and other receivables 1,839 1,613 2,025
Trade and other payables 11,600 8,894 8,670

The share capital and un-appropriated profit of CPL as on 1 July 2014 was Rs. 10,400 million and Rs. 19,089 million
respectively.
The details of dividend declared are as follows:

2016 2015
Cash dividend – Interim 10% 5%
– Final 15% 10%

Required:
(a) Prepare a correction of error note to be included in the financial statements for the year ended 30 June
2016. (Ignore earnings per share and taxation) (10)
(b) Prepare the statement of changes in equity for the year ended 30 June 2016. (08)

Q. 3
The following information has been taken from the financial statements of Asif engineering Limited (AEL) for the
year ended 31 December 2015:

2015 (draft) 2014 2013


--------------- Rs. In million ---------------
Property, plant equipment 2,430 2,402 2,105
Stores and spares 73 80 70
Retained earnings as at 31 December 353 224 101
Net profit 129 123 112

Page 13 of 43
245
In the above financial statements, AEL has recognised consumption of spare parts as expense. AEL has now
decided to change its above policy and classify consumption of spares having useful life of more than one year as
capital spares under property, plant and equipment.
Following information pertains to capital spares consumed during the past there years:

Parts issued during the year Rs. In


Year ended Useful life of the issued parts
million
31 December 2013 55 5 years
31 December 2014 39 3 years
31 December 2015 44 4 years

Depreciation on these parts is to be charged using straight line method over its useful life.

Required:
In accordance with the requirements of International Financial Reporting Standards, prepare the revised extracts
(including comparative figures) of the following:
(a) Statement of financial position as at 31 December 2015. (04)
(b) Statement of comprehensive income for the year ended 31 December 2015. (03)
(c) statement of changes in equity for the year ended 31 December 2015. (03)
(Ignore taxation)

Q. 4
The following information pertains to draft financial statements of Pak Ocean Limited (POL) for the year ended 31
December 2014.
(i)
2014 2013
Rs. In million
Profit after tax 90 47
Incremental depreciation on revaluation of property, plant and
equipment (means transfer of Revaluation surplus) 1.5 2.3

(ii) Installation of an assembly plant was completed in December 2012 at a cost of Rs. 60 million and it was
ready for use on 1 February 2013. However, depreciation for the year ended 31 December 2013 amounting
to Rs. 4.5 million was worked out from the date of production i.e. 1 April 2013. the mistake was corrected
by adjusting the profit and loss account for the year ended 31 December 2014.
(iii) Shareholders’ equity as at 1 January 2013 was as follows:

Rs. In million
Share Capital (Rs. 100 Each) 200
Retained Earning 45
Revaluation surplus 100

On 30 November 2014, POL issued 25% right shares to its ordinary shareholders at Rs. 120 per share.

(iv) Cash dividend and bonuses declared/paid during the last three years:

Page 14 of 43
246
Final *Interim
For the year ended
Cash Bonus Cash Bonus
31 December 2012 - 15% 16% -
31 December 2013 18% - 20% -
31 December 2014 - 25% - 10%

*Declared with half yearly accounts

Required:
Prepare Statement of Changes in Equity for the year ended 31 December 2014 in accordance with the
requirements of the Companies Act 2017 and International Financial Reporting Standards. (15)
(Ignore taxation)

Q.5 For the purpose of preparation of statement of changes in equity for the year ended 31 December 2017,
Daffodil Limited (DL) has extracted the following information:

2017 2016 2015


Draft Audited Audited
--------- Rs. in million ---------
Net profit 650 318 214
Transfer to general reserves 112 - 141
Transfer of incremental depreciation - 49 55
Final cash dividend - - 7.5%

Additional information:
1. Details of share issues:
i. 25% right shares were issued on 1 May 2016 at Rs. 18 per share. The market
price per share immediately before the entitlement date was also Rs. 18 per
share.
ii. A bonus issue of 10% was made on 1 April 2017 as final dividend for 2016.
iii. 50 million right shares were issued on 1 July 2017 at Rs. 15 per share. The market
price per share immediately before the entitlement date was Rs. 25 per share.
iv. A bonus issue of 15% was made on 1 September 2017 as interim dividend.

2. After preparing draft financial statements, it was discovered that depreciation on a plant costing
Rs. 700 million has been charged @ 25% under reducing balance method, from the date of
commencement of manufacturing i.e. 1 July 2014. However, the plant was available for use on 1
February 2014.

3. Share capital and reserves as at 31 December:


2015 2014
------ Rs. in million ------
Ordinary share capital (Rs. 10 each) 1,600 1,600
General reserves 1,850 1,709
Retained earnings 1,430 1,302

Page 15 of 43
247
Required:

Prepare DL’s statement of changes in equity for the year ended 31 December 2017 along
with comparative figures. (Ignore taxation) (14)

Page 16 of 43
248
Solutions
A.1
Mohani Manufacturing Limited
Statement of Changes in Equity (Extracts)
For the year ended 31-12-2008

Retained Earning
Balance as on 1-1-2007 (89-21) 68
Effect of Change in Policy (1.5)
Balance as on 1-1-2007 Restated 66.5
Profit – 2007 as restated 39.7
Balance as on 31-12-2007-Restated 106.2
Profit – 2008 8.8
Balance as on 31-12-2008 115

Workings:

W-1 Effect of correction of error on profit:


2008 2007 2006
Opening Stock (2.2) 1.5 -
Closing Stock (4) 2.2 (1.5)

(6.2) 3.7 (1.5)

W-2
Statement of Comprehensive Income (Extracts)
For the year ended 31-12-2008
2008 2007
Profit as Given 15 21
Effect of error in depreciation 0 15
Effect of policy change (W-1) (6.2) 3.7
Profit as restated 8.8 39.7

A.2 a)

i) Depreciation journal: Debit Credit

Depreciation 50 000
Accumulated depreciation: equipment 50 000
Depreciation of equipment

ii) Adjustment to depreciation journal:

Accumulated depreciation: equipment 50 000

Depreciation 50 000
Adjustment to depreciation of equipment

Page 17 of 43
249
Working
Date of purchase 1-4-2001
Cost of purchase Rs 600,000
Depreciation upto 31-3-2003 is 600,000/6 x 2 = Rs 200,000
WDV= 600,000 – 200,000/8 = 50,000

A.3 Solution

Wonder Limited
a) Extracts of Statement of financial position
For the year ended 30 June 2012
2012 2011
(Restated)
Rs. in million
Property, plant and equipment 178.50 111.50
Retained earnings-SOCE 187.5 89.50
PPE: Year2012:[189 – 18 . 5+2+6] PPE: Year 2011:(130 -18.5)

b) Wonder Limited
Extracts of Statement of profit or loss
For the year ended 30 June 2012
2012 2011
(Restated)
Rs. in million
Profit for the year 98.00 101.50
Profit for the year: Year 2012: [90+2+6]
Profit for the year: Year 2011: [120-18.5]

c) Wonder Limited
Extracts of statement of changes in equity
For the year ended 30 June 2012
Retained earnings
Rs. in million
Balance as on 1 July 2010 (108-78) 30.00
Profit for the year ended 30 June 2011-restated 59.50
Balance as at 30 June 2011 - restated 89.50
Profit for the year ended 30 June 2012 98.00
Balance as at 30-June 2012 187.5

d) Wonder Limited
Notes to the financial statements
For the year ended 31 December 2012
Correction of error

Page 18 of 43
250
During the year ended 30 June 2011, the repair works was erroneously debited to machinery account. The effect
of this error is as follows:
2011
Rs. in million
Effect on the income statement
Increase in Repairs and maintenance (20.00)
Decrease in Depreciation 1.50
Decrease in profit for the year (18.50)
Effect on the statement of financial position
Decrease in Property, plant and equipment (18.50)
Decrease in retained earnings (18.50)

Workings
Effect of error
1-10-2010 Repair capitalized to machinery 20M
During the year ended 30-6-2011

Decrease in Profit (20M) Repair 20M


Machinery 20M
Increase in Profit 1.5M Acc Depreciation 1.5M
(20M ÷ 10 x 9/12) Depreciation 1.5M
Net Decrease in Profit (18.5)

During the year ended 30-6-2012


Increase in Profit 2M Acc Depreciation 2M
Depreciation 2M
Net Increase in Profit 2M

Effect of changes in Estimates


During the year ended 30-6-2012
Cost 70M
Acc depreciation (70 ÷ 5) 14M
WDV as on 30-6-2011 56M
Depreciation to be charged
(56M÷7) 8M
Depreciation already charged 14M
Depreciation to be reversed 6M Acc Depreciation 6M
Depreciation 6M

A.4 Solution
Fine Fibre Limited

Page 19 of 43
251
a) Statement of Changes in Equity for the year ended 30 June 2013 (extracts)

Retained
Earnings

Rs, in million
Balance as at 1 July 2011 (W-1) 38.00
Final cash dividend for the year ended 30 June 2011 @ Rs. 5 per share (30÷ 10x5) (15.00)
Profit after tax for the year ended 30 June 2012 - restated (W. a) 91.00
Balance as at 30 June 2012 - restated 114.00
Final cash dividend for the year ended 30 June 2012 @ Rs. 4 per share (30÷10X4) (12.00)
Profit after tax for the year ended 30 June 2013 (W. a) 91.20
Balance as at 30 June 2013 193.20

W. a : Corrected profit after tax 2013 2012


Profit after tax before correction 93.00 82.00
Depreciation excess/(short) charged [W] (1.80) 9.00
91.20 91.00

(b) Fine Fiber Limited


Notes to the financial statements for the year ended 30 June 2013 (extracts)
1 Correction of error
During the year ended 30 June 2012, accounting depreciation was incorrectly accounted for at Rs. 21.8 million
instead of Rs. 12.8 million. The effect of the correction is as under:
2012
Rs. in million
Effect on the statement of comprehensive income
Decrease in depreciation expense 9.00
Increase in Profit for the year 9.00
Effect on the statement of financial position
Increase in property, plant and equipment 9.00
Increase in retained earnings (114-105) 9.00

Workings
1. Calculation of opening retained earnings as on 1-7-2011
Retained Earnings
Dividend (3x5) 15 b/d (Bal) 38
Profit after tax 82
(128-36-10)
c/d 105
Dividend (3x4) 12 b/d 105
Profit after tax 93
(140-39-8)
c/d 186

Page 20 of 43
252
2. Accounting adjustment in 2012
Acc Depreciation 9
Depreciation 9

3. Accounting adjustments in 2013:


In this period, accounting depreciation would have been less recorded because of less WDV at the
beginning of this period because of wrongly charging extra depreciation in previous period.
Depreciation 1.8
Acc Depreciation 1.8
(9 x 20%)

A.5 Solution

Working 2004 2003 2002 2001


Increase in Amortization 10,000 10,000 10,000 10,000
Decrease in profit for the 10,000 10,000 10,000 10,000
year

a)
Journals Debit Credit

2001
Amortization expense 10 000
Intangible asset: accumulated amortization 10 000
Correction of error: 2001

2002
Amortization expense 10 000
Intangible asset: accumulated amortization 10 000
Correction of error: 2002

2003
Amortization 10 000
Intangible asset: accumulated amortization 10 000
Correction of error: 2003

2004
Amortization 10 000
Intangible asset: accumulated amortization 10 000
Correction of error: 2004

These above entries are just for understanding, following entries should be prepared in an exam question
1 January 20X4
Retained earnings 30,000
Intangible asset: accumulated. Amortization (10,000 x 3) 30 000
Correction of error: years prior to 2004

31 December 20X4
Amortization 10 000
Intangible asset: accumulated amortization 10 000
Correction of error: effect on 2004

Page 21 of 43
253
Autumn Limited
b) Notes to the Financial Statements
For the year ended 31 Dec 2004

i)Correction of error note:


Rights to manufacture have been amortized over a period of 20 years instead of 15 years. The effect of
correction of this error is as follows

Effect on Statement of Comprehensive Income


2004 2003
Increase in amortization (10,000) (10,000)
Decrease in Profit for the year (10,000) (10,000)

Effect on Statement of Financial Position


Restated Restated
2004 2003 2002
Decrease in Intangible assets (40,000) (30,000) (20,000)
Decrease in Retained Earnings (40,000) (30,000) (20,000)

b ii)
Autumn Limited
Statement of Changes in Equity (Extracts)
For the year ended 31-12-2004
Retained Earnings
2004
Balance as on 1-1-2003 800,000
Effect of correction of error (10,000 x 2) (20,000)
Balance as on 1-1-2003 Restated 780,000

Profit for the year-Restated (270,000 – 10,000) 260,000

Balance as on 1-1-2004 Restated 1,040,000


Profit for the yaer-2004 (370,000 – 10,000) 360,000

Balance as on 31-12-2004 1,400,000

b iii)
Autumn Limited
Statement of Financial Position (Extracts)
As on 31-12-2004
Restated Restated
2004 2003 2002
Non-current Assets
Intangible assets 440,000 480,000 520,000

Equity & Liabilities:


Retained earnings 1,400,000 1,040,000 780,000

Page 22 of 43
254
A.6 Solution

Zain Pharma Limited


Accounting entries for patents

Debit Credit
Date Description
(Rs. in million)
1-1-2013 Accumulated amortization/Intangible assets W-4 4.80
Retained earnings W-4 4.80
(Amortization expense over charged for the period up to 31
December 2012)

31-12-2013 Amortization expense (P&L account) W-2 6.60


Acc. amortization/Intangible assets W-2
6.60
(Amortization expense for the year based on sales)

31-12-2013 Impairment (P&L account) W-3 2.20


Intangible assets/Acc Impairment loss 2.20
(Impairment of patent product B as at 31 December 2013)

Workings

W-1: Patents' amortization as recorded upto 31-12-2012 Total


(based on number of years) Product-A Product-B (Rs. in
million)
Date of purchase 1-Jan-2011 1-Jul-1998
Acquisition cost of patents A 37.50 36.00
Estimated life of the patents B 15 years 18 years
No. of years upto 31-12-2012 C 2 years 14.5 years
Amortization upto 31-12-2012 D (A ÷ B x C) 5.00 29.00

W-2: Patents’ amortization as corrected (based on sales)


Sales upto 31-12-2012 E 800.00 11,200.00
Sales for the year ended 31-12-2013 F 600.00 1,600.00
Future estimated sales 6,100.00 3,200.00
Total sales G 7,500.00 16,000.00
Amortization upto 31-12-2012 H (A ÷ G x E) 4.00 25.20
Amortization for the year ended 31-12-2013 J (A÷ G x F) 3.00 3.60 6.60
K (H+J) 7.00 28.80

W-3: Impairment as at 31-12-2013


Carrying value as at 31-12-2013 A-K 30.50 7.20
Recoverable amount as at 31-12-2013 35.00 5.00
Impairment loss - (2.20) (2.20)

W-4 : Amortization erroneously over booked (1.00) (3.80) (4.80)


(H-D) (5-4) (29-25.2)
upto 31-12-2012

Page 23 of 43
255
A.7

SCCL
Statement of Changes in Equity
For the Year ended 31-12-2012
Rs in millions
Share Share Unappropriated Total
Capital Premium Profit
Balance as on 1-1-2011 7,833 - 4,508 12,341

Final Bonus Shares-2010 (20%) 1,567 - (1,567) -


(7833x20%)
Interim Bonus Shares-2011(10%) 940 - (940) -
(7,833+1,567)x10%
Profit for the year –Restated (W) - - 4,975.50 4,975.50
Balance as on 31-12-2011 10,340 - 6,976.50 17,316.50

Final Bonus Shares-2011 (10%) 1034 (1034) -


(10,340x10%)
Final Cash Dividend-2011 (15%) (1,551) (1,551)
(10,340x15%)
Interim bonus Shares-2012(5%) 569 (569) -
(10,340+1034) x 5%
*Issue of Right Shares 9,554 7,643 - 17,197
Profit for the year (W) 5,083.70 5,083.70
Balance as on 31-12-2012 21,497 7,643 8,,906.20 38,046.2

*Right share=(10,340+1034+569)=11,943÷10 x 4/5 = 955.4 m shares


Share capital = 955.4 x 10 = 9,554
Share Premium = 955.4 x 8 =7,643
Workings
For 2012 2011
Machine 35
Repair 35
Depreciation 6.3 Depreciation 3.5
Machine 6.3 Machine 3.5 35x20%x6/12
(35-3.5)x20%

W-2 Effect on Profit for the year:

2012 2011
5,090 4,944
- 35
(6.3) (3.5)
5,083.70 4,975.50

Page 24 of 43
256
A.8
a) Effect of Change in Estimates:
Was Is Difference
Cost on 1-1-2000 250,000 -
Depreciation upto 31-12-2002 (73,500) -
(250,000-5000)÷10x3
Carrying Amount 176,500 176,500
Residual value (10,000)
Depreciable Amount 166,500
Remaining useful life 12
Depreciation for 2003 24,500 13,875 Depreciation to be
reversed Rs 10,625
Carrying Amount as on 31-12-2003 152,000 162,625

Future Depreciation
Original (152,000-5000) 147,000 Difference in future
Revised (162,625-10,000) 152,625 depreciation Rs 5,625 to be
disclosed.

b) Accounting Entries
Effect of correction of Error
2001 2002 2003
Increase in cost of sale (300,000)
Decrease in depreciation 15,000 30,000 30,000
Increase/Decrease in Profit for the year (285,000) 30,000 30,000

1-1-2003
Retained Earnings 255,000
(285,000-30,000)
Acc Depreciation (15+30) 45,000
Equipment 300,000
31-12-2003
Acc Depreciation 30,000
Depreciation 30,000

Change in Estimate (figures as in (a) above)


31-12-2003

Acc Depreciation 10,625


Depreciation 10,625

c)i) Oranges Limited


Statements of Comprehensive Income (Extracts)
For the year ended 31-12-2003
2003 2002
(Restated)
Profit for the year

Page 25 of 43
257
2003 (300,000 + 10,625 + 30,000) 340,625

2002 (250,000 + 30,000) 280,000

ii) Oranges Limited


Statements of Changes in Equity (Extracts)
For the year ended 31-12-2003
Retained Earning
Balance as on 1-1-2002 1,000,000
Effect of correction of Error [(b) above] (285,000)
Balance as on 1-1-2002-(Restated) 715,000
Total Comprehensive Income-2002-(Restated) 280,000
Balance as on 31-12-2002 (Restated) 995,000
Total Comprehensive Income 2003 340,625
Balance as on 31-12-2003 1,335,625

iii) Oranges Limited


Statement of Financial Position (Extracts)
As on 31-12-2003
Restated Restated
2003 2002 2001
Property, Plant & Equipment (W) 1,002,625 1,136,500 1,281,000
Retained Earnings (SOCE) 1,335,625 995,000 715,000

Workings
Plant & Machinery(2001)
Unadjusted 1,566,000 COS 300,000
Dep 15,000
C/D 1,281,000

Plant & Machinery(2002)


Unadjusted 1,391,500 COS 300,000
Dep 15,000
Dep 30,000
C/D 1,136,500

Plant & Machinery(2003)


Unadjusted 1,217,000 COS 300,000
Dep 15,000
Dep 30,000
Dep 30,000

10,625 C/D 1,002,625

Page 26 of 43
258
d) Oranges Ltd
Notes to the Financial Statements (Extracts)
For the year ended 31-12-2003

i) Change in Accounting Estimate


In 2003, the company changed the estimate of residual value of machinery from Rs 5,000 to Rs 10,000. In
addition it also changed its estimate of useful life from total 10 years to 15 years.
The effect of change in estimates is as follows.
Increase in current period profits 10,625
Decrease in future period profits 5,625

ii) Correction of Error:


During the year ended 31-12-2001, cost of inventory sold on 1-7-2001 for Rs 300,000 has been incorrectly
debited to equipment. The effect of corrections are as under:
Effect on Statement of Comprehensive Income
2002 2001
Increase in cost of sales - (300,000)
Decrease in depreciation 30,000 15,000
Increase/Decrease in Profit for the period 30,000 (285,000)
Effect on Statement of Financial Position
Increase/decrease in equipment (255,000) (285,000)
Increase/decrease in retained earnings (255,000) (285,000)

Page 27 of 43
259
Solution of Self-Practice Questions

A.1 Solution
a) M LIMITED
STATEMENT OF COMPREHENSIVE INCOME (Extracts)
FOR THE YEAR ENDED 31 DECEMBER 20X6
20X6 20X5

Revenue Given 800 000 650 000


Profit for the year (329 000 - 28 800) 300 200 236 000

M LIMITED
STATEMENT OF CHANGES IN EQUITY (Extracts)
FOR THE YEAR ENDED 31 DECEMBER 2006

Retained
Earnings

Balance 31/12/2004 (40 000)


Profit for the year: 2005 236 000
Dividends (10 000)

Balance 31/12/2005 186 000

Profit for the year: 2006 300 200


Dividends (15 000)

Balance 31/12/2006 471 200

Workings
W1 Change in estimate - re-allocation method
There are two estimates that have changed:
• The estimated residual value has been changed (from Rs 0 to Rs 16 000); and
• The estimated pattern of consumption of the asset/ method of depreciation (from reducing balance
method to straight-line method).

Date Calculations Was Is Difference


Cost 1/1/2003 500 000
Depreciation 2003 (500 000 x 0.20) (100 000)
Carrying amount 31/12/2003 400 000
Depreciation 2004 (400 000 x 0.20) (80 000)
Carrying amount 31/12/2004 320 000
Depreciation 2005 (320 000 x 0.20) (64 000)
Carrying amount 31/12/2005 256 000 256 000
Residual value 0 (16 000)
Depreciable amount 256 000 240 000
Rate or remaining useful life 20% 3 years
Depreciation 2006 (256 000 x 0.20) (240 000 / 3) (51 200) (80 000) (28 800)
increase in
depreciation

Page 28 of 43
260
Carrying amount 31/12/2006 204 800 176 000 (28 800)
Future depreciation Was: (204,800 – 0) 204,800 160,000 44,800
Is: (176,000 – 16,000) decrease in
depreciation

Accounting Entries
i) Depreciation 28,800
Acc Depreciation 28,800

b) M LIMITED
NOTES TO THE FINANCIAL STATEMENTS FOR THE YEAR ENDED 31 DECEMBER 20X6
1.Change in estimate
The company changed the method of estimating depreciation on plant from the reducing balance to the straight-
line method and changed the estimated residual value from Rs 0 to Rs 16 000.

The (increase)/ decrease caused by the change in estimate is as follows:


Figures from working 1
• Decrease in Current profits 28 800
• Increase in Future profits 44 800

A. 2(a)
Chand Paints Limited
Notes to the Financial Statements
For the year ended 30 June 2016
The effect of retrospective restatement on statement of comprehensive income is tabulated below:

2015
Increase / (decrease) in income
Rs. In million
Increase in sales tax, commission and discounts (7,916 – 8,246) (330)
Decrease in cost of sales (44,633 – 44,606) 27
Increase in selling and distribution expenses (2,441 – 2,635) (194)
Increase in administration expenses (2,149 – 2,254) (105)
Decrease in operating income (602)
Decrease in other operating charges (515 – 467) 48
Decrease in other operating income (509 – 427) (82)
Decrease in profit for the year (3,723 – 4,359) (636)

The effect of retrospective restatement on statement of financial position for 2015 is tabulated below:

Decrease in trade debts (2,025 – 1,613) (412)


Increase in trade and other payables (8,894 – 8,670) (224)
Decrease in un-appropriated profit (636)

Page 29 of 43
261
A. 2(b)
Chand Paints Limited
Statement of changes in equity
For the year ended 30 June 2016

Retained
Share Capital Total
Earnings
----------- Rs. In million ----------
Balance as on 1 July 2014 10,400 19,089 29,489
Interim dividend for the year ended 30 June 2015 (10,400 × 5) (520) (520)
Total comprehensive income for the year 2015 – restated 3,723 3,723
Balance as at 30 June 2015 - restated 10,400 22,292 32,692
Final dividend for the year ended 30 June 2015 (10,400 × 10%) (1,040) (1,040)
Interim cash dividend for the year 2016 (10,400 × 10%) (1,040) (1,040)
Total comprehensive income for the year ended 30 June 2016 4,089 4,089
Balance as at 30 June 2016 10,400 24,301 34,701

A. 3
(a) Asif Engineering Ltd.
Extracts from statement of financial position

2014 2013
2015
(Restated) (Restated)
-------------- Rs. In million ---------------
Property, plant & equipment (W-2) 2,498 2,461 2,149
Stores and spares – Given 73 80 70
Retained earnings 421 283 145

(b) Extracts from statement of comprehensive income

2015 2014 (Restated)


------------- Rs. In million ------------
Net profit (W-1) 138 138

(c) Extracts from statement of changes in equity

Retained earnings Rs.


In million
Balance as at 1 January 2014 101
Effect of retrospective change in accounting policy (W-1) 44
Balance at 1 January 2014 – restated 145
Total comprehensive income – 2014 (W-1) 138
Balance as at 1 January 2015 – (restated) 283

Page 30 of 43
262
Total comprehensive income – 2015 138
Balance as at 31 December 2015 421

(W-1) Computation of net profit


Depreciation expense for the year
2013 2014 2015
-------------- Rs. In million ---------------
Depreciation for 2013 (55÷5) 11 11 11
Depreciation for 2014 (39÷3) - 13 13
Depreciation for 2015 (44÷4) - - 11
11 24 35
Less: Amount already charged 55 39 44
Adjustment to be made in net profit 44 15 9
Profit for the year – Given 123 129
Adjusted profit for the year 138 138

(W-2) Property, plant and equipment

2013 2014 2015


-------------- Rs. In million ---------------
As given 2,105 2,402 2,430
Add: Stores issued 2013 55 55 55
Add: Stores issued 2014 0 39 39
Add: Stores issued 2015 0 0 44
Less: Accumulated depreciation as calculated above 2014:

11 + 24; 2015: 11 + 24 + 35 (11) (35) (70)


Revised carrying value 2,149 2,461 2,498

A.4
PAK OCEAN LIMITED
Statement of changes in Equity
For the year ended 31-12-2014
“Rs In Millions”
Share Share Retained Revaluation Total
Capital Premium Earnings Surplus
Balance as on 1-1-2013 200 - 45 100 345
Bonus Share-2012 (200 X 15%) 30 (30) -
Interim Cash dividend-2013 (230 X 20%) (46) (46)
Profit after tax-Restated -(W-1) 46 46
Transfer of Surplus 2.3 (2.3) -
Balance as on 1-1-2014 (-Restated) 230 - 17.3 97.7 345
Final cash dividend-2013 (230 X 18%) (41.4) (41.4)

Page 31 of 43
263
Interim bonus-2014(230 X 10%) 23 (23) -
Issue of right shares- (W-2) 63.25 12.65 - 75.90
Profit after tax 91 91
Transfer of surplus 1.5 (1.5)
Balance as on 31-12-2014 316.25 12.65 45.4 96.2 470.5

W-1 Prior period error


2014 2013
Profit After tax-Given 90 47
Effect of depreciation* 1 (1)
Profit after tax [adjusted] 91 46

4.5/9 X 2 = 1 (effect of depreciation of Feb & March 2013 to be reversed in 2014 and charged in 2013.

W-2) [230+23] /100 = 2.53 M shares X 25 % = 0.6325 M shares


0.6325M X 100 = 63.25
0.6325 M X 20 = 12.65
Ans.5 Daffodil Limited
Statement of changes in equity
For the year ended 31 December 2017
Ordinary Share General Retained Revaluation Total
Share
capital premium reserves earnings surplus
-------------------- Rs. in million --------------------
Balance as at 31
December 2015 (As
given) 1,600.00 1,850.00 1,430.00 4,880.00
Effect of correction of
error (W-1) (54.69) (54.69)
Balance as at 31
December 2015 –
Restated 1,600.00 1,850.00 1,375.31 4,825.31
Final cash dividend @
7.5% - 2015
(1,600×7.5%) (120.00) (120.00)
Right issue @ 25% 400.00 320.00 720.00
(1,600×25%) (160×25%×8)
Net profit - 2016 -
Restated[318+13.67(W-
1)] 331.67 331.67
Transfer of incremental
depreciation 49.00 (49.00) 49.00
Balance as at 31
December 2016 -
Restated 2,000.00 320.00 1,850.00 1,635.98 5,805.98
Final bonus dividend @
10% - 2016
(2,000×10%) 200.00 (200.00) ) -
Right issue 500.00 250.00 750.00
(50×10) (50×5)
Interim bonus dividend
@ 15% - 2017 405.00 (370) (35) -

Page 32 of 43
264
(2,700×15%)
Net profit - 2017 [650 +
10.25 (W-1)] 660.25 660.25
Transfer to general
reserves 112.00 (112.00) -
Balance as at 31
December 2017 3,105.00 - 1,962.00 2,149.22 7,216.23

W-1: Correction of error


Correct Wrong Increase/(decrease)
depreciation @ 25% depreciation @ 25% in depreciation
-------------------------- Rs. in million --------------------------

Cost 700 700


2014 160.42 87.50 72.92
(700 × 25% × 11 ÷ 12) (700 × 25% × 6 ÷ 12)
2015 134.90 153.13 (18.23)
(700 – 160.42) × 25% (700 – 87.50) × 25%
54.69
2016 101.17* 114.84** (13.67)
(134.90 × 75%) (153.13 × 75%)
2017 75.88*** 86.13**** (10.25)
(101.17 × 75%) (114.84 × 75%)

*(700 – 160.42 – 134.9) x 25%


**(700 – 87.5 – 153.13) x 25%
***(700 – 160.42 – 134.9 – 101.17) x 25%
****(700 – 87.5 – 153.13 – 114.84) x 25%
ICAP study text
Selection of accounting policies
Selection of accounting policies – Areas covered by IFRS
If an IFRS (or an Interpretation) applies to an item in the financial statements, the accounting policy or
policies applied to that item must be determined by applying the Standard or Interpretation and any
relevant implementation guidance issued.
Selection of accounting policies – Area not covered by IFRS
If there is no rule in IFRS that specifically applies to an item in the financial statements, management must use its
judgement to develop and apply an accounting policy that results in information that is:
1. relevant to the economic decision-making needs of users; and
2. reliable in that the financial statements:
 represent faithfully the financial performance, financial position and cashflows of the entity;
 reflect the economic substance of transactions and other events, and not merely the legal form;
 are neutral, ie free from bias;
 are prudent; and
 are complete in all material respects.

In making the judgement management must consider the following sources in descending order:
1. the requirements and guidance in IFRS dealing with similar and related issues;

Page 33 of 43
265
2. the definitions, recognition criteria and measurement concepts for assets, liabilities, income and expenses
set out in the “Framework”.
Management may also consider the most recent pronouncements of other standard-setting bodies that use a
similar conceptual framework to the extent that these do not conflict with the above sources.

Professional Judgement
Sometime, it may be difficult to understand how an accounting policy would be applied and you may need to use
your professional judgement. Professional judgement is required usually when you want to develop your own
accounting policy for certain transaction.

The professional judgement should be based on:


1. standards and interpretations on similar issues;
2. the definitions, recognition criteria and measurement concepts in the Conceptual Framework; and on
condition that they do not conflict with the standards and interpretations on similar issues and the Conceptual
Framework, we are also allowed to consider recent pronouncements from other standard setting bodies,
other accounting literature and industry accepted practices.
Consistency of accounting policies
An entity must apply consistent accounting policies in a period to deal with similar transactions, and other events
and circumstances, unless IFRS specifically requires or permits categorisation of items for which different policies
may be appropriate.

Illustration: Consistency

IAS 16: Property, plant and equipment allows the use of the cost model or the revaluation model for measurement
after recognition.

This is an example of where IFRS permits categorisation of items for which different policies may be appropriate.
If chosen, each model must be applied to an entire class of assets. Each model must be applied consistently within
each class that has been identified.

Example 1:

Company A has 2 machines, costing Rs. 1 million and 5 million respectively. Company A decides to use Cost model
for 1 Machine and Revaluation Model for 1 Machine. Are they allowed to do this as per IAS 8?
No, Accounting policy for a specific class of assets shall be consistent. Therefore, Company A may either use Cost
Model or Revaluation Model for all assets under the category of Machines.
Example 2:

Company A has Land costing Rs. 100 million, Plant & Machinery Costing Rs. 500 million, Vehicles costing Rs. 10
million and Electrical equipment costing Rs. 5 million. Company wish to apply Cost Model for Plant & Machinery,
Vehicles and Electrical equipment and Revaluation Model for Land. Whether Company A is permitted under IAS 8
to do this?
Yes, as the Company is applying same policy for a category of asset, it is allowed to apply different policies for
different class / category of assets.
Example of change in accounting policies:

 Valuation of inventory using FIFO or weighted average cost method


 Measurement of financial assets and liabilities
 Method used to measure non-current assets such as historical cost or revaluation model
 Accruals basis of preparation of financial statements

Page 34 of 43
266
 Presentation (e.g. an entity changes from presenting a classified statement of financial position (current
and non-current assets and current and non-current liabilities shown as separate classifications) to a
liquidity presentation (items presented in order of liquidity without current/non-current classification)
Illustration:

IAS 23 requires the capitalisation of borrowing costs directly attributable to the acquisition, construction or
production of a qualifying asset.
Previously, IAS 23 allowed companies to expense or capitalise borrowing costs.
The revision to IAS 23 led to a change in accounting policy for some companies as it affected:
 recognition – the interest cost previously recognised as an expense had to be recognised as an asset; and
 presentation – the interest cost previously presented in the statement of profit or loss had to be
presented in the statement of financial position.

IAS 8 specifies that the application of a new accounting policy to transactions or events that did not occur
previously or differ in substance from those that occurred previously, is not a change of accounting policy. It is
simply the application of a suitable accounting policy to a new type of transaction.[para 16]
Important Note:
Usually a question is being asked as to why change from Cost Model to Revaluation Model is not accounted for
under IAS 8, the answer is:
The initial application of a policy to revalue assets in accordance with IAS 16 Property, Plant and Equipment or IAS
38 Intangible Assets is a change in an accounting policy. However, it is accounted for in accordance guidance in
those standards rather than in accordance with IAS 8.

IAS 8 specifically mentions that when there is a change in accounting policy, the accounting standard which was
applicable on that element of financial statements be looked into first and if the treatment with regard to its
accounting is given then such treatment will be used, in case no treatment of such transaction is given under
relevant accounting standard, then IAS 8 will be followed, as per which in case of change in Accounting Policy,
there is a retrospective application.

Retrospective application of a change in accounting policy


When a change in accounting policy is required, and there are no transitional provisions relating to the
introduction of a new accounting standard, the change in policy should be applied retrospectively.
The entity should adjust the opening balance for each item of equity affected by the change, for the earliest prior
period presented, and the other comparative amounts for each prior period presented, as if the new accounting
policy had always been applied.
IAS 1: Presentation of Financial Statements requires a statement of financial position at the beginning of the
earliest comparative period when a new accounting policy is applied retrospectively.

Illustration:
A company presents comparatives for the previous year only.
During the year ended 31 December 2016 it changes an accounting policy and this change must be applied
retrospectively.

If there were no change in accounting policy the company would present statements of financial position as at
December 2016 and December 2015 only.

However, because there is a change in policy the company must also present a statement of financial position as
at 1 January 2015 (the beginning of the earliest comparative period).

Page 35 of 43
267
The change in accounting policy is applied retrospectively. This means that the change should be applied to the
balances at as at 1 January 2015 as if the new policy had always been applied.

Similarly, any other comparative amounts in previous periods should be adjusted as if the new accounting policy
had always been applied.
If this is impracticable, retrospective application should be applied from the earliest date that is practicable.
Limitation on retrospective application
It might be impracticable to retrospectively apply an accounting policy. This could be because the information
necessary for the application of the policy to earlier periods is not available because it had not been collected then.
There are different degrees of impracticability.
Period specific effect
It might be impracticable to determine the effect of changing an accounting policy on comparative information for
one or more prior periods presented. For example, it might be impracticable to determine the impact on profit for
the prior year.

In this case a company must apply the new accounting policy to the carrying amounts of assets and liabilities (and
therefore equity) as at the beginning of the earliest period for which retrospective application is practicable. This
may be the current period.
Cumulative effect
It might be impracticable to determine the cumulative effect, at the beginning of the current period, of applying a
new accounting policy to all prior periods,

In this case a company must adjust the comparative information to apply the new accounting policy prospectively
from the earliest date practicable.
When the cumulative effect of applying the policy to all prior periods cannot be determined, a company must
apply the new policy prospectively from the start of the earliest period practicable. This means that it would
disregard the portion of the cumulative adjustment to assets, liabilities and equity arising before that date.
Disclosure of a change in accounting policy
Disclosure: Change due to IFRS Voluntary change
The title of the IFRS Y
The nature of the change in Y Y
accounting policy
When applicable, that the change in Y
accounting policy is made in
accordance with its
y transitional
provisions
A description of any transitional Y
provisions
The reason why the new accounting Y
policy provides reliable
and more relevant information
If retrospective application is Y
impracticable, an explanation of
how the accounting policy change
has been applied

Page 36 of 43
268
ACCOUNTING ESTIMATES
Illustration:

Accounting policy: Depreciating plant and equipment over its useful life.
Accounting estimate: How to apply the policy. For example whether to use the straight line method of
depreciation or the reducing balance method is a choice of accounting estimate.
Illustration:

IAS 16: Property, plant and equipment allows the use of the cost model or the revaluation model for
measurement after recognition.
This is a choice of accounting policy.

Illustration:
A non-current asset was purchased for Rs. 200,000 two years ago, when its expected economic life was ten years
and its expected residual value was nil. The asset is being depreciated by the straight-line method.
A review of the non-current assets at the end of year 2 revealed that due to technological change, the useful life of
the asset is only six years in total, and the asset therefore has a remaining useful life of four years.
The original depreciation charge was Rs. 20,000 per year (Rs. 200,000
/10 years) and at the beginning of Year 2, its
carrying value was Rs. 180,000 (Rs. 200,000 - Rs. 20,000).
The change in the estimate occurs in Year 2. The change in estimate should be applied prospectively, for years 2
onwards (years 2 – 6). From the beginning of year 2, the asset has a revised useful remaining life of five years.
The annual charge for depreciation for year 2 (the current year) and for the future years 3 – 6 will be changed from
Rs. 20,000 to Rs. 36,000 ( Rs. 180,000/5 years).

Limitation on retrospective restatement of prior period errors


A prior period error must be corrected by retrospective restatement except to the extent that it
is impracticable to determine either the period-specific effects or the cumulative effect of the
error.

Period specific effect


It might be impracticable to determine the effect of correcting an error in comparative information for one or
more prior periods presented. For example, it might be impracticable to determine the impact on profit for the
prior year.
In this case a company must restate the carrying amounts of assets and liabilities (and therefore equity) as at the
beginning of the earliest period for which retrospective restatement is practicable. This may be the current period.
Cumulative effect
It might be impracticable to determine the cumulative effect, at the beginning of the current period, of correcting
an error in all prior periods.
In this case a company must correct the error prospectively from the earliest date practicable.

Disclosure of prior period errors:


The following information must be disclosed:
 the nature of the prior period error;
 for each period presented in the financial statements, and to the extent practicable, the amount of the
correction for each financial statement item and the change to basic and fully diluted earnings per share;

Page 37 of 43
269
 the amount of the correction at the beginning of the earliest prior period in the statements (typically, at the
start of the previous year);
 if retrospective re-statement is not practicable for a prior period, an explanation of how and when the error
has been corrected.
IAS 8 therefore requires that a note to the financial statements should disclose details of the prior year error, and
the effect that the correction has had on ‘line items’ in the prior year.

Reclassification of items of Statement of Profit and Loss Statement and Statement of Financial Position:
Sometime, an item is wrongly classified in a wrong category, for example, an expense that has to be accounted for
under Cost of Sales was inadvertently categorized in Administrative expenses or an item of Administrative
expenses was wrongly classified in Cost of Sales. Such wrong reclassification is corrected once this is identified,
even though have to change last year’s financial statements figures, but since there will be no impact on the
overall financial statements, this is regarded as reclassification and not restatement. If such case a rise, where the
impact is Nil on the Financial Statements and if the item is material, a note is given in Financial Statement
describing the reclassification.
A illustrative note is given below:

Description Head of Account Earlier Head of Account Now Amount in PKR


Vehicle running and Cost of Sales Administrative 1,000,000
Maintenance Expenses

Page 38 of 43
270
Test

Q.1 Following information has been extracted from the draft financial statements of Marvellous Limited (ML) for
the year ended 30 June 2017:

Statement of financial position


2017 2016
Rs. in million
Property, plant and equipment 700 612
Retained earnings 330 265

Statement of profit or loss


Profit for the year 65 85

The following matters are under consideration of the management:

 It was identified that ML has incorrectly changed Rs. 36.75 millions as maintenance expenses, incurred on
installation of plant. The plant was available for use on 01.07.2014 and has been depreciated on straight line
basis over a useful life of four year.
 In view of significant change in the expected pattern of economic benefits from an item of the equipment, it
has been decided to change the depreciation method from reducing balance to straight line. The equipment
was purchased on 1 July 2015 at a cost of Rs. 80 million having estimated useful life of 5 years and residual
value of Rs. 16 million. The depreciation at the rate of 27.5% on reducing balance method is included in the
above draft financial statements.

The following balances pertain to ML’s statement of financial position as on 30 June 2015:

Rs. in million
Property, plant and equipment 650
Retained earnings 180

Required:
Prepare extracts from the following (including comparative figures) for the year ended 30 June 2017:
a) Statement of financial position
b) Statement of profit or loss
c) Correction of error note

Page 39 of 43
271
A. 1
Marvellous Ltd.
Statement of Financial Position (Extracts)
As on 30-06-2017
Restated Restated
2017 2016 2015
Property , Plant & Equipment: 714.68 630.37 677.56
(2015: 650+36.75-9.19);(2016: 612+36.75-9.19 X 2)
(2017: 700+36.75-(9.19 X3) +5.5)
Retained earnings (W-3) 344.68 283.37 207.56

b) ML
Statement of profit or loss (extracts)
For the year ended 30-06-2017
Restated
2017 2016
Profit for the year 61.31 75.81
(2016: 85-9.19);(2017: 65-9.19+5.5)

c) Correction of error note


It was identified in current year that installation cost of Rs. 36.75 million to be capitalized was incorrectly charged
as maintenance expense on july 01,2014. The effects of corrections of this error as follows:
Effect on statement of profit or loss
2016
Increase in Depreciation (9.19)
Decrease in profit for the year (9.19)

Effect on Statement of Financial Position:


2016 2015
Increase in P.P.E 18.37 27.56
(630.37-612) (677.56-650)
Increase in R.E 18.37 27.56
(283.37) (207.56-180)

Workings:
W-1)
1-07-2014 Plant 36.75
Maintenance expense 36.75

30-06-2015 Dep. 9.19


Plant 9.19
(36.75/4 =9.19)
For 30-06-2016:
30-06-2016 Dep. 9.19
Plant (36.75/4) 9.19
For 30-06-2017:
30-06-2017 Dep. 9.19
Plant (36.75/4) 9.19

Page 40 of 43
272
W- 2) Change in estimate
1-07-2015 Equipment cost 80
30-06-2016 Dep. (80 X 27.5%) (22)
30-06-2016 WDV 58
30-06-2016 Dep. (16)
30-06-2017 WDV 42

Dep. Charged during 2017 = 16


Dep. to be charged (58-16/4) = 10.5
Dep. To be reversed = 5.50

30-06-2017 Acc. Dep/ Equipment 5.5


Dep. 5.5
(16-10.5) = 5.5

(W-3) Statement of changes in equity:


Balance on 1-07-2015 180
Effect of error (36.75-9.19) 20.56
Balance as on 1-07-2015- Restated 207.56
PAT for 2016 –Restated 75.81
Balance as on 30-06-2016- Restated 283.37
PAT- 2017 61.31
Balance as on 30-06-2017 344.68

Q.2
During the year, it was discovered that due to some calculation error in excel sheet, fair value of CL’s office
building was taken incorrectly as Rs. 460 million instead of Rs. 360 million. Resultantly, the building was recorded
based on incorrect revaluation amount in CL’s financial statements for the year ended 30 June 2017.

This building was acquired on 1 July 2015 for Rs. 500 million and then revalued for the first time on 30 June 2017.

CL follows revaluation model for subsequent measurement of its building classified as property, plant and
equipment and charges depreciation over its useful life of 10 years using straight line method. CL accounts for
revaluation on net replacement value method and transfers the maximum possible amount from the revaluation
surplus to retained earnings on an annual basis.

As on 30 June 2019, the revalued amount of building has been determined at Rs. 320 million.

Required:
Prepare extracts from CL’s statement of financial position and related notes to the financial statements for the
year ended 30 June 2019 alongwith comparative figures for the above.
(Note on Property, plant and equipment is not required)

Page 41 of 43
273
A.2 Coal Limited
Statement of financial position
As on 30 June 2019
2019 2018 2017
Non-current assets: Restated Restated
------------ Rs. in million ------------
Building 320 315 360
(360-45)
Equity
Revaluation surplus (W-2) 20.00 - -

Correction of error note:


It was identified in current year that revalued amount of one of its buildings was taken as Rs. 460 million instead of
360 million in 2017's financial statements of the company.

Effect on the statement of profit or loss 2018


Rs. in million
Decrease in depreciation expense (100÷8) 12.50
Effect on the statement of financial position
2018 2017
---- Rs. in million ----
(100.00)
Decrease in PPE [(460-57.50) – 315] or (-100+12.5) (87.50) [460-360]
Decrease in Revaluation surplus (-60+7.5) (52.50) (60.00)
Decrease in retained earnings (-40 +12.5 -7.5) (35.00) (40.00)

W-2
Prior Period Error
01-07-2015
Cost 500
Accumulated depreciation [500÷10×2] (100)
WDV (30.6.2017) 400

Adjustment that would have been made on30.6.2017


WDV = 400
Fair value = 460
Revaluation surplus = 60

Office building 60
R. Surplus 60

Page 42 of 43
274
Adjustment that should have been made on 30.6.2017:
WDV = 400
Fair value = 360
R. Loss = 40

Revaluation loss 40
Office building 40
Rectification on 30.6.2017
R. loss (R.E) 40
R. Surplus 60
Office building 100
Depreciation that would have been charged in 2018
460 ÷ 8 = 57.5
Depreciation that should have been charged in 2018
360 ÷ 8 = 45
Extra Depreciation of 12.5 to be reversed in 2018

Office building 12.5


Depreciation (R.E) 12.5

Transfer of surplus that would have been made in 2018:


60 ÷ 8 = 7.5
However no transfer of surplus should be made in 2018:
Retained Earning 7.5
Revaluation surplus 7.5
Depreciation to be charged in 2019:
360 ÷ 8 = 45
Revaluation as on 30.6.2019
WDV [ 360- (45×2) ] = 270
Fair value = 320
Revaluation surplus = 50

Reversal of loss 40 Cr Revaluation Surplus 20


Extra dep’n to be charged 10 Dr
[50-45]×2
Net reversal of loss 30 Cr

Office building 50
Reversal of loss (IS) 30
Rev. surplus 20

Page 43 of 43
275
Test Revaluation Date: -- / -- / --

QUESTION
The following information pertains to the property, plant and equipment of Orchid Limited (OL), a listed
company:
Description Date of Cost Rs. In Original useful Depreciation Subsequent
purchase millions life method measurement
model
Buildings 01.01.2015 600 30 years Straight line Revaluation
Plant 01.01.2015 475 25 years Straight line Cost

Buildings
The revalued amount of buildings as determined by Shabbir Associates, an independent valuer, on 31.12.2015 and
31.12.2017 was Rs. 700 million and Rs. 463 million respectively.

On 30.06.2017 a building having original cost of Rs. 66 million was sold to Baqir limited for Rs. 85 million. It was last
revalued at 87 million. OL incurred a cost of Rs. 2 million on disposal.
OL transfers maximum possible amount from revaluation surplus to retained earnings on an annual basis.

Plant
During 2017, OL has decided to change the depreciation method for plant from straight line to reducing balance
method. The new depreciation rate would be 10%.

Required:
In accordance with International Financial Reporting Standards, prepare a note on ‘Property plant & equipment’
(including comparative figures) for inclusion in OL’s financial statements for the year ended 31 December 2017. (18)

Page 1 of 55
276
A.
Orchid Limited
Notes to financial statements
For the year ended 31-12-2017

Property, plant and equipment


2017
Building Plant Total
-------- Rs. In million ---------
Gross carrying amount:
Opening balances 700 475 1,175
Addition … … …
Disposal (87) … (87)
Elimination (42.28) … (42.28)
Revaluation:
Revaluation surplus (90.12) … (90.12)
Revaluation loss (17.6) … (17.6)
Closing Balance 463 475 938
Accumulated depreciation
Opening balance 24.14 38 62.14
Depreciation 22.64 43.7 66.14
Disposal (4.5) … (4.5)
Elimination (42.28) … (42.28)
Closing … 81.7 81.7
Carrying amount 463 393.3 856.3

Property, plant and equipment


2016
Building Plant Total
-------- Rs. In million ---------
Gross carrying amount:
Opening balance 700 475 1,175
Addition/Deletion … … …
Closing balance 700 475 1,175
Accumulated depreciation
Opening balance … 19 19
For the year 24.14 19 43.14
Closing balance 24.14 38 62.14
Carrying amount 675.86 437 1,112.86
Disclosures:
Measurement base Revaluation model Cost model
Useful life 30 years 10%
Method of depreciation Straight line Reducing balance
The last revaluation was performed by Shabbir Associates, an independent firm of valour’s on 31-12-2017.
Carrying Amount of building if carried at cost model:
2017 2016
Cost 534 600
Accumulated depreciation (53.4) (40)
Carrying Amount 480.6 560

Page 2 of 55
277
Working for the above figures
Building A/c
1-1-2015 Cash 600
c/d 600
b/d 600
c/d 600
b/d 600 30-6-2017 Disposal 66
c/d 534

Accumulated Depreciation A/c


31-12-2015 Depreciation 600/30 20
c/d 20
b/d 20
c/d 40 31-12-15 Depreciation600/30 20
30-6-2017 Disposal[66/30x2.5] 5.5
b/d 40
c/d 53.4 Depreciation[600- 18.9
66)/30+(66/30x6/12)]

Ledgers according to question:


Building
1-1-2015 b/d ----
1-1-2015 Cash 600 31-12-2015 Acc Depreciation 20
31-12-2015 Revaluation Surplus(w-1) 120 c/d 700

1-12-2016 b/d 700 c/d 700


b/d 700 31-06-2017 Disposal 87
31-12-2017 Acc. Depreciation 42.28
R.surplus 90.12
R.loss(W-2) 17.6
31-12-2017 c/d 463

Accumulated Depreciation
1-1-2015 b/d ----
31-12-2015 Building 20 31-12-2015 Depreciation (600/30) 20
c/d -
b/d -
c/d 24.14 Depreciation(700/29) 24.14
30-06-2017 Disposal(87/29X1.5) 4.5 b/d 24.14
Building 42.28 31-12-2017 Depreciation (1.5+21.14) 22.64
c/d -

Revaluation Surplus
1-1-2015 b/d ----
31-12-2015 Building 120
c/d 120

Retained earnings 4.14 b/d 120


[120/29]
c/d 115.86
30-06-2017 Retained earnings 0.4 b/d 115.86
30-06-2017 Retained earnings 22

Page 3 of 55
278
31-12-2017 Retained earnings 3.34
31-12-2017 Building 90.12
c/d -

Workings :

W-1) 31.12.2015
WDV [ 600 – 20] = 580
FV = 700
Rev. Surplus = 120

W-2) 30.6.2017 (Rev. Surplus related to disposal)


WDV [ 66 – (66/30)] = 63.8
FV = 87.0
Rev. Surplus = 23.2

W-3) 31.12.2017
WDV
FV [ 700 – 87 ] = 613
Accumulated Depreciation = 42.28
[ 613 / 29 x 2 ] 570.72

FV = 463.0
Rev. Loss 107.72

Journal Enteries : (for the year ended 31-12-2017)


Dr. Cr.
30-06-2017
Depreciation 1.5
Accumulated Depreciation 1.5
[ 87 / 29 x 6/12 ]

Revaluation Surplus 0.4


Retained Earnings 0.4
[ 23.2(w-2) / 29 x 6/12 ]

Cash ( 85 – 2 ) 83
Accumulated Depreciation 4.5
( 87 / 29 x 1.5 years )
Building 87
Gain ( P.L ) 0.5

Revaluation Surplus 22
Retained Earnings 22
[ 23.2 – (23.2 / 29 x 1.5 ] = 22

31-12-2017
Depreciation 21.14
Accumulated Depreciation 21.14
[ 700 – 87 ] / 29 = 21.14

Page 4 of 55
279
31-12-2017
Rev.Surplus 3.34
Retained Earnings 3.34
[ 120 – 23.2 ] / 29 = 3.34

31-12-2017
Accumulated Depreciation 42.28
Building 42.28

31-12-2017 90.12
Revaluation Surplus 17.60
Revaluation Loss ( P.L ) 107.72
Building ( W-3 )

Workings for Plant:


Plant
1-1-2015
Cash 475
c/d 475
1-1-2016
b/d 475
c/d 475
1-1-2017
b/d 475
c/d 475

Accumulated depreciation
1-1-2015
b/d -

31-12-2015
Depreciation 19
(475 /25)
c/d 19

b/d 19

31-12-2016
Depreciation 19
c/d 38
b/d 38

31-12-2017
Depreciation 43.7
c/d 81.7

*[475 – 38] x 10% = 43.7

Page 5 of 55
280
Test revaluation:

Q.1 The following information pertains to Gava Limited (GL):

Equipment

 Date of acquisition 1 January 2015 1 July 2015


 Cost Rs. 500 million Rs. 360 million
 Estimated useful life 10 years 12 years
 Residual value Rs. 60 million Nil
 Depreciation method Straight line method Straight line method

 Fair value Rs. 526 million Rs. 280 million


 Residual value Rs. 78 million Nil

 Fair value Rs. 310 million Rs. 230 million


 Residual value Rs. 64 million Nil

Additional information:
(i) GL uses revaluation model for subsequent measurement and accounts for
revaluation on net replacement value method.
(ii) There is no change in useful life of plant. The remaining useful life of equipment
was estimated as 14 years and 10 years after revaluation on 31.12.2016 and
31.12.2018 respectively.
(iii) GL transfers maximum possible amount from the revaluation surplus to retained
earnings on an annual basis.
(iv) GL’s financial year ends on 31 December.

Required:
(a) Calculate depreciation on each asset for 2015 to 2018. (08)
(b) Prepare entries to record revaluation in 2018. (Entries to record depreciation expense,
incremental depreciation and elimination of accumulated depreciation are not required.
Further, entries prior to 2018 are also not required.) (08)

Page 6 of 55
281
Ans.1 Guava Limited

a)
Rs. in Million
Plant
Cost [1.1.2015] 500
Depreciation 2015 - [500 – 60 / 10] (44)
Carrying Amount [31.12.2015] 456
Depreciation 2016 - [500 – 60 / 10] (44)
Carrying Amount [31.12.2016] 412
Revaluation Surplus (bal.) 114
Fair Value [31.12.2016] 526
Depreciation 2017 - [526 – 78 / 8] (56)
Carrying Amount [31.12.2017] 470
Depreciation 2018 - [526 – 78 / 8] (56)
Carrying Amount [31.12.2018] 414

Rs. in Millions
Equipment
Cost [1.7.2015] 360
Depreciation 2015 - [360 / 12 x 6/12] (15)
Carrying Amount [31.12.2015] 345
Depreciation 2016 - [360 / 12] (30)
Carrying Amount [31.12.2016] 315
Revaluation Loss (P.L) (bal.) (35)
Fair Value [31.12.2016] 280
Depreciation 2017 - [280 / 14] (20)
Carrying Amount [31.12.2017] 260
Depreciation 2018 - [280 / 14] (20)
Carrying Amount [31.12.2018] 240

b) Accounting enteries to record revaluation:


31.12.2018 Dr. Cr.
Revaluation Surplus 85.5
[114 – (114/8 x 2)]
Revaluation Loss (P.L) (bal.) 18.5
Plant (W-1)
104
31.12.2018
Revaluation Loss (P.L) 10
Equipment (W-1) 10

Page 7 of 55
282
Workings:

W-1) 31.12.2018

Plant:
Carrying Amount [From (a)] 414
Fair Value [Given] 310
Revaluation Loss 104

Equipment:
Carrying Amount [From (a)] 240
Fair Value [Given] 230
Revaluation Loss 10

Page 8 of 55
283
Test Statement of changes in Equity Date: -- / -- / --

Q.1 The following information pertains to Wednesday Limited (WL) for the year ended
30 une 2019:
(i) Shareholders' equity as at 1 July 2018:

Rs. In million
Share capital (Rs. 100 each) 200
Share premium 85
Retained earnings 124
Revaluation surplus 65

(ii) On 30 November 2018, WL issued 30% right shares at a premium of Rs. 120 per share.
(ii) Cash dividend and bonus shares for the last two years:
Final dividend *Interim dividend
For the year ended
Cash Bonus Cash Bonus
30 June 2018 18% - 20% -
30 June 2019 - 25% - 10%

*Declared with half yearly accounts


(iii) Profit for the year amounted to Rs. 95 million.

(iv) Revaluation surplus arising during the year amounted to Rs. 35 million whereas transfer of
incremental depreciation for the year was Rs. 9 million.

Required:
Prepare WL’s Statement of Changes in Equity for the year ended 30 June 2019. (07)
(Column for total and comparative figures are not required)

Page 9 of 55
284
A.1 Wednesday Limited
Statement of changes in equity
For the year ended 30 June 2019
Share Share Retained Revaluation
capital premium earnings surplus
--------------- Rs. in million ---------------
Balance as at 1 July 2018 (As given) 200 85 124 65
Final cash dividend @ 18% for 2018
(200×18%) (36)
Right issue @ 30% 60 72
(200/100 x 30%) (0.6 x 100) (0.6×120)
Interim bonus dividend @ 10% for 2019
(260×10%) 26 (26)
Profit for the year 95
Increase in revaluation surplus 35
Transfer of incremental depreciation 9 (9)
Balance as at 30 June 2019 286 131 192 91

Page 10 of 55
285
Test SOCE Date: -- / -- / --
Q. 1
The following information pertains to draft financial statements of Pak Ocean Limited (POL) for the year ended 31
December 2014.
(i)
2014 2013
Rs. In million
Profit after tax 90 47
Incremental depreciation on revaluation of property, plant and
equipment (means transfer of Revaluation surplus) 1.5 2.3

(ii) Installation of an assembly plant was completed in December 2013 at a cost of Rs. 60 million and it was
ready for use on 1 February 2014. However, depreciation for the year ended 31 December 2014 amounting
to Rs. 4.5 million was worked out from the date of production i.e. 1 April 2014.
(iii) Shareholders’ equity as at 1 January 2013 was as follows:

Rs. In million
Share Capital (Rs. 100 Each) 200
Retained Earning 45
Revaluation surplus 100

On 30 November 2014, POL issued 25% right shares to its ordinary shareholders at Rs. 120 per share.
(iv) Cash dividend and bonuses declared/paid during the last three years:

Final *Interim
For the year ended
Cash Bonus Cash Bonus
31 December 2012 - 15% 16% -
31 December 2013 18% - 20% -
31 December 2014 - 25% - 10%

*Declared with half yearly financial statements


Required:
Prepare Statement of Changes in Equity for the year ended 31 December 2014 in accordance with the
requirements of the Companies Act 2017 and International Financial Reporting Standards. (15)
(Ignore taxation)

Page 11 of 55
286
Ans. 1

PAK OCEAN LIMITED


Statement of changes in Equity
For the year ended 31-12-2014
“Rs In Millions”
Share Share Retained Revaluation Total
Capital Premium Earnings Surplus
Balance as on 1-1-2013 200 - 45 100 345
Bonus Share-2012 (200 X 15%) 30 (30) -
Interim Cash dividend-2013 (230 X 20%) (46) (46)
Profit after tax 47 47
Transfer of Surplus 2.3 (2.3) -
Balance as on 1-1-2014 230 - 18.3 97.7 346
Final cash dividend-2013 (230 X 18%) (41.4) (41.4)
Interim bonus-2014(230 X 10%) 23 (23) -
Issue of right shares- (W-2) 63.25 12.65 - 75.90
Profit after tax (W 1) 89 89
Transfer of surplus 1.5 (1.5)
Balance as on 31-12-2014 316.25 12.65 44.4 96.2 469.5

W-1 Current period error


2014
Profit After tax-Given 90
Effect of depreciation* (1)
Profit after tax [adjusted] 89

4.5/9 X 2 = 1 (effect of depreciation of Feb & March 2014).

W-2) [230+23] /100 = 2.53 M shares X 25 % = 0.6325 M shares


0.6325M X 100 = 63.25
0.6325 M X 20 = 12.65

Page 12 of 55
287
Test statement of changes in equity:
Q.1 Following information have been extracted from the financial statements of Fakhr
Limited (FL) for the year ended 31 December 2019:

(i) 2019 2018 2017


Draft Audited Audited
--------- Rs. in million ---------
Net profit 84 98 72

(ii) Share capital and reserves as at 1 January:

2018 2017
----- Rs. in million -----
Share capital (Rs. 10 each) 300 300
Retained earnings 348 276

(iii) On 1 March 2018, FL declared a final cash dividend of 10% for the year ended 31
December 2017. On 1 November 2018, FL issued 40% right shares to its ordinary
shareholders at Rs. 24 per share. On 1 August 2019, an interim bonus of 15% was
declared.

Following information need to be incorporated in the draft financial statements of FL:

On 1.1.2019 company changed the method of valuation of subsequent measurement of property plant
and equipment from cost model to revaluation model.

Useful life Cost Acc. Depreciation Revalued Amount


Factory Rs in millions
Buildings 30 years 300 75 280

On 31.8.2019, one of the factory building having written down value of Rs 20.5875 millions and revalued
amount of Rs.25.65 millions was sold for Rs.22 millions.

The cost of disposal was 0.5 millions.

On 31.12.2019 remaining factory buildings were revalued at Rs.205 millions.

There is no change in the estimated useful life and company transfers maximum amount of revaluation
surplus to retained earnings on annual basis.

Required:
Prepare FL’s statement of changes in equity (including comparative figures) for the year ended 31
December 2019. (‘Total’ column is not required) (18)

Page 13 of 55
288
Ans.1 Fakhr Limited
Statement of Changes In Equity
For the year ended 31 December 2019

Ordinary share Share Revaluation Retained


capital premium surplus earnings
---------- Rs. in million ----------
Balance as at 31 December 2017 (As 300.00 - 348.00
given)
Final cash dividend @ 10% for 2017 - - - (30.00)
( 300 x 10% )
Right issue @ 40% 120.00 168.00 - -
( 300 / 10 x ( 300 / 10 x
40% x 10) 40% x 14)
– Net profit for 2018 - - - 98
Balance as at 31 December 2018 420.00 168.00 - 416
Interim bonus dividend @ 15% for 63.00 (63.00) - -
2019 (420 x 15%)
Revaluation Surplus (workings) 55
– Net profit (workings) - - - 68.55
- Transfer of Surplus (workings) - - (7.2825) 7.2825
-Revaluation Loss (workings) - - (38.05) -
Balance as at 31 December 2019 483.00 105.00 9.6675 491.8325

Workings

Date Particular Debit Credit


1.1.2019 Accumulated depreciation Factory 75
Building 75
1.1.2019 Factory building 55
Revaluation Surplus 55
C.A [300 – 75 ] = 225
F.V = 280
Revaluation Surplus = 55
31.08.2019 Depreciation 0.76
Accumulated depreciation 0.76
[25.65 / 22.5(Working) x 8/12]
31.8.2019 Revaluation Surplus 0.15
Retained Earnings 0.15
[25.65 – 20.5875 = 5.0625 / 22.5 x 8/12]
31.8.2019 Cash (22 – 0.5) 21.5
Accumulated depreciation 0.76
Loss(P.L) 3.39
Factory building 25.65
31.8.2019 Revaluation surplus 4.9125

Page 14 of 55
289
Retained earnings [5.0625 – 0.15] = 4.9125 4.9125
31.12.2019 Depreciation 11.30
Accumulated Depreciation(280-25.65) /22.5 11.30
31.12.2019 Revaluation Surplus 2.22
Retained Earnings (55 – 5.0625) / 22.5 = 2.22 2.22
31.12.2019 Accumulated Depreciation 11.30
Factory buildings 11.30
31.12.2019 Revaluation Surplus (W-2) 38.05
Factory Buildings 38.05

Workings:
(W-1)
Remaining useful life on 01.01.2019 after revaluation:
300/30 = 10 per anum
75/10 = 7.5 years
30 – 7.5 = 22.5 remaining years
Carrying Amount (280 – 25.65) – 11.30 = 243.05
Fair Value = 205.00
Revaluation Loss 38.05

Remaining revaluation surplus before revaluation on 31.12.2019 [55 – 5.0625 – 2.22] = 47.7175
After this revaluation on 31.12.2019, fair value of Rs.205 million will be depreciated over remaining useful life of
21.5 years from next year onwards. In addition remaining surplus of Rs.9.6675 (47.7175 – 38.05) will be
transferred to retained earnings over remaining useful life of 21.5 years from next year onwards.

Effect on net profit for the year ended 31.12.2019


Given profit 84 Cr.
Depreciation 0.76 Dr.
Loss 3.39 Dr.
Depreciation 11.30

Dr. Adjusted profit 68.55

Cr.

(W-3)

Revaluation Surplus
31.08.2019 Retained Earnings 0.15 01.01.2019 Factory Building 55
31.08.2019 Retained Earnings 4.9125
31.12.2019 Retained Earnings 2.22

31.12.2019 Factory Building 38.05


c/d 9.6675

Total transfer of revaluation surplus during the year: 0.15 + 4.9125 + 2.22 = 7.2825

Page 15 of 55
290
FAR-I TEST
IAS 23 Time 1:00hr

Q.1 Octagon Pakistan Limited (OPL) in is process of preparation of financial statements for the year ended 31
December 2018. During the year, OPL completed the construction of its head office building. Relevant
details in this respect are as follows:

(i) Payments related to the construction of the building were as follows:


Description Date of payment Rs. In million
Construction permit fee 1-Jan-18 30
Advance to contractor 1-Jan-18 80
1st bill of contractor 1-Feb-18 250
2nd bill of contractor 1-May-18 360
3rd bill of contractor 1-Sept-18 170
Last bill of contractor (Received on 25-12-2018) 1-Jan-19 150
1,040

(ii) The project was financed through the following sources:


Excess cash of Rs. 200 million available with OPL on 1 January 2018 in a saving account at 10% per annum.

Page 16 of 55
291
Loan of Rs. 350 million at the rate of 16% per annum obtained on 1 February 2018. The principal is payable in 5
equal annual instalments alongwith interest, from 1 February 2019. The surplus funds available from the loan
were invested in a saving account at 10% per annum.

Withdrawals from running finance facilities arranged on 1 Jan 2018. The facilities were also used to finance other
needs of OPL. Details of these facilities are as follows:

Name of bank Balance as on Limit Average Finance


31 December 2018 Balance cost
------------------------ Rs. in million ------------------------
Bank X 130 150 140 15.4
Bank Y 340 600 390 44.2

Payment of 3rd bill of contractor includes Rs. 10 million which was charged by the contractor for damages
sustained at the site on account of unexpected rains.

The work was stopped from 16 to 31 May 2018 to meet mandatory technical requirements. Further, on 16
September 2018, the building control authority stopped the construction work as it raised objections on the
design of the building. The matter was resolved on 30 September 2018.

Construction of the building was completed on 31 October 2018. However, it was inaugurated on 1 December
2018. The building has an estimated useful life of 30 years.

Required:
Prepare relevant extracts from OPL's statement of profit or loss for the year ended 31 December 2018 and
statement of financial position as on that date.
(Notes to the financial statements are not required. Borrowing costs are to be calculated on the basis of
number of months) (17)

292

Page 17 of 55
Test Solution
Ans.1 Octagon Limited
Statement of financial position as on 31 December 2018
Rs. in million
Non-current assets:
Building (W-1) 1,069.95

Non-current liabilities:
Term loan 350–70 280.00

Current Liabilities:
Loan 350÷5 70.00
Running finance 130+340 470.00
Interest payable on loan 350×16%×11/12 51.33
Payable to contractor 150.00

Statement of profit or loss for the year ended 31 December 2018

Investment income (200 – 30 – 80 = 0.75


90 x 10% x 1/12)
Interest expense: (60.26)
 Specific loan (350×16%×2.5/12) 11.67
(2.5 = N, D & From 16 – 9 to
30-9)
General borrowings (15.4+44.2 – 11.01) 48.59
Depreciation expense (5.98)
Other expenses (damages due to rain) (10.00)

W-1: Building Rs. in million


Permit fee 30.00
Payment to contractor 80+250+360+170+150–10 1,000.00
Borrowing cost capitalized (W-2) 45.93
1,075.93
Depreciation 1,075.93÷30×2/12 (5.98)
1,069.95

W-2: Borrowing cost capitalized Rs. in million


Specific loan (1 Feb. to 31 Oct. excl. 15* days) 350×16%×8.5/12 39.67
*15= (16-9 to 30-9)
Investment income (W-3) 190×10%×3/12 (4.75)
34.92
General borrowings (W-4) 11.01
45.93

293
W- 3
Detail of Payments ;

Funds from
Owned Specific fund General
Date Particulars Amount
1-1-2018 Permit fee 30 30
1-1-2018 Advance 80 80
1-2-2018 Ist bill 250 90 160
1-5-2018 2nd bill 360 190 170
1-9-2018 3rd bill 160 160
(170 – 10*)
*Abnormal wastage to be expensed

W-4: Capitalization rate


Calculation of capitalization rate
59.6/530 x 100 = 11.25% p.a
General borrowings :
170 x 11.25% x 5.5/12 = 8.76
160 x 11.25 % x 1.5/12 =2.25
11.01

294
Test Cash Flow Date: -- / -- / --

Question 1
The following information has been extracted from the draft financial statements of Alpha Limited for the year
ended 31 December 2015.

2015 2014 2015 2014


Assets Equity & Liabilities
Rs. In million Rs. In million
Property, plant & equipment 223 193 Share capital (Rs. 10 each) 180 150
Intangible assets 68 23 Share premium 15 -
Trade receivables 45 33 Retained earnings 114 53
Advances and prepayments 84 70 Long term loan 40 -
Inventories 60 46 Trade payables 57 66
Short-term investments 12 9 Accrued expenses 60 70
Cash at bank 8 7 Tax payable 34 42

500 381 500 381

Following relevant information is available:


(i) On 1 September 2015, the company purchased new machinery costing Rs. 65 million.
(ii) A portion of building costing Rs. 20 million which was purchased on 1 July 2013 was sold for Rs. 20 million
on 30 June 2015.The useful life of building was 20 years.
(iii) Trade receivables written off during the year amounted to Rs. 5 million. It is the policy of the company to
maintain the provision for doubtful debts at 5% of trade receivables.
(iv) Advances and prepayments include advance tax of Rs. 8 million (2014: Rs. 6 million).
(v) Long term loan was obtained on 1 August 2015. Interest on loan @ 13% is payable on 31 st July each year.
Interest payable for 5 months has been accrued.
(vi) Tax expense for the year was Rs. 17 million. (2014: Rs. 8 million).
(vii) Right shares were issued on 1 December 2015 at Rs. 15 per share in the ratio of 1 right share for every 5
shares held.
Required:
Prepare statement of cash flows for the year ended 31 December 2015 in accordance with the requirements of
International Financial Reporting Standards using the indirect method. (15)

295
Answer 1
Alpha Limited
Statement of Cash Flow
For the year ended 31-12-2015
Cash Flow From Operating Activities:

Profit before tax (61 + 17) 78


Interest expense 2.17
Depreciation 17.00
Gain on disposal (2.00)
Bad debts (5+0.63) 5.63
Profit before working capital changes 100.8

Working Capital Changes:

Trade debtors (17.63)


Inventories (60 – 46) (14.00)
Advances and Prepayments (12.00)
Trade payables (57 – 66) (9.00)
Accrued expense (12.17) (64.8)
Cash for generated from operations 36
Tax paid (27)
Net cash from operating Activities 9.00
Cash flow From Investing Activities:
Purchase of machinery (65)
Receipt from disposal 20
Acquisition of intangibles (45)
Purchase of investments (12 – 9) (3)
Net Cash From Investing Activities (93)

Cash Flow From Financing Activities:


Proceeds from issue of right shares (150/10 = 15 × 1/5 × 15) 45
Proceeds from long term loan 40
85
Net Cash Flow 1
Cash and cash equivalents at the beginning of the period 7
Cash and cash equivalents at the end of the period 8

296
Workings:
PPE – WDV

b/d 193 Disposal (20 – 2) 18


Cash 65 Depreciation (bal.) 17
c/d 223

20 ÷ 20 × 6/12 = [0.5 + 1 + 0.5] = 2


Intangibles

b/d 23
Cash 45
c/d 68

Receivables

b/d 34.74 Bad debts 5


17.63
c/d 47.37

33/95% = 34.74
45/95% = 47.37
Provision

b/d 1.74
Bad debts 0.63
c/d 2.37

Advances

b/d (70 – 6) 64
12
c/d (84 – 8) 76

Advances tax + tax Payable

b/d 6 b/d 42
Cash 27 Expenditure 17
c/d 34 c/d 8

Share Capital

b/d 150
Cash 30
c/d 180

Share Premium

b/d --

297
Cash 15
c/d 15

Retained Earnings

b/d 53
PAT 61
c/d 114

Loan

b/d --
40
c/d 40

Accrued Expenses

12.17 b/d 70

c/d (60 – 2.17*) 57.83

*(40 × 13% × 5/12 = 2.17)


Interest Payable

b/d --
Expenses 2.17
c/d 2.17

Disposal A/c

PPE 18 Cash 20

Gain 2

298
Test Cash Flow (Direct Method) Date: -- / -
- / --
Q.Following are the extracts from the financial statements of Emporium Mall (EM) for the year ended 30 June
2018:
Statement of Financial Position as on 30 June 2018
201 2017 2018 201
Equity & Liabilities 8 Asset 7
Rs. in s Rs. in
000 000
Share capital (Rs.10 700,000 300,000 Property, plant and 740,188 228,000
each) equipment
Share premium 70,000 - Stock in trade 116,000 90,000
Retained earnings 58,868 43,000 Trade receivables 72,812 112,000
Revaluation Surplus 20,000 - Cash 95,868 96,000
Debentures 36,000 52,000 Service income receivable 4,000 8,000
Interest payable 2,000 5,000 Prepaid Expenses 2,000 4,000
Trade payables 84,000 86,000
Accrued Expense 40,000 36,000
Provision for taxation 20,000 16,000
1,030,868 538,000 1,030,868 538,000

Statement of profit or loss for the year ended 30 June 2018


Rs. in ‘000’
Sales 546,000
Cost of sales (375,000)
Gross profit 171,000
Operating expenses (93,532)
Other income 22,400
Profit before interest and tax 99,868
Interest expense (4,000)
Profit before tax 95,868
Tax expense (30,000)
Profit after tax 65,868

Additional information:
i. 85% of sales were made on credit.
ii. EM maintains a provision for doubtful receivables at 3%. During the year, trade receivables of
Rs.14 million were written off.
iii. Depreciation expense for the year was Rs.45 million. 55% of the depreciation was charged to cost of sales.
iv. On January 1, 2018 EM made a bonus issue of one share for every six held and 35 million
right shares were issued on March 31, 2018 at Rs.12 per share.
v. Other income comprises of:
 Gain of Rs.6 million on disposal of vehicles for Rs.24 million;
 Service income of Rs 16.4 million

Required:
Prepare EM’s statement of cash flows for the year ended 30 June 2018 using direct method. (15)

299
A.
Emporium Mall Statement of Cash Flow
For the year ended 30-6-2018
Cash Flow from Operating Activities:
Cash receipt from customer [546,000 x 15% + 572,400
490,500]
Payment to suppliers (W) (378,250)
Payment for expenses (W) (54,494)
Receipt from services income (W) 20,400
Cash generated from operations 160,056
Interest paid (7,000)
Tax paid (26,000)
Net cash from operation activities 127,056
Cash Flow from Investing Activities:
Receipt from disposal 24,000
Purchase of PPE (555,188)
Net cash from investing activities (531,188)
Cash Flow from Financing Activities:
Receipt from issue of shares (35,000 x 12) 420,000
Repayment of debentures (16,000)
Net cash from financing activities 404,000
Net cash flow (132)
Cash and cash equivalents at the beginning of the 96,000
period
Cash and cash equivalents at the end of the period 95,868
Workings:
Cash and Cash Equivalents:
2018 2017
Cash 95,868 96,000

PPE
b/d 228,000 Disposal 18,000
R.S 20,000 Depreciation 45,000
Cash 555,188
c/d 740,188

Stock
b/d 90,000 Cost of sales 350,250
[375,000 – (45,000 × 55%)]
Purchases(bal) 376,250
c/d 116,000

Provision A/C
b/d (112,000/97 x 3)
3,464
Expense 1,212
c/d (72,812/97 x 3)
2,252

300
Trade Receivables
b/d 112,000/97 x 100] 115,464 Bad debts 14,000
Sales (546,000 x 85%) 464,100 Cash 490,500
c/d [72,812/97 x 100] 75,064

Share Capital
b/d 300,000

Dividend [Bonus Share(300,000/10 = 50,000


30,000 x 1/6 x 10)]
Cash (35,000 x 10) 350,000
c/d 700,000

Share Premium
b/d
Cash (35,000 x 2) 70,000
c/d 70,000

R/Earnings
Dividend 50,000 b/d 43,000
PAT 65,868
c/d 58,868

301
R.Surplus
b/d --
PPE 20,000
c/d 20,000

Debentures
b/d 52,000
Cash 16,000
c/d 36,000

Prepaid and Accrued Expences


b/d 4,000 b/d
36,000
Cash 54,494 Expense (93,532 -14,000 – 45,000 x 45 % + 1,212)
60,494
c/d 40,000 c/d
2,000

Service Income Receivable


b/d 8,000
Income 16,400 Cash 20,400
c/d 4000

Interest Payable
b/d 5,000
Cash 7,000 Exp 4,000
c/d 2,000

Trade Payable

b/d 86,000
Cash 378,250 Purchases 376,250
c/d 84,000

Disposal A/c

PPE 18,000 Cash 24,000


Gain 6,000
Provision for Tax
b/d 16,000
Cash 26,000 Expence 30,000
c/d 20,000

302
Test IAS-23 Date: -- / -- / --

Q. On 1 July 2015, Minhas Manufacturers Limited (MML) commenced construction of its new factory
building and completed the work on 30 June 2016. Following information is available in this respect:

The agreed price of the contract was Rs. 100 million which was financed through the following sources:

Bank loan of Rs. 80 million was obtained on 1 June 2015. The loan carries a mark-up of 10.9436% per annum and is
repayable in five semi-annual instalments of Rs. 16 million each along with interest commencing from 30
November 2015.

The remaining amount was financed through cash withdrawals from MML’s existing running finance facilities.
Details of these facilities are as follows:

Running finance
Name of Balance as on Average
Bank Limit 30 June 2016 Balance Mark-up %
-------Rs. in million -------
Bank A 50 33 40 11%
Bank B 40 5 30 13%

Due to delay in supply of construction material, construction work was suspended from 1 November 2015 to 30
November 2015.

The following payments were made to the contractor net of 5% retention money which is refundable one year
after completion of the building:

Date of payment 1-06-2015 1-08-2015 1-12-2015 1-04-2016 1-08-2016


Net amount paid
(Rs. in million) 9.5 28.5 28.5 19.0 9.5

Surplus funds, if any, were invested @ 7% per annum.

Required:

Show how the above information would be disclosed in MML’s statement of financial position as on 30 June 2016
in accordance with the International Financial Reporting Standards.

(17)
(September 2016)

303
A.
Minhas Manufacturing Limited
Statement of Financial Position (Extracts)
As on 30-6-2016
2016
Rs. In
Million
Non-Current Assets
Factory Building (Not a CWIP now asset is now complete) (W-1) 106.943

Non-current Liabilities
Bank Loan 16.00

Current Liabilities
Current Portion of loan (16 x 2) 32.00
Short term running finance (33 + 5) 38.00
Bills payable (Payable to contractor) 9.5*
Retention money payable (9.5 + 28.5 +28.5 + 19 + 9.5) ÷ 95% x 5% 5.0*
Interest payable (48 x 10.9436% x 1/12) 0.44

Entry of last bills: (As the work is completed on 30-6-2016, therefore last bill should be accrued in this year).
CWIP-Building (9.5 ÷ 95%) 10
Retention money 0.5
Payable to contractor 9.5
W-1) Cost of building:
*Progress bills (10 + 30 + 30 + 20 + 9.5/95 x 100) 100
Interest on specific borrowings (w-4) 6.86
Investment income on surplus funds (w-5) (1.15)
Interest on general borrowings (w-6) 1.233
106.943
Not required; just for extra information:
As the work is commenced on 1-7-2015, therefore interest related to June 2015 and related investment income
should not be considered; i.e.
Interest incurred:
80 x 10.9436% x 1/12 = 0.73
Interest income:
(80 – 9.5) 70.5 x 7% x 1/12 = 0.41
The above two amounts should be recognized as interest expense and interest income respectively during the year
ended
30-6-2015.

W-2) Computation of capitalization rate:


Average bal. Interest rate Interest (A x B)
Bank A 40 11% 4.4
Bank B 30 13% 3.9
70 8.3
8.3/70 x 100 = 11.857%

304
W-3) Detail of payments:

Total Used from Used from


Date Particular Net Amount
Amount specific general
1-6-2015 1st Bill 10 9.5 9.5 -
1-7-2015 Current year
1-8-2015 2nd Bill 30 28.5 28.5 -
Repayment of loan (16 + 80 20.377 20.377
30-11-2015 x .109436 x 6/12) 20.377 -
1-12-2015 3rd Bill 30 28.5 21.623 6.877
1-4-2016 4th Bill 20 19 - 19
Repayment of loan (16 + 64 19.50 19.50
31-5-2016 x .109436 x 6/12) - 19.50

W-4) Specific Borrowings:


80 x 10.9436% x 4/12 (J, A, S, O) = 2.92
64 x 10.9436% x 6/12 (Dec to May) = 3.5
48 x 10.9436% x 1/12 (June) = 0.44
6.86

W-5) Investment Income on surplus funds:


70.5 (80-9.5) x 7% x 1/12 ( July) = 0.41
42(80-9.5-28.5) x 7% x 3/12 (A, S, O) = 0.74
21.623 (80-9.5-28.5-20.377) x 7% x 0/12 = -
1.15

W-6) Interest on General loan:


6.877 x 11.857% x 7/12 (Dec to June) = 0.48
19 x 11.857% x 3/12 (April to June) = 0.563
19.5 x 11.857% x 1/12 (June) = 0.19
1.233

W-7) Interest expense of November and investment income of November is ignored in above workings because of
suspension period

305
Test Cash Flow(Indirect Method) Date: -- / -- / --

Q. 1 MARVEL ENGINEERING LIMITED


Following are the extracts from the draft financial statements of Marvel Engineering Limited (MEL), a listed
company, for the year ended 30 June 2015:

Statement of Financial Position


Rs. in million
2015 2014 2015 2014
Non current assets Share capital and
reserves
Property, plant and equipment 633 410 Share capital (Rs. 10 each) 494 440
Long term investments 130 100 Share premium 8 -
Retained earnings 133 110
763 510 635 550
Current assets
Stock-in-trade 97 68 Non current liabilities
Trade debts 133 57 Long term loans 400 181
Other Assets 100 120
Cash at bank 31 39

361 284

Current liabilities

Trade and other 73 56


payables
Tax payable 12 5
Dividend payable 4 2
89 63
1,124 794 1,124 794

Statement of profit or loss


2015
Rs. in million
Revenue 654
Cost of sales (458)
Gross profit 196
Operating expenses (68)
Financial charges (75)
Other income 35

(108)
Profit before tax 88
Income tax expense (21)
Profit after tax 67

Additional information:
i. During the year, the company recognised depreciation for the year amounted to Rs. 61 million.

306
ii. It is the policy of the company to maintain a provision for doubtful debts at 5% of trade debts. During the
year, trade debts amounting to Rs. 6 million (2014: Rs. 2 million) were written off.
iii. Trade and other payables include accrued financial charges amounting to Rs. 7 million (2014: Rs. 3
million).
iv. MEL’s board of directors declared cash dividend of 5% on 15 July 2014 and 5% bonus declared on 20
December 2014. These shares were issued at the year end.
v. Other income comprises of the following:

Rs. m
Dividend income 30
Gain on sale of vehicles (carrying value of Rs. 5 million) 2
Gain on sale of investments (carrying value of Rs. 10 million) 3
35
Required
Prepare the statement of cash flows for Marvel Engineering Limited for the year ended 30 June 2015.
(15)
Q.2 APOLLO INDUSTRY LIMITED
Following are the relevant extracts from the financial statements of Apollo Industry Limited, a listed company,
for the year ended December 31, 2015.
Statement of financial position as at December 31, 2015
2015 2014
Rs. 000 Rs. 000
Issued, subscribed and paid up capital 25,000 20,000
Unappropriated profit 20,900 22,000
Surplus on revaluation of property, plant & equipment 7,000 8,000
52,900 50,000
Non-current liabilities
Long term Loan 1,990 1,190
Trade and other payables 4,200 6,250
59,090 57,440
Non current Assets:
Property, plant and equipment 40,500 35,500
Intangible assets 1,100 1,140
41,600 36,640
Long term deposits and prepayments 400 650
42,000 37,290
Current Assets
Tax refundable / Advance tax 950 800
Other current assets 15,700 12,125
Cash and bank balances 440 7,225
17,090 20,150
59,090 57,440

Statement of comprehensive Income for the year ended December 31, 2015
2015
Rs. 000
Sales 146,700
Cost of sales (127,500)
Gross profit 19,200

Operating expenses (15,000)

307
Financial charges (500)
Other income 2,800
(12,700)
Profit before tax 6,500
Tax for the year (5,600)
Profit after tax 900

Other relevant information is as under:


i. During the year, the company has issued 10% bonus shares.
ii. Depreciation and amortization for the year amounted to Rs. 7 million.
iii. WDV of assets disposed off during the year amounted to Rs. 1.2 million. (The assets had not been revalued).
During the year assets were not revalued because there was no significant difference between carrying
amount and fair value.
iv. Other income includes interest earned on short term placements, amounting to Rs. 1 million. The remaining
amount represents gain on disposal of property, plant and equipment.
v. Intangible assets worth Rs. 50 thousand were acquired during the year.

Required
Prepare the Statement of Cash Flows for the year ended December 31, 2015 in accordance with the requirements
of IAS - 7 (Statement of Cash Flows) using ‘indirect method’. (18)

308
A1
Marvel Engineering Limited
Cash Flow Statement
For the year ended 30 June 2015
2015
Cash flows from operating activities Rs. in million
Profit before taxation 88.00
Adjustment for non-cash charges and other items:
Depreciation 61.00
Financial charges 75.00
Bad debts(6+4) 10.00
Gain on sale of fixed assets (2.00)
Gain on sale of investments (3.00)
Dividend income (30.00)
Working capital changes
Stock-in-trade (97 - 68) (29.00)
Trade debts (86.00)
Other assets (100 - 120) 20.00
Trade and other payables (73 – 7) – (56 - 3) 13.00
Cash generated from operations 117.00
Financial charges paid (3 + 75 - 7) (71.00)
Income tax paid (14.00)
Net cash generated from operating activities 32.00

Cash flows from investing activities


Purchase of property, plant and depreciation (289.00)
Proceeds from sale of property, plant and equipment (5+2) 7.00
Proceeds from sale of investments (10+3) 13.00
Purchase of long term investments (130-100+10) (40.00)
Dividend received 30.00
Net cash used in investing activities (279.00)

Cash flows from financing activities


Insurance of ordinary shares (8+32) 40.00
Proceeds from long term loan (400 - 181) 219.00
Payment of dividend (2 + (440 x 5%) - 4) (20.00)
Net cash from financing activities 239.00
Net decrease in cash and cash equivalents (8.00)
Cash and cash equivalent at the beginning of the year 39.00
Cash and cash equivalent at the end of the year 31.00

WORKINGS (All amount in million rupees)

Provision for Trade


bad debts Debtors
Closing balance (133 ÷ 0.93)- 133 7.00 (133 ÷ 0.95) 140.00
Add: Bad debts written off 6.00 6.00
Less: Opening balance (57÷ 0.95)- 57 (3.00) (57 ÷ 0.95) (60.00)

309
10.00 86.00

PPE-WDV Share Capital


b/d 410 b/d 440
Depreciation 61 Dividend 22
Cash 289 Disposal 5 c/d 494 Cash 32
c/d 633

Retained Earnings
Dividend (Cash) 22 b/d 110 Dividend Payable
(440*5%) b/d 2
Dividend (Bonus) 22 Profit after tax 67 Cash 20 Dividend 22
(440*5%) c/d 4
c/d 133

Share Premium Interest Payable


b/d - b/d 3
c/d 8 Cash 8 Cash 71 Expense 75
c/d 7
Long term investment
Income Tax Payable b/d 100 disposal 10
Cash 14 b/d 5 Cash 40 c/d 130
c/d 12 Expense 21

Debtors Provision
b/d 60 Bad 6 b/d 3
debts Bad 4
Increase 86 c/d 140 debts
c/d 7
A.2
Apollo Industry Limited
Statement of cash flows
For the year ended December 31, 2015 Rs. in ‘000
Cash used in operating activities
Profit before taxation 6,500
Adjustment for: (non-cash items / separately disclosed items)
Depreciation for the year (7,000-90) 6,910
Amortization for the year 90
Profit on sale of fixed assets (2,800-1,000) (1,800)
Interest income (1,000)
Interest Expense 500
Operating profit before working capital changes 11,200
Trade and other payable(6,250-4,200) (2,050)
Other current assets(12,125-15,700) 3,575)
Cash generated from operations 5,575
Payment for taxation (950 + 5,600 - 800) (5,750)
Interest paid (500)
Net cash from operating activities (675)
Cash used in investing activities

310
Purchase of PPE (13,110)
Proceeds from sale of PPE (1,200 + 1,800) 3,000
Acquisition of intangible assets (50)
Interest received 1,000
Receipt from Long term deposits (400-650) 250
(9,410)
Cash used in financing activities
Issue of ordinary share capital (25,000-2,000-20,000) 3,000
Dividend paid (1,000)
Long term loan(1,990-1,190) 800
2,800
Net decrease in cash and cash equivalents (6,785)
Opening balance: cash and cash equivalents 7,225
Closing balance: cash and cash equivalents 440

Property, plant & equipment


b/d 35,500 Disposal 1,200
Depreciation 6,910
(7,000 - 90)
Cash 13,110
c/d 40,500

Intangible Assets
b/d 1,140 Amortization(bal) 90
Cash 50 c/d 1,100

Tax
b/d 800 Expense 5,600
Paid (Bal) 5,750 c/d 950

Retained Earning
Dividend(S.C) 2,000* b/d 22,000
Dividend(bal) 1,000 PAT 900
c/d 20,900 R.S 1,000

*(20,000*10%)
Disposal A/C
PPE 1,200 Cash(bal) 3,000
Gain 1,800

Share Capital
b/d 20,000
Dividend 2,000
Cash (Bal) 3,000
c/d 25,000
Revaluation surplus
R.E(bal) 1,000 b/d 8,000
c/d 7,000

311
Test Ratios Date: -- / -- / --

Q.1 Progressive Steel Limited (PSL) commenced business in 2015. The following comparative data pertains
to the year ended 30 June 2017:

PSL Industry
Description
2017 2016 2017
Gross profit margin 13% 13% 16%
Net profit margin 8% 7% 10%
Return on shareholders’ equity 22% 18% 25%
Current ratio 1.2 1.6 1.5
Debt to equity ratio 40:60 30:70 50:50
Cash operating cycle in days 119 135 118

Required:
For each ratio/data give possible reasons for variation from comparative and industry data. (12)

312
Ans.

Ans.1 Reasons for fluctuation with


Ratios Reason for fluctuation with Industry
previous year
Gross profit In line with previous year. No variation. Lower than industry
margin The company is in initial phase and may
have kept the selling prices lower than
the
industry to gain the market share.
The company may not have been able to
purchase raw material at prices which is
available to its competitors.
The company may not have been able to
obtain economies of scale in its
production which may have been
obtained
by its competitors.
Possibility of higher production costs.
Lower than industry however, the
Net profit Higher than previous year: difference
margin Tight control over operating costs. is mainly attributed to lower gross profit
Increase in other income. margin.
Decrease in fixed cost per unit due to
increase in sale.
Return on Higher than previous year: Lower than industry
Reduction in tax
shareholder's rates. Lower gross profit and net profit margins.
equity Reduction in interest rates. Lower leverage.
Decrease in equity might be due to Higher net assets resulting in higher
buyback of shares. equity.
Distribution of profits from previous year
which resulted in decrease in equity.
(means more dividend)
Increase in profit due to increase in
. revenue or decrease in expenses
Current ratio Lower than previous year: Lower than industry
The company might have obtained Since the debt equity ratio is lower than
running finance facility to fund it's the industry, company might have
operations in the current year. obtained running finance or might have
availed extended credit terms from
Long term loan payments might have
suppliers.
become due in the next 12 month, which
decreases the current
ratio.

Decrease in current assets due to better


inventor
y management/ reduction in
credit period of debtors.
Debt to equity Higher than previous year Lower than industry
ratio Decrease in reserves due to dividend pay- Being a new entrant the company may be
out. in the phase of expansion thereby raising

313
Further debt obtained during the period. debt accordingly.
. Less loans than an average company in this
Decrease in equity might be due to industry.
buyback of shares.
Cash operating Lower than previous year In line with industry.
cycle Increase in creditor days might be due
to increase in credit
period.
Decrease in stock turnover days which
might be
due to greater stock turnover or better
inventory management
(means stock turnover
days are lower)

By giving lower credit days to debtors.

314
Test Ratios Date: -- / -- / --
Q.1
An investor wants to analyze the performance of Zee Limited for which he has collected the following information
for the year ended 30 June 2015 and 2014:

2015 2014
Rs. In million
Profit after interest and tax 100.00 75.00
Interest expense at 12% per annum on a long-term loan acquired on 1 January
2014 (9.60) (4.80)
Tax expense (44.00) (43.00)
Interim bonus issue 12% 10%
Final cash dividend (2013: 30%) 20% 25%

The break-up of shareholders’ equity as at 1 July 2013 was as under:

Rs. In million
Share capital (Rs. 10 each) 200
Share premium 20
Retained earnings 40
260

Required:
Compute Return on Capital employed and Return on shareholders’ Equity for the year ended 30 June 2015. (07)

315
A.1
(i) Return on capital employed

Rs. In million
Average capital employed as at 30 June 2015:
Average equity (W-1) (275 + 320) ÷ 2 297.50
Average long term loan (9.6 ÷ 12%)=80+80/2 80.00
A 377.50
Profit before interest and tax (100+9.6+44) B 153.60
Return on capital employed B÷A 40.69%

(ii) Return on shareholders’ equity

Rs. In million
Average shareholders’ equity C 297.50
Profit after interest and tax D 100.00
Return on shareholders’ equity D÷C 33.61%

(W-1) Average equity

Share Share Retained Total


Capital Premium Earnings Equity
Balance as at 1 July 2013 200.00 20.00 40.00 260.00
Final cash dividend for 2013 at 30% (200 x 30%) - - (60.00) (60.00)
10% Interim bonus issue 20.00 (20.00) -
Profit after tax for the y.e. 30 Jun. 2014 - - 75.00 75.00
Balance as at 30 June 2014 220.00 - 55.00 275.00
Final cash dividend for 2014 at 25% (220 × 25%) - - (55.00) (55.00)
12% Interim bonus issue (220 × 12%) 26.40 - (26.40) -
Profit after tax for the y.e. 30 Jun. 2015 - - 100.00 100.00
Balance as at 30 June 2015 246.40 - 73.60 320.00

316
Spring 2020
Q.2 You are working as Finance Manager in Broad Peak Limited (BPL). Faraz has recently joined BPL as an
internee for three months. You have asked him to develop an understanding of the statement of cash
flows. After going through few statements, he has raised the following queries:

(i) Depreciation is not a cash flow but was still appearing as an addition in the statement of cash
flows.
(ii) In the statement of cash flows of a competitor, interest paid was shown as a financing activity but
BPL showed it in operating activities.
(iii) BPL purchased inventories throughout the year but total purchases of inventory were not shown in
the statement. However, only decrease in inventory was added.
(iv) Cash and bank balance in the statement of financial position was not in agreement with the
opening and closing balances at the end of statement of cash flows.

(v) In statement of cash flow, tax paid is deducted in operating activities but tax expense is not added
back I n profit before tax.

(vi) The working capital changes does not include increase/decrease in all current assets and current
liabilities.

(vii) What is the treatment of transactions between the components of cash and cash equivalentse.g.
cash deposited /withdrawn from bank.

Required:
Briefly answer the queries raised by Faraz. (08)

Q.3 Briefly describe the measurement bases that may be used to measure the value of assets in the financial
statements. (06)

317
Q.4 Select the most appropriate answer(s) from the options available for each of the following Multiple
Choice Questions (MCQs).

(i) Which of the following companies is most likely to face cash flow problems?

(a) A loss making government organisation


(b) A company which has recently sold part of its operations so as to concentrate on its core
areas

(c) A reasonably profitable and long established company with no expansion plans
(d) A profitable retailer about to embark on ambitious expansion plans (01)

(ii) A plant has a carrying amount of Rs. 1,500,000 as at 31 December 2019. Its fair value is Rs. 900,000
and costs of disposal are estimated at Rs. 50,000. A new plant would cost Rs. 2,500,000. Cash flows
from the plant for the next four years are estimated at Rs. 350,000 per annum. Applicable discount
rate is 10%.

What is the approximate impairment loss on the plant to be recognised in the financial statements
as at 31 December 2019?

(a) Rs. 650,000 (b) Rs. 390,000


(c) Rs. 1,000,000 (d) Nil (02)

(iii) A debit balance on the retained earnings account indicates that the company has:
(a) made more dividend payments than the profit earned
(b) redeemed some of its share capital
(c) accumulated losses
(d) issued bonus shares (01)

(iv) The correct accounting treatment of initial operating losses incurred during the commercial
production due to under-utilization of the plant would be to:

(a) capitalise as a directly attributable cost


(b) defer and charge to profit or loss account when profit is earned from the plant
(c) charge directly to retained earnings since these are not considered to be normal operating
losses
(d) charge to profit or loss account (01)
(v) In measuring value in use, the discount rate used for discounting the cash flows
should be the:

(a) pre-tax rate that reflects the market assessment of time value of money and
risks specific to the asset
(b) pre-tax rate that reflects the market assessment of time value of money and
risks specific to the entity
(c) post-tax rate that reflects the entity’s assessment of time value of money and
risks specific to the asset
(d) pre-tax rate that reflects the entity’s assessment of time value of money and risks
specific to the asset (01)

(vi) Which of the following is NOT considered as an item of property, plant and
equipment?

318
(a) A standby generator expected to be used for seven years
(b) A plot of land held for resale
(c) A bus for pick and drop of staff members
(d) A generator for rental to others (01)

(vii) Capitalization of borrowing costs should be suspended:


(a) when substantially all the activities necessary to prepare a qualifying asset for its
intended use or sale are complete
(b) during a temporary delay which is a necessary part of the process of getting an asset
ready for its intended use or sale
(c) during extended periods in which active development of a qualifying asset is
interrupted
(d) all of the above (01)

Q.6
Following are the summarised financial statements of Shispare Limited (SL) and its competitor Trivor
Limited (TL) for the year ended 31 December 2019:

Statement of financial
position
SL TL SL TL
Assets Equity & liabilities
Rs. in million Rs. in million
Fixed assets 5,400 7,800 Capital and reserves 8,400 9,450
Current assets: Long-term loan 1,900 4,600
Inventory 4,800 7,100 Current liabilities:
Debtors 2,700 3,200 Creditors 2,900 4,500
Cash 1,200 800 Accrued expenses 900 350
8,700 11,100 3,800 4,850
14,100 18,900 14,100 18,900

Statement of profit or loss


SL TL
--- Rs. in million ---
Sales 16,700 35,400
Cost of goods sold (11,400) (27,800)
Gross profit 5,300 7,600
Operating expenses (3,500) (4,900)
Finance cost (250) (600)
Net profit 1,550 2,100

Required:
Compute relevant ratios for SL and TL to assess which company seems to:
(i) Give more incentives to its customers to pay on time
(ii) Avail extended credit terms from its suppliers
(iii) Be more efficient in the use of capital
(iv) Keep lower selling prices to gain the market share

319
(v) Have better liquidity position
(vi) Have higher ability to convert its assets into profit
(vii) Control operating expenses more efficiently
(viii) Have higher ability to raise bank loan in future (16)

Q.8
Following information pertains to non-current assets of Distaghil Limited (DL):

(i) DL purchased specialised vehicles for Rs. 370 million on 1 July 2017. The vehicles have an
estimated useful life of 10 years with residual value of Rs. 30 million.

The revalued amounts of the vehicle as at 31 December 2018 and 2019 were determined at Rs.
302 million and Rs. 290 million respectively. There was no change in useful life or residual value.

(ii) DL setup a manufacturing plant in a remote area at a cost of Rs. 280 million. The plant had a useful
life of 8 years. The plant was purchased on 1 January 2018 and was available for use on 1 April
2018. The commercial production started on 1 June 2018.

On 1 July 2018, DL received a government grant of Rs. 120 million towards the cost of the plant.
The sanction letter states that if DL ceases to use the plant in the remote area before 31 December
2021, DL would be required to repay the grant in full.

(iii) A warehouse was given on rent on 1 January 2018. Previously, the warehouse was in use of DL.

On 1 January 2018, carrying value and remaining useful life of the warehouse was Rs. 80 million
and 16 years respectively. Fair value of the warehouse on various dates are as follows:

Rs. in million
01 January 2018 104
31 December 2018 96
31 December 2019 115

Other information:
DL uses cost model for subsequent measurement of property, plant and equipment
except for specialised vehicles for which revaluation model is used.
DL transfers the maximum possible amount from the revaluation surplus to retained earnings on
an annual basis.
Government grant is recorded as deferred income and a part of it is transferred to income each
year.
Investment property is carried at fair value model.

Required:
Prepare relevant extracts from DL’s statement of profit or loss and other comprehensive income for the
year ended 31 December 2019 and statement of financial position as on that date. (Show comparative
figures) (20)

320
A.2
(i) A statement of cash flows begins with net profit which is arrived after deducting depreciation expense.
So to convert the net profit into net cash flow the deduction of depreciation is reversed (i.e. added).
(ii) As per IAS 7, interest paid can be shown as either cash flow from financing activities or cash flow from
operating activities. Both classifications are correct as long as they are consistently applied by an entity.
(iii) A statement of cash flows begins with net profit which is arrived after deducting cost of sales. So to
convert the effect of cost of goods sold into outflow for purchases of inventory, change in inventory is
adjusted i.e. increase is deducted and decrease is added.
(iv) Statement of financial position shows cash and bank balances while the statement of cash flows ends
with cash and cash equivalents which may differ from cash and bank balances due to existence of bank
overdraft and short term investments.
(v) As the operating activities are started with profit before tax and tax expense is therefore not deducted
so there is no reason to add back the tax expense.
(vi) The purpose of working capital changes is to convert the items of income and expenses within the
profit before tax into receipts and payments indirectly. Therefore,
 Debtors and advance from customers should be considered to convert sale into receipts from
customers.
 Inventories should be considered to convert cost of sales in to purchase.
 Creditors and advance to suppliers should be considered to convert purchases into payment to
suppliers.
 Similarly expenses payable and prepaid expenses should be considered to convert expenses into
payment for expenses.
Other items e.g. interest payable, tax payable and dividend payable should not be considered because total
payment is to be presented.
Cash and cash equivalents should be considered at the end of the statement of cash flow. Short term
investments should either be classified as investing activities or cash and cash equivalents.
Interest and dividend receivable should not be considered because their total recipt is to be presented in
investing activities.
(vii) These transactions cancel out each other therefore not presented anywhere in the statement of cash
flows.
A.3
(i) Historical cost
The historical cost of an asset, when it is acquired or created is the value of the cost incurred in acquiring or
creating the asset, comprising the consideration paid to acquire or create the asset plus transaction cost.

Current value
Current value measures provide monetary information about assets using information updated to reflect
conditions at the measurement date.
Current value measurement bases include:
– Fair value
– Value in use for assets
– Current cost

321
Fair value
Fair value is the price that would be received to sell an asset in an orderly transaction between market
participants at the measurement date. Fair value reflects the perspective of market participants.

Value in use
Value in use is the present value of the cash flows or other economic benefit that an entity expects to derive from
the use of an asset and from its ultimate disposal. Value in use reflect entity specific assumptions rather than
assumptions by market participants.
Current cost
The current cost of an asset is the cost of an equivalent asset at the measurement date comprising the
consideration that would be paid at the measurement date plus the transaction cost that would be incurred at
that date.
Current cost, like historical cost is an entry value; while fair value is an exit value. However, unlike historical cost,
current cost reflects conditions at the measurement date

A.4

The most appropriated answer(s) for Multiple Choice Questions (MCQs)

(i) (d) A profitable retailer about to embark on ambitious expansion plans.


(ii) (b) Rs. 390,000 (W – 1)
(iii) (c) accumulated losses
(iv) (d) charge to profit or loss account
(vi) (a) Pre-tax rate that reflects the market assessment of time value of money and risks specific to
the asset
(vii) (b) A plot of land held for resale
(viii) (c) during extended periods in which active development of a qualifying asset is interrupted.

Workings:

W- 1:
Ans:
Option (b)

31-12-2019
Carrying Amount 1,500,000

Recoverable Amount:
Higher of:
Value in use:

1− (1 + 0.1)−4
[350,000 x ] = 1,109,453
0.1

FV less CTS:
[900,000 – 50,000] = 850,000
Higher of: 1,109,453
Impairment loss (1,500,000 – 1,109,453) = 390,548 (which approximates to 390,000)

322
W – 2:
Ans:
Option (b)
As total in all cost types is different, therefore non of the cost is 100% fixed cost.
Cost per unit in case of A is constant at all levels i.e.
(i) 1,500 ÷ 100 = 15/unit; and
(ii) 2,250 ÷ 150 = 15/unit

Therefore, it is 100% variable cost similarly cost per unit in case of C is constant at all levels, i.e.
(i) 2,000 ÷ 100 = 20/unit; and
(ii) 3,000 ÷ 150 = 20/unit

There it is 100% variable as well


However, in case of B and D cost per unit is also not constant, i.e.

B:
(i) 1,800 ÷ 100 = 18/unit; and
(ii) 2,400 ÷ 150 = 16/unit

Similarly;

D:
(i) 3,000 ÷ 100 = 30/unit; and
(ii) 4,200 ÷ 150 = 28/unit
Therefore, cost type B and D are semi-variable cost.

A.6
Relevant ratios SL TL
(i) Debtors collection period 59.01 days 32.99 days
Debtors 2,700 3,200
= ×365 = ×365
= ×365 16,700 35,400
Sales
TL is giving more incentives to its customers to
pay on time.
(ii) Creditors payment period 92.85 days 59.08 days
Creditors
= ×365 2,900 4,500
Purchases/Cost of sales = ×365 = ×365
11,400 27,800
SL avail extended credits terms
(iii) Return on capital 17.48% 19.22%
Profit before interest 1,550+250 2,100+600
= ×100 = ×100 = ×100
Capital employed 8,400+1,900 9,450+4,600
TL is more efficient in the use of capital
(iv) Gross profit margin 31.74% 21.47%
Gross profit 5,300 7,600
= ×100 = ×100
= ×100 16,700 35,400
Sales
TL is deliberately keeping selling prices lower to
gain the market share.
(v) Current ratio 2.29 2.29

323
Current assets 8,700 11,100
= = =
Current liabilities 3,800 4,850
Quick ratio 1.03 0.82

Current assets-inventory 8,700 − 4,800 11,100 − 7,100


= = =
Current liabilities 3,800 4,850
SL has better liquidity position
(vi) Return on assets 12.77% 14.29%
Profit before interest 1,550+250 2,100+600
= ×100 = ×100 = ×100
Total assets 14,100 18,900
TL has higher ability to convert its assets into profit

(vii) Operating expenses %age 20.95% 13.84%


Operating expenses 3,500 4,900
= ×100 = ×100
= ×100 16,700 35,400
Sales
TL is efficiently controlling the operating expenses.

(viii) Gearing ratio 0.18% 0.33%


Debt 1,900 4,600
= = ×100 = ×100
Debt + Equity 8,400 + 1,900 9,450 + 4,600

SL is going to raise a bank loan relatively


easily in future

A.8
Distaghil Limited
Extracts from statement of financial position as on 31 December 2019
2019 2018
---- Rs. in million ----
Non-current assets:
Property, plant and equipment:
 Vehicles 290.00 302.00
 Plant (253.75 – 35) / (280 – 26.25) 218.75 253.75

Investment property 115.00 96.00

Share capital and reserves:


Revaluation surplus:
 Vehicles 5.00 -
 Warehouse (22.5 – 2.4) / (24 – 1.5) 21.00 22.50

Non-current liabilities:
Deferred government grant (108.75 – 15-15) / (120 – 11.25-15) 78.75 93.75
Current liabilities
Deferred government grant 15.00 15.00

324
Extracts from statement of profit or loss and other comprehensive income For
the year ended 31 December 2019
2019 2018
------ Rs. in million ------
Profit or loss:
Depreciation:
 Vehicles (32.00) (34.00)
 Plant (35.00) (26.25)

Revaluation loss – vehicles - (17.00)


Revaluation gain – vehicles 15.00
Grant income – Plant 15.00 11.25
Change in fair value of investment property - warehouse
(2019: 115–96), (2018: 96–104) 19.00 (8.00)

Other comprehensive income:


Revaluation surplus:
 Vehicles 5.00
 Warehouse (104 – 80) - 24.00
Workings:
i) Vehicles:
1-7-2017
Vehicle 370
Cash 370

31-12-2017 370 - 30/10 = 34 x 6/12 = 17


Dep 17
Acc. dep 17

31-12-2017
Dep 34
Acc. Dep 34

31-12-2018
Acc. dep 51
Vehicle 51

R. Loss 17
Vehicle 17

WDV (370 – 51) = 319


FV = 302
R. Loss 17

31-12-2019
Dep 32
Acc. Dep 32
302 – 30/8.5 = 32

325
31-12-2019
WDV (302 – 32) = 270
FV = 290
R.S 20

Acc. dep 32
Vehicle 32
Vehicle 20
R.S (OCI) 5
Reversal of loss 15

Reversal of loss 17Cr.


Extra dep (34 - 32) 2Dr.
Net reversal of loss 15 Cr.
ii) Plant:
1-1-2018
Plant 280
Cash 280

1-7-2018
Cash 120
Deferred Income 120

31-12-2018
Dep 26.25
Acc. dep 26.25
(280/8 x 9/12) =

31-12-2018
Deferred income 11.25
Other income 11.25
(120 ÷ 8 x 9/12) =

31-12-2019
Dep 35
Acc. Dep 35
(280/8)

31-12-2019
Deferred Income 15
Other Income 15
(120 ÷ 8)

iii) Investment Property:


1-1-2018
Warehouse 24
R. Surplus (OCI) 24

C.A = 80
24
FV = 104

326
1-1-2018
Investment Property(WH) 104
Warehouse (PPE) 104

31-12-2018
FV Loss (I.S) 8
Investment Property 8

C.A = 104
8 Loss
FV = 96

31-12-208
R. Surplus 1.5
R. Earnings 1.5 (24 ÷ 16)

31-12-2019
C.A = 96
19 Gain
FV = 115
Investment property 19
FV gain (P.L) 19

31-12-2019
R. Surplus 1.5
R. Earnings 1.5

327
Test of Cash flow:
Q.1 The following information has been extracted from the draft financial statements of Alpha Limited for the year
ended 31 December 2015.
Assets 2015 2014 Equity & Liabilities 2015 2014
Rs. In millions Rs. In millions
Property Plant & Equipment 223 193 Share capital (Rs. 10 each) 180 150
Intangible assets 68 23 Share Premium 15 -
Trade receivables 45 33 Retained earnings 114 53
Advances and prepayments 84 70 Long term loan 40 -
Inventories 60 46 Deferred Liabilities 15 10
Short term investments 12 9 Trade Payables 42 56
Cash at bank 8 7 Accrued expenses 60 70
Tax payable 34 42
500 381 500 381

Following relevant information is available:


i. Depreciation has been provided on straight line basis. Estimated useful lives are as under:

Building 20 years
All other fixed assets 10 years

ii. On 1 September 2015, the company purchased new machinery costing Rs. 65 million.
iii. A portion of building costing Rs. 20 million which was purchased on 1 July 2013 was sold for Rs. 20 million
on 30 June 2015.
iv. Trade receivables written off during the year amounted to Rs. 5 million. It is the policy of the company to
maintain the provision for doubtful debts at 5% of trade receivables.
v. Advances and prepayments include advance tax of Rs. 8 million (2014: Rs. 6 million).
vi. Long term loan was obtained on 1 August 2015. Interest on loan @ 13% is payable on 31 July each year.
Interest payable for 5 months has been accrued.
vii. Deferred liabilities comprise of unfunded gratuity of Rs. 6 million (2014: Rs. 3 million) and deferred tax of
Rs. 9 million (2014: Rs. 7 million). During the year, the company paid gratuity of Rs. 6.5 million to outgoing
employees.
viii. Tax expense for the year was Rs. 17 million. (2014: Rs. 8 million),
ix. Right shares were issued on 1 December 2015 at Rs. 15 per share in the ratio of 1 right share for every 5
shares held.
Required:
Prepare statement of cash flows for the year ended 31 December 2015 in accordance with the requirements of
International Financial Reporting Standards using the indirect method. (15)

328
A.
Alpha Limited Statement of Cash Flow For
the Year ended 31-12-2015
Cash flow from Operating Activities:
Profit before tax (61 + 17) 78
Interest Expense 2.17
Depreciation 17.00
Gain on disposal (2.00)
Bad debts 5.63
Gratuity Expense 9.5
Profit before working capital changes 110.3
Working Capital Changes:
Trade debtors (17.63)
Inventories (60 – 46) (14.00)
Advances and Prepayments (12.00)
Trade payables (42 – 56) (14.00)
Accrued expenses (12.17)
Cash generated from operations 40.5
Cash generated from Operations:
Tax Paid (25)
Gratuity Paid (6.5)
Net Cash From Operating Activities 9.00
Cash Flow From Investing Activities:
Purchase of machinery (65)
Receipts from disposal 20
Acquisition of intangibles (45)
Purchase of investments (12-9) (3)
Net Cash Flow From Investing Activities (93)
Cash Flow From Financing Activities:
Proceeds from issue of right shares (150/10 = 15 x 1/5 x 15) 45
Proceeds from long term loan 40
Net Cash flow 1
Cash and Cash equivalent at the beginning of the period 7
Cash and Cash equivalent at the end of the period 8

Workings:
PPE-WDV Intangibles
b/d 193 Disposal (20 – 2) 18 b/d 23
Cash 65 Dep (bal.) 17 45
c/d 223 c/d 68

Receivables Provision
b/d 34.74 Prov 5 Debtors 5 b/d 1.74
17 5.63
c/d 47.37
.6
33/95% = 34.74 c/d 2.37
3
45/95% = 47.37

329
Advance Advance tax + tax payable
b/d 64 b/d 6 b/d 42
(70 – 6) Exp. (17 – 2) 15
c/d 76 Cash 25
12
(84 – 8)
.6
c/d 34 c/d 8
3
Share Capital Share Premium
b/d 150 b/d

30 25
c/d 180 c/d 15

Retained Earnings Loan


b/d 53 b/d -

PAT 61 40
c/d 114 c/d 40

D.T.L Provision for Gratuity


b/d 7 b/d 3
Cash 6.5
2 Exp 9.5
c/d 9 c/d 6

Accrued Expenses Interest Payable


b/d 70 b/d -

12.17 2.17
c/d 57.83 c/d 2.17
(60 – 2.17) (40 x 13% x 5/12 = 2.17)

Disposal A/c
Cash 20
PPE 18

Gain
2

330

You might also like